Вы находитесь на странице: 1из 133

The Official SAT Online Course 1/1

Help | Profile | My Organizer | My Bookmarks | Logout

Answers and Explanations

Test Sections Back to Score Report

Section 1 Essay
Section 2 Online - Practice Test #3

Section 3
These sample essays were originally handwritten by students but are shown typed here for ease
Section 4 of reading. The essays are displayed exactly as students wrote them, without any corrections to
Section 5 spelling, punctuation, or syntax. One handwritten sample essay is provided to illustrate the need
for legible and clear handwriting.
Section 7
Section 8
Section 9 Exemplars:
Section 10 Essay Prompt

Think carefully about the issue presented in the following excerpt and the assignment below.


The people we call heroes do not usually start out as unusual. Often they are ordinary people
subject to ordinary human weaknessesfear, doubt, and self-interest. In fact, they live
ordinary lives until they distinguish themselves by having to deal with an injustice or a difficult


d
situation. Only then, when they must respond in thought and in action to an extraordinary
challenge, do people begin to know their strengths and weaknesses.

re
Do people learn who they are only when they are forced into action? Plan and write an essay
in which you develop your point of view on this issue. Support your position with reasoning
and examples taken from your reading, studies, experience, or observations.


te

is


eg

Back to Score Report

Copyright 2006 The College Board. All rights reserved. Privacy Policy Terms of Use Contact Us


nR

file://E:\\c2.htm 2006-11-12
The Official SAT Online Course 1/22

Help | Profile | My Organizer | My Bookmarks | Logout

Answers and Explanations

Test Sections Back to Score Report

Section 1 View Answers and Explanations


Section 2 Online - Practice Test #3

Section 3
1
Section 4
Responding to criticism that the script was rambling and -------, the new screenwriter
Section 5
revised the dialogue for greater succinctness and -------.
Section 7
(A) engaging. . simplicity
Section 8
Section 9 (B) subjective. . ambiguity
Section 10 (C) muddled. . clarity

(D) terse. . emptiness

(E) difficult. . abstraction



ANSWERS AND EXPLANATIONS

d
re
Explanation for Correct Answer C :


Choice (C) is correct. "Muddled" means confused or aimless. "Clarity" means
clearness. If one were to insert these terms into the text, the sentence would read
"Responding to criticism that the script was rambling and muddled, the new


e
screenwriter revised the dialogue for greater succinctness and clarity." The missing
terms describe a second negative quality of the script and a corresponding positive
st
quality of the revised version. A muddled script would need to be revised for clarity.


eg

Explanation for Incorrect Answer A :


Choice (A) is incorrect. "Engaging" means interesting. "Simplicity" means


uncomplicated. If one were to insert these terms into the text, the sentence would
nR

read "Responding to criticism that the script was rambling and engaging, the new
screenwriter revised the dialogue for greater succinctness and simplicity." The


missing terms describe a second negative quality of the script and a corresponding
positive quality of the revised version. A screenplay would not be criticized for being
engaging.

Explanation for Incorrect Answer B :


Choice (B) is incorrect. "Subjective" means dependent on personal taste or views.
"Ambiguity" means uncertainty. If one were to insert these terms into the text, the
sentence would read "Responding to criticism that the script was rambling and
subjective, the new screenwriter revised the dialogue for greater succinctness and
ambiguity." The missing terms describe a second negative quality of the script and
a corresponding positive quality of the revised version. If a screenplay were
criticized for being subjective, the screenwriter would probably revise it to make it
less ambiguious.

Explanation for Incorrect Answer D :


Choice (D) is incorrect. "Terse" means brief or succinct. "Emptiness" means, in this
context, meaninglessness. If one were to insert these terms into the text, the
sentence would read "Responding to criticism that the script was rambling and
terse, the new screenwriter revised the dialogue for greater succinctness and
emptiness." The missing terms describe a second negative quality of the script and
a corresponding positive quality of the revised version. A screenplay cannot be both
rambling and terse, and one would not revise it to give it greater emptiness.

Explanation for Incorrect Answer E :


Choice (E) is incorrect. "Difficult" means hard. "Abstraction" means theoretical. If
one were to insert these terms into the text, the sentence would read "Responding
to criticism that the script was rambling and difficult, the new screenwriter revised
the dialogue for greater succinctness and abstraction." The missing terms describe
a second negative quality of the script and a corresponding positive quality of the
revised version. If a screenplay is difficult, then adding more abstraction would only
make it more difficult.

file://E:\\c3.htm 2006-11-12
The Official SAT Online Course 2/22

2 During the 1990s, Shanghai benefited from an architectural -------, the result of a
dramatic increase in innovative and artistic building.
(A) intransigence
(B) plentitude
(C) desecration

(D) stagnation

(E) renaissance

ANSWERS AND EXPLANATIONS


Explanation for Correct Answer E :
Choice (E) is correct. "Renaissance" means a rebirth or revival. If one were to insert
this term into the text, the sentence would read "During the 1990s, Shanghai
benefited from an architectural renaissance, the result of a dramatic increase in
innovative and artistic building." An "architectural renaissance" describes a period
of renewed excitement in the field of architecture, and properly describes the result
of a dramatic increase in new artistic construction.



ed
Explanation for Incorrect Answer A :
Choice (A) is incorrect. "Intransigence" means being uncompromising or stubborn.
If one were to insert this term into the text, the sentence would read "During the


1990s, Shanghai benefited from an architectural intransigence, the result of a
dramatic increase in innovative and artistic building." It is unlikely that

r
"intransigence" would be the result of an increase in architectural innovation;
te

innovation is in some ways the opposite of stubbornness.

Explanation for Incorrect Answer B :


is
Choice (B) is incorrect. A "plenitude" means an abundance. If one were to insert
this term into the text, the sentence would read "During the 1990s, Shanghai
benefited from an architectural plenitude, the result of a dramatic increase in


eg

innovative and artistic building." An abundance of architecture would be the result


of an increase in construction. But this would not account for the "innovative and
artistic" quality of the architecture.


Explanation for Incorrect Answer C :
nR

Choice (C) is incorrect. A "desecration" means an outrageous mistreatment of


somethingthe vandalizing of a sacred building, for instance. If one were to insert
this term into the text, the sentence would read "During the 1990s, Shanghai
benefited from an architectural desecration, the result of a dramatic increase in
innovative and artistic building." Desecration would not benefit a city.

Explanation for Incorrect Answer D :


Choice (D) is incorrect. "Stagnation" means a lack of movement or development. If
one were to insert this term into the text, the sentence would read "During the
1990s, Shanghai benefited from an architectural stagnation, the result of a
dramatic increase in innovative and artistic building." Stagnation is the exact
opposite of a dramatic increase in innovation.

3 Many subatomic nuclear particles are ------- and nearly -------: they are hard to track
as well as to detect.

(A) unstable . . explosive


(B) elusive . . imperceptible
(C) minute . . immobile

(D) charged . . reactive

(E) tenuous . . indivisible

ANSWERS AND EXPLANATIONS


Explanation for Correct Answer B :
Choice (B) is correct. "Elusive" means hard to find. "Imperceptible" means difficult

file://E:\\c3.htm 2006-11-12
The Official SAT Online Course 3/22

to perceive or detect. If one were to insert these terms into the text, the sentence
would read "Many subatomic nuclear particles are elusive and nearly imperceptible:
they are hard to track as well as to detect." The statement following the colon
explains or expands on the statement preceding it. The first missing term indicates
that the particles are hard to track: this perfectly defines "elusive." The second
missing term indicates that they are hard to detect: this perfectly defines
"imperceptible."

Explanation for Incorrect Answer A :


Choice (A) is incorrect. "Unstable" means likely to change suddenly. "Explosive"
means likely to blow up. If one were to insert these terms into the text, the
sentence would read "Many subatomic nuclear particles are unstable and nearly
explosive: they are hard to track as well as to detect." The statement following the
colon explains or expands on the statement preceding it. The first missing term
indicates that the particles are hard to track: this does not define "unstable."

Explanation for Incorrect Answer C :


Choice (C) is incorrect. "Minute" means extremely small. "Immobile" means not
moving. If one were to insert these terms into the text, the sentence would read
"Many subatomic nuclear particles are minute and nearly immobile: they are hard to
track as well as to detect." The statement following the colon explains or expands
on the statement preceding it. The first missing term indicates that the particles are


hard to track: this does not precisely define minute. The second missing term
indicates that they are hard to detect: this does not define immobile. Anything that
does not move should be easy, not difficult, to detect.


d
Explanation for Incorrect Answer D :
Choice (D) is incorrect. "Charged" means full of energy. "Reactive" means readily

e
responsive to a stimulus. If one were to insert these terms into the text, the
sentence would read "Many subatomic nuclear particles are charged and nearly
er
reactive: they are hard to track as well as to detect." The statement following the


colon explains or expands on the statement preceding it. The first missing term
indicates that the particles are hard to track: this does not define charged. The
second missing term indicates that they are hard to detect: this does not define
st


reactive.

Explanation for Incorrect Answer E :


i


eg

Choice (E) is incorrect. "Tenuous" means flimsy. "Indivisible" means not capable of
being broken down into smaller parts. If one were to insert these terms into the
text, the sentence would read "Many subatomic nuclear particles are tenuous and


nearly indivisible: they are hard to track as well as to detect." The statement
following the colon explains or expands on the statement preceding it. The first
nR

missing term indicates that the particles are hard to track: this does not define
"tenuous." The second missing term indicates that they are hard to detect: this


does not define "indivisible."

4 The crafty child tricked his innocent brother, a particularly ------- and trusting boy,
into committing a mischievous prank.

(A) guileless
(B) intrusive
(C) astute

(D) opportunistic

(E) circumspect

ANSWERS AND EXPLANATIONS


Explanation for Correct Answer A :
Choice (A) is correct. "Guileless" means innocent and naive. If one were to insert
this term into the text, the sentence would read "The crafty child tricked his
innocent brother, a particularly guileless and trusting boy, into committing a
mischievous prank." The missing term characterizes a brother who is innocent,
trusting, and able to be tricked. "Guileless" suggests such a person.

Explanation for Incorrect Answer B :

file://E:\\c3.htm 2006-11-12
The Official SAT Online Course 4/22

Choice (B) is incorrect. "Intrusive" means invasive or nosy. If one were to insert this
term into the text, the sentence would read "The crafty child tricked his innocent
brother, a particularly intrusive and trusting boy, into committing a mischievous
prank." The missing term characterizes a brother who is innocent, trusting, and able
to be tricked. A brother who is intrusive may or may not be such a person.

Explanation for Incorrect Answer C :


Choice (C) is incorrect. "Astute" means knowledgeable and wise. If one were to
insert this term into the text, the sentence would read "The crafty child tricked his
innocent brother, a particularly astute and trusting boy, into committing a
mischievous prank." The missing term characterizes a brother who is innocent,
trusting, and able to be tricked. An astute person would not likely be tricked.

Explanation for Incorrect Answer D :


Choice (D) is incorrect. "Opportunistic" means taking advantage of opportunities
when they arise. If one were to insert this term into the text, the sentence would
read "The crafty child tricked his innocent brother, a particularly opportunistic and
trusting boy, into committing a mischievous prank." The missing term characterizes
a brother who is innocent, trusting, and able to be tricked. Someone who is
opportunistic may or may not be such a person.

Explanation for Incorrect Answer E :


Choice (E) is incorrect. "Circumspect" means possessing caution or prudence. If one
were to insert this term into the text, the sentence would read "The crafty child
tricked his innocent brother, a particularly circumspect and trusting boy, into


committing a mischievous prank." The missing term characterizes a brother who is
innocent, trusting, and able to be tricked. It would be hard to trick a circumspect
person.


d
5
re
Ellen Ochoas ------- with the apparatus in the space shuttle Discovery was apparent
when she adroitly manipulated the shuttles robot arm.


te
(A) compromise
(B) humility


is
(C) machinations

(D) synergy


eg

(E) deftness


nR

ANSWERS AND EXPLANATIONS


Explanation for Correct Answer E :
Choice (E) is correct. "Deftness" means skill or dexterity. If one were to insert this
term into the text, the sentence would read "Ellen Ochoa's deftness with the

apparatus in the space shuttle Discovery was apparent when she adroitly
manipulated the shuttle's robot arm." The missing term must have a meaning close
to that of "adroit," which means skillful or dexterous. Deft and adroit are
synonymous.

Explanation for Incorrect Answer A :


Choice (A) is incorrect. "Compromise" is a settlement of mutual concessions. If one
were to insert this term into the text, the sentence would read "Ellen Ochoa's
compromise with the apparatus in the space shuttle Discovery was apparent when
she adroitly manipulated the shuttle's robot arm." The missing term must have a
meaning close to that of "adroit," which means skillful or dexterous. A compromise
in this context has nothing to do with an adroit action.

Explanation for Incorrect Answer B :


Choice (B) is incorrect. "Humility" is a modest attitude. If one were to insert this
term into the text, the sentence would read "Ellen Ochoa's humility with the
apparatus in the space shuttle Discovery was apparent when she adroitly
manipulated the shuttle's robot arm." The missing term must have a meaning close
to that of "adroit," which means skillful or dexterous. Humility has nothing to do
with performing an adroit action.

Explanation for Incorrect Answer C :


Choice (C) is incorrect. "Machinations" are crafty schemes. If one were to insert this
term into the text, the sentence would read "Ellen Ochoa's machinations with the

file://E:\\c3.htm 2006-11-12
The Official SAT Online Course 5/22

apparatus in the space shuttle Discovery was apparent when she adroitly
manipulated the shuttle's robot arm." The missing term must have a meaning close
to that of "adroit," which means skillful or dexterous. Machinations have nothing to
do with an adroit action.

Explanation for Incorrect Answer D :


Choice (D) is incorrect. "Synergy" describes an action or operation that is greater
than the sum of its parts. If one were to insert this term into the text, the sentence
would read "Ellen Ochoa's synergy with the apparatus in the space shuttle Discovery
was apparent when she adroitly manipulated the shuttle's robot arm." The missing
term must have a meaning close to that of "adroit," which means skillful or
dexterous. Synergy has nothing to do with an adroit action.

Passage 1
When I entered journalism school in the
1920s, I
found out that perennial and fundamental laws
governing
the art of good writing had been discovered.
Experts
had stubbornly and rigorously analyzed

Line
readers modest

d
capacity to dedicate their attention to the
5
printed page
and had established once and for all, re

te
apparently with the
mathematical precision of astronomers, the
order of

is

readers natural preferences. They found that


eg

effective
prose was composed of a limited number of


very simple
nR

and common words grouped in short, crisp


10


sentences.
When designed rigorously, such prose could


penetrate the
U

opaque barrier of millions of readers


indifference, apathy,
inattention, and obtuseness.
Passage 2
Beginning writers are often taught that
effective prose
is crisp and concise and that most readers have
15
no patience
with densely complex sentences and obscure
vocabulary.
While clarity and succinctness are certainly
worthy goals,
I sometimes worry that our assumption that the
reading
public can comprehend only such writing might
be selling

file://E:\\c3.htm 2006-11-12
The Official SAT Online Course 6/22

them short. Assuming that readers are merely


20
able to digest
simple words, and that they have no interest in
puzzling
through more challenging prose, turns that
theory into a
self-fulfilling prophecy. Its our responsibility
as writers
to offer the public something beyond
workmanlike writing:
if we dont, readers will never appreciate
25
writing as an art
rather than as a mechanical craft.
6 Both passages address which of the following topics?
(A) Why people choose writing as a career
(B) The kind of writing that readers can understand


(C) How readers convey their preferences to writers

(D) The difference between beginning writers and experts


ed
(E) Why long sentences are easier to read than short ones

ANSWERS AND EXPLANATIONS


r
te

Explanation for Correct Answer B :
Choice (B) is correct. Each passage specifically addresses readers' abilities to
understand writing. Passage 1 addresses journalistic practices of writing short,


simple sentences designed to be understood by a large audience. Passage 2
is
challenges the idea that readers can only handle short, simple sentences.


eg


Explanation for Incorrect Answer A :
nR

Choice (A) is incorrect. Although both passages are about people who write, neither
addresses why people choose to write as a career.


Explanation for Incorrect Answer C :
Choice (C) is incorrect. Although admitting readers may prefer a certain style of

writing, neither passage says anything about how writers know what readers prefer
or that readers convey, or communicate, their preferences to writers.

Explanation for Incorrect Answer D :


Choice (D) is incorrect. Neither passage discusses different levels of writing
experience.

Explanation for Incorrect Answer E :


Choice (E) is incorrect. Both passages acknowledge the common belief that short
sentences are easier to understand than long ones.

7
Which statement in Passage 2 most directly contradicts the assertion in Passage 1,
lines 8-10 (They found . . . sentences) ?

(A) Beginning writers . . . concise (lines 14-15)


(B) most readers . . . vocabulary (lines 15-16)

(C) clarity and . . . goals (line 17)

(D) they have . . . prose (lines 21-22)

(E) Its our . . . writing (lines 23-24)

ANSWERS AND EXPLANATIONS

file://E:\\c3.htm 2006-11-12
The Official SAT Online Course 7/22

Explanation for Correct Answer E :


Choice (E) is correct. By stating that it is writers' responsibility to offer "challenging
prose," Passage 2 contradicts the assertion in Passage 1 that "effective prose" must
be very simple.

Explanation for Incorrect Answer A :


Choice (A) is incorrect. Both statements say the same thing, that effective prose is
simple and straightforward.

Explanation for Incorrect Answer B :


Choice (B) is incorrect. The statement in Passage 2 that most readers "have no
patience" with complex writing supports Passage 1's description of effective prose;
in fact, Passage 1 uses this same argument.

Explanation for Incorrect Answer C :


Choice (C) is incorrect. Both passages agree that, as Passage 2 concedes, "clarity
and succinctness," or conciseness, are "worthy goals."

Explanation for Incorrect Answer D :


Choice (D) is incorrect. The statement in Passage 2 that people have "no interest" in
"challenging prose" supports the argument for simple prose presented in Passage 1.



ed
8 The author of Passage 2 would most likely respond to the view of readers expressed
in the last sentence of Passage 1 (lines 11-13) with
(A) concern
r

(B) perplexity
te
(C) disdain


(D) humor
is

(E) appreciation


eg

ANSWERS AND EXPLANATIONS


Explanation for Correct Answer A :
nR

Choice (A) is correct. The author of Passage 2 argues that readers would like


challenging writing if only it were given to them. Consequently, the author of
Passage 2 would likely show concern, or unease, with Passage 1's scornful
description of readers. The author states that "I sometimes worry" that a negative


view of readers is being taught to young writers.
U

Explanation for Incorrect Answer B :


Choice (B) is incorrect. The author of Passage 2 would not likely be perplexed, or
confused, by the views expressed at the end of Passage 1, but would simply
disagree with them.

Explanation for Incorrect Answer C :


Choice (C) is incorrect. Disdain, or scorn, would be inconsistent with the reasonable
and concerned tone of Passage 2.

Explanation for Incorrect Answer D :


Choice (D) is incorrect. Passage 2 does not use humor in framing an argument that
challenges the commonly held negative view of readers; rather, the author is
concerned about the negative view of readers that is affecting the way young
writers compose their work.

Explanation for Incorrect Answer E :


Choice (E) is incorrect. The author of Passage 2 would likely disagree with the
opinions in Passage 1, rather than showing appreciation.

file://E:\\c3.htm 2006-11-12
The Official SAT Online Course 8/22

9 In comparison to Passage 2, the tone of Passage 1 is more


(A) earnest
(B) inspirational
(C) complacent

(D) defensive

(E) sarcastic

ANSWERS AND EXPLANATIONS


Explanation for Correct Answer E :
Choice (E) is correct. Passage 1 mocks the "experts" and also its own subject
(writing) by using long sentences and uncommon words to explain that good
writing should do the opposite. The language is especially sarcastic when the author
portrays the "experts'" treatment of writing as a science that has "perennial and
fundamental laws" that can be calculated "apparently with the mathematical
precision of astronomers." The tone of Passage 2 is concerned, not sarcastic.
Passage 2 speaks of the responsibility of writers to raise the reading level of their
audience.

Explanation for Incorrect Answer A :



Choice (A) is incorrect. The tone of Passage 1 is sarcastic and cutting when it


ed
describes the certainty of "experts" on the essence of good writing, which is the
opposite of being earnest, or sincere. Passage 2 clearly expresses earnest, or
heartfelt, feelings about the practice of writing and the capabilities of readers.


er
Explanation for Incorrect Answer B :
Choice (B) is incorrect. Passage 1 presents negative opinions about readers, which


could hardly be called inspirational. Passage 2, on the other hand, does attempt to
inspire young writers to believe that readers are better than they think and that
t
they can read more complex prose.


is

Explanation for Incorrect Answer C :


Choice (C) is incorrect. The tone of ridicule in Passage 1 is far from complacent, or


eg

easy-going. Neither of the passages are complacent, or untroubled; Passage 1 is


sarcastic, or cutting, and Passage 2 is more earnest and inspirational.


Explanation for Incorrect Answer D :
nR

Choice (D) is incorrect. Neither passage is defensive in tone; Passage 1 is mocking


in tone, while Passage 2 makes a reasoned argument.

The following passage is from a 1994 collection of essays about animals, written by a
poet, philosopher, and animal trainer.

The question that comes first to my mind is


this: What
would it mean to say that an animal has the
right to the
pursuit of happiness? How would that come
about, and in
Line relationship to whom?
In speaking of animal happiness, we
5
often tend to
mean something like creature comforts. The
emblems of
this are the golden retriever rolling in the grass,
the horse
with his nose deep in the oats, kitty by the fire.
Creature

file://E:\\c3.htm 2006-11-12
The Official SAT Online Course 9/22

comforts are important to animals: Grub first,


then ethics
is a motto that would describe many a wise
10
Labrador
retriever, and I have a bull terrier named Annie
whose
continual quest for the perfect pillow inspires
her to
awesome feats. But there is something more to
animals,
something more to my Annie, a capacity for
satisfactions
that come from work in the full sense
15
something
approximately like what leads some people to
insist that
they need a career (though my own
temperament is such

that I think of a good woodcarver or a dancer
or a poet

d
sooner than I think of a business executive

20
when I
re
contemplate the kind of happiness enjoyed by


te
an
*
accomplished dressage horse). This happiness,


is
like the
artists, must come from something within the


eg

animal,
something trainers call talent, and so cannot be
imposed

nR

on the animal. But at the same time it does not

25
arise in a
vacuum; if it had not been a fairly ordinary


thing in one
U

part of the world at one point to teach young


children to
play the harpsichord, it is doubtful that
Mozarts music
would exist. There are animal versions, if not
equivalents,
of Mozart, and they cannot make their
spontaneous
passions into sustained happiness without
30
education, any
more than Mozart could have.
Aristotle identified happiness with ethics and
with work,
unlike Thomas Jefferson, who defined
happiness as
Indolence of Body; Tranquility of Mind, and
thus what I

file://E:\\c3.htm 2006-11-12
The Official SAT Online Course 10/22

call creature comforts. Aristotle also excluded


35
as unethical
anything that animals and artists do, for
reasons that look
wholly benighted to me. Nonetheless, his
central insights
are more helpful than anything else I know in
beginning
to understand why some horses and dogs can
only be
described as competent, good at what they do,
40
and there-
fore happy. Not happy because leading lives of
pleasure,
but rather happy because leading lives in which
the sensa-
tion of getting it right, the click, as of the
pleasure that
comes from solving a puzzle or surmounting
something,

d
45 is a governing principle.

e
er
* Dressage is a complex series of movements signaled to
a horse by its rider.

t

is
10 The author presents examples in lines 7-8 in order to

(A) illustrate the variety of activities in which animals engage


eg

(B) suggest that appearances of happiness are deceptive


(C) evoke images of contentment
nR

(D) support an apparently implausible argument


(E) arouse nostalgic longings


ANSWERS AND EXPLANATIONS
U

Explanation for Correct Answer C :


Choice (C) is correct. The examples given in the lines "the golden retriever rolling
the grass, the horse with his nose deep in the oats, and kitty by the fire" are all
presented as examples of "creature comforts," images that evoke, or bring to mind,
notions of happiness or contentment.

Explanation for Incorrect Answer A :


Choice (A) is incorrect. Although the examples in lines 7-8 do show animals
engaged in a variety of activities, this is not their primary purpose. Their primary
purpose is to offer examples of animal activities that humans believe indicate that
animals are content.

Explanation for Incorrect Answer B :


Choice (B) is incorrect. Although the passage argues that animal happiness differs
from human happiness, it does not suggest that the examples presented in lines 7-
8 are deceptive, or misleading.

Explanation for Incorrect Answer D :


Choice (D) is incorrect. The passage does not suggest that anyone would be likely
to find its arguments "implausible," or hard to believe.

Explanation for Incorrect Answer E :

file://E:\\c3.htm 2006-11-12
The Official SAT Online Course 11/22

Choice (E) is incorrect. The author does not mention nostalgic longings, even
though the activities described could certainly evoke those feelings in readers.

11 The motto in line 9 indicates that animals


(A) are much more intelligent than many people believe
(B) have been forced to develop keen survival skills
(C) desire consistency in their daily lives

(D) enjoy close relationships with human beings

(E) are concerned primarily with immediate physical gratification

ANSWERS AND EXPLANATIONS


Explanation for Correct Answer E :
Choice (E) is correct. In the context of the passage, the motto, "Grub first, then
ethics," means that it is important to satisfy physical needs before one addresses
ethical or moral dilemmas. For animals, this means that they tend to be concerned
about physical needs first.



ed
Explanation for Incorrect Answer A :
Choice (A) is incorrect. Although the passage says that the motto, "Grub first, then
ethics," "would describe many a wise Labrador retriever" (lines 10-11), there is no


suggestion that such an animal would be any wiser than would be expected. The
motto simply suggests that animals are concerned with their physical needs before
all others.
er
Explanation for Incorrect Answer B :

st
Choice (B) is incorrect. Although ensuring that one has enough food could be called


a "survival skill," the use of the motto, "Grub first, then ethics," in this passage
does not suggest that these skills have been forced on animals. Furthermore, the
i

passage never discusses survival skills.


eg

Explanation for Incorrect Answer C :


Choice (C) is incorrect. Although animals may desire a consistent routine, the motto


is about food, not consistency.
nR

Explanation for Incorrect Answer D :


Choice (D) is incorrect. The passage does not address the topic of animals'
relationships with people.

12
Which of the following statements is most consistent with the authors discussion of
temperament in lines 17-21?

(A) The author believes a poet can be successful in business.

(B) The author considers artistic pursuits to be the most personally fulfilling of
all endeavors.
(C) The author suspects that a busy life can have its own rewards.

(D) The author believes that few people are ever satisfied with the jobs they
have chosen.
(E) The author considers subjectivity and self-knowledge to be critical to
human gratification.

ANSWERS AND EXPLANATIONS


Explanation for Correct Answer B :
Choice (B) is correct. "Temperament" means disposition or character. The author
feels that the careers "of a good woodcarver or a dancer or a poet" are more
consistent with his own temperament than a career in business. This suggests that
the author believes artistic careers to be the most "personally fulfilling of all
endeavors."

file://E:\\c3.htm 2006-11-12
The Official SAT Online Course 12/22

Explanation for Incorrect Answer A :


Choice (A) is incorrect. Although the author does mention a poet as one of the
examples in lines 17-21, the issue of whether or not a poet could have a successful
business career is not addressed.

Explanation for Incorrect Answer C :


Choice (C) is incorrect. In lines 17-21, the author does not discuss the issue of
whether or not "a busy life can have its own rewards." Instead, the author seems
concerned with the satisfaction that can be gained through work that is personally
fulfilling (such as the work done by a "good woodcarver or a dancer or a poet").

Explanation for Incorrect Answer D :


Choice (D) is incorrect. Although the author may hold the view that "few people are
ever satisfied with the jobs they have chosen," this view is not addressed in the
passage. On the contrary, the passage suggests that work can be an extremely
rewarding pursuit.

Explanation for Incorrect Answer E :


Choice (E) is incorrect. The author probably believes this to be true, but it is not
relevant to the author's discussion of temperament in lines 17-21.


13 The authors discussion of Mozart in lines 25-28 primarily emphasizes the

(A) role of social circumstances in the emergence of a musical genius


d
(B) fact that young children are sometimes pushed to excel
(C) observation that genius was more common in the past than it is today

(E)
early talent
re
(D) belief that the harpsichord was the ideal musical instrument for Mozarts

pleasure that artists derive from achievement


te


ANSWERS AND EXPLANATIONS
is

Explanation for Correct Answer A :


eg

Choice (A) is correct. The statement points out that a social circumstance, namely
musical education, enabled Mozart's genius to find expression. The passage also
suggests that without this musical education Mozart's natural talent may not have


found expression.
nR


Explanation for Incorrect Answer B :

Choice (B) is incorrect. Although teaching young children to play the harpsichord
might be described as pushing them to excel, the primary focus of the discussion in
lines 25-28 is on the role of musical education in the development of musical
genius.

Explanation for Incorrect Answer C :


Choice (C) is incorrect. The author makes no comment about the relative incidence,
or numbers of occurrences, of genius in the past or in the present.

Explanation for Incorrect Answer D :


Choice (D) is incorrect. Even though the harpsichord may have been the ideal
musical instrument for Mozarts early talent, the emphasis in the passage is not
on which instrument he played, but on his having been given a musical education.

Explanation for Incorrect Answer E :


Choice (E) is incorrect. Although the author discusses the pleasure that artists
derive from achievement earlier in the passage, in this section the emphasis is on
the role of musical education in the development of Mozart's talent.

14 In line 30, passions most nearly means

(A) love affairs


(B) violent outbursts

file://E:\\c3.htm 2006-11-12
The Official SAT Online Course 13/22

(C) enthusiasms

(D) prejudices

(E) sufferings

ANSWERS AND EXPLANATIONS


Explanation for Correct Answer C :
Choice (C) is correct. In the passage, the author describes "animal versions, if not
equivalents, of Mozart," arguing that "they cannot make their spontaneous passions
into sustained happiness without education, any more than Mozart could have." The
author uses the word "passions" in the sense of impulses, talents, or enthusiasms
that require training and education if they are to lead to "happiness."

Explanation for Incorrect Answer A :


Choice (A) is incorrect. "Passions" does not refer to love or ardent affections in this
context, but to strong impulses, or enthusiasms, such as Mozart's feeling for music.

Explanation for Incorrect Answer B :


Choice (B) is incorrect. In this context, "passions" refer to a talent, or enthusiasm,
that could be cultivated into happiness through education. Consequently, it would
be strange to say that animals "cannot make" their violent outbursts "into sustained
happiness without education."


d
Explanation for Incorrect Answer D :

re
Choice (D) is incorrect. "Passions" does not refer to prejudices or strong opinions in
this context; rather, it refers to positive impulses, such as talents, that have the
possibility of being shaped into masterpieces like Mozart's music.


te
Explanation for Incorrect Answer E :
Choice (E) is incorrect. The context does not mention suffering nor does the


passage have anything to do with suffering.
is


eg

15
Which situation most accurately illustrates the authors definition of a happy animal?

(A)


A bird finding its one lifetime mate
nR

(B) A dog herding sheep into a pen


(C) A horse being carefully groomed for a show

(D) A monkey escaping from a city zoo

(E) A cat caring devotedly for her kittens

ANSWERS AND EXPLANATIONS


Explanation for Correct Answer B :
Choice (B) is correct. The passage states, "there is something more" to animals'
happiness than mere comfort, namely, the "satisfactions that come from work." A
dog herding sheep into a pen is performing the kind of work that the author
describes as satisfying to animals.

Explanation for Incorrect Answer A :


Choice (A) is incorrect. Although finding a mate could be thought of as contributing
to happiness, the kind of happiness the author is discussing involves work.

Explanation for Incorrect Answer C :


Choice (C) is incorrect. "A horse being carefully groomed for a show" is a creature
receiving attention, not one that is working or performing a challenging task.

Explanation for Incorrect Answer D :


Choice (D) is incorrect. Although escaping from a zoo might be pleasurable as well
as challenging for a monkey, it is not the kind of "work in the full sense" (line 15)
that would give the animal pleasure for a job well done.

file://E:\\c3.htm 2006-11-12
The Official SAT Online Course 14/22

Explanation for Incorrect Answer E :


Choice (E) is incorrect. Caring for kittens is an action of devotion, but it is not work
or a career in the sense discussed in the passage.

This passage was written in 1996 after the discovery of a meteorite that appeared to
contain fossil evidence of microscopic life on Mars.

The rock that sprang to Martian life late


last summer
did not shock me by offering up apparent
fossils of an extinct
alien form of life. I had long believed that the
universe teems
with life elsewhere, and that our failure to find
Line
it simply
results from a lack of exploration. What did
5
amaze me about
the potato-size rock that fell from Mars was
that it had trav-
eled millions of miles across space to land here,

d
blasted

re
from world to world by a planetary collision of
the sort that


purportedly killed off our dinosaurs, and had
te
lain waiting


for millennia upon an Antarctic ice field, until an
is
10
observant
young woman traveling in an expedition party


eg

picked it up,
because she figured that it had come from
another world.

nR

How could she know such a thing?


The composition of ALH 84001, as the much
scruti-

nized rock is designated, closely matches the


15
makeup
of Martian matter that was analyzed on site in
1976 by
miniature chemistry laboratories aboard two
Viking
Mars landers. As a result of this positive
identification,
no astronomer seriously doubts the meteorite
s Martian
origin. Researchers think they have pinpointed
20
its former
resting place to just two possible sitesa region
called
Sinus Sabaeus, fourteen degrees south of the
Martian
equator, or a crater east of the Hesperia
Planitia region.

file://E:\\c3.htm 2006-11-12
The Official SAT Online Course 15/22

The bold precision of this assessment is for me


the most
stunning surprise dealt by the rock from Mars
25
even more
mind-boggling than the suggestive traces of
something that
might once have lived and died in its
microscopic fissures.
I cannot resist comparing this new intimacy
with our solar
system to the shoebox diorama of the planets I
designed for
my grade-school science fair. I used marbles,
30
jack balls,
and Ping-Pong balls, all hanging on strings and
painted
different colors, all inside a box representing
our solar

system. This crude assortment of materials
allowed a

d
reasonable representation of what was known

35
40 years
re
ago about the nine planets: Mars was red and


had two
te
moons; Jupiter dwarfed the other planets (I


should have
is

used a basketball but it wouldnt fit in the


box); Saturn
eg

had rings. If my school-age daughter were to


attempt such


a construction today, shed need handfuls of
nR

jelly beans
40
and gum balls to model the newly discovered
satellites of

the giant planets. Shed want rings around


Jupiter, Uranus,
Neptune, too, not to mention a moon for Pluto.
Similarly, our solar system, once considered
unique,
now stands as merely the first known example
of a
planetary system in our galaxy. Since October
45
of 1995,
astronomers at ground-based observatories in
Europe
and the United States have announced that
theyve found
evidence of at least seven alien planets orbiting
other stars.
As yet, not one of these large planetssome of
which
are many times the mass of Jupiterhas

file://E:\\c3.htm 2006-11-12
The Official SAT Online Course 16/22

50 actually been
seen through a telescope; we know about them
indirectly
through the gravitational effects they exert on
their parent
stars. Yet, even though we have no picture of
what they
look like, enough information has been deduced
about
their atmospheric conditions to grant the
55
nickname
Goldilocks to a planet attending the star 70
Virginis,
an appellation suggesting that the cloud-top
temperature
is just right, as the storybook Goldilocks
would say, for
the presence of liquid water. Liquid water, not
known to
exist anywhere in our solar system now except

d
60
on Earth,

re
is thought crucial to biological life; thus, only a
short


te
leap of faith is needed to carry hopeful
scientists from


the presence of water to the existence of
is

extraterrestrial
life. To raise the specter of the Mars rock once


eg

again,
the primitive life-forms that pressed their
65
memory inside

nR

it likewise suggest an era when dry-as-dust


Mars was a

wet world, where rivers flowed.

16 In lines 5-12, the author suggests that the expeditionists discovery of the meteorite
was surprising primarily because it

(A) defied scientists doubts that such an object could reach Earth
(B) occurred after her party had given up any hope of success
(C) resulted from a seemingly unlikely sequence of events

(D) provided evidence to contradict a long-standing scientific theory

(E) led to an unprecedented degree of scrutiny

ANSWERS AND EXPLANATIONS


Explanation for Correct Answer C :
Choice (C) is correct. The author writes, "[w]hat did amaze me about the potato-
size rock that fell from Mars was that it had traveled millions of miles to land
here" (lines 5-7). Further, the author describes the rock's journey as one in which it
was "blasted from world to world," and had "lain waiting for milennia upon an
Antarctic ice field" (lines 7-10). The author is clearly amazed at the "seemingly
unlikely sequence of events" that led to the rock's discovery.

file://E:\\c3.htm 2006-11-12
The Official SAT Online Course 17/22

Explanation for Incorrect Answer A :


Choice (A) is incorrect. The passage does not mention whether the scientists had
"doubts that such an object could reach earth." The author expresses her awe or
wonder over the process by which the meteorite came to earth and was found, but
this has little to do with the beliefs of scientists.

Explanation for Incorrect Answer B :


Choice (B) is incorrect. The passage does not mention the expedition party's
expectations or attitudes prior to finding the rock.

Explanation for Incorrect Answer D :


Choice (D) is incorrect. Far from contradicting a "long-standing scientific theory,"
the "apparent fossils of an extinct alien form of life" (lines 2-3) in the rock tend to
confirm the author's belief that life may exist on other planets.

Explanation for Incorrect Answer E :


Choice (E) is incorrect. It is not surprising that such a rare find would be studied so
closely.

17 In line 15, designated most nearly means

(A) drawn


(B) called
(C) stipulated


(D) selected

d
(E) allocated

ANSWERS re
AND EXPLANATIONS

te
Explanation for Correct Answer B :
Choice (B) is correct. "Designated" means named. The passage makes it clear that


is
ALH 84001 is the name given to the rock by scientists.


eg

Explanation for Incorrect Answer A :


Choice (A) is incorrect. The term ALH 84001 is not a picture, drawing, or means of
nR

depicting an object. ALH 84001 is a name.


Explanation for Incorrect Answer C :
Choice (C) is incorrect. "Stipulated" means required or demanded. There is no
mention that the naming of the rock ALH 84001 was stipulated, required, or

demanded.

Explanation for Incorrect Answer D :


Choice (D) is incorrect. "Selected" means chosen, and its use in this context would
imply that there were other rocks to choose from. The author makes the unique
nature of this rock clear.

Explanation for Incorrect Answer E :


Choice (E) is incorrect. "Allocated" means gave or assigned. Line 15 does not refer
to any allocation or assignment; rather, it refers to the name given to the rock.

18 The author considers the researchers conclusion bold (line 24) primarily
because it

(A) daringly offers two conflicting answers to a single question


(B) firmly supports a young geologists tentative theory
(C) confidently uses a small piece of evidence to build an exact hypothesis

(D) courageously defies a conventional interpretation of Antarctic evidence

(E) defiantly espouses an unpopular theory about comets in our solar system

ANSWERS AND EXPLANATIONS

file://E:\\c3.htm 2006-11-12
The Official SAT Online Course 18/22

Explanation for Correct Answer C :


Choice (C) is correct. The author views the scientists' ability to use minute details of
chemical composition to match the rock found in Antarctica to rocks from particular
regions studied on Mars as "bold," or confident.

Explanation for Incorrect Answer A :


Choice (A) is incorrect. The passage does not offer conflicting answers to the
question of the rock's origin; rather, the author explains that researchers "think
they have pinpointed its [the rock's] former resting place to just two possible sites."

Explanation for Incorrect Answer B :


Choice (B) is incorrect. The author does not mention a "young geologist" in the
passage.

Explanation for Incorrect Answer D :


Choice (D) is incorrect. Researchers were concerned with the rock's place of origin
(lines 20-23), not its Antarctic discovery site.

Explanation for Incorrect Answer E :


Choice (E) is incorrect. The author does not mention comets or theories about them.



d
19 The author uses the phrase this new intimacy (line 28) to refer to the

re
hands-on quality of the learning experience repre-sented by the shoebox


(A)
diorama
(B) understanding that nonspecialists now have about meteorological
phenomena
e

(C) general acceptance of the theory that biological life once existed on Mars
st
(D) increased knowledge that scientists have about our solar system

(E)


way that events on one planet affect those on another
i


eg

ANSWERS AND EXPLANATIONS


Explanation for Correct Answer D :
nR

Choice (D) is correct. In line 28, "intimacy" means close acquaintance. The author
is referring to how much scientists have learned about the solar system in recent


decades as compared to what was known 40 years ago (lines 33-35).

Explanation for Incorrect Answer A :


Choice (A) is incorrect. The context of the phrase "this new intimacy" refers to how
much scientists have learned about our solar system, not their "learning
experience."

Explanation for Incorrect Answer B :


Choice (B) is incorrect. Although the author does use herself and her daughter
(both nonspecialists) as examples in this passage, her use of the phrase "this new
intimacy" refers to what astronomers (specialists) know about our solar system.

Explanation for Incorrect Answer C :


Choice (C) is incorrect. In the context of the passage, the phrase "this new
intimacy" specifically refers to the solar system and does not allude to the
possibility of life on Mars.

Explanation for Incorrect Answer E :


Choice (E) is incorrect. The author uses the phrase "this new intimacy" to refer to
increasing knowledge about our solar system. The author does not discuss whether
events happening on one planet influence events on another.

20 In line 33, crude most nearly means

file://E:\\c3.htm 2006-11-12
The Official SAT Online Course 19/22

(A) natural and unaltered


(B) rough and inexpert
(C) obvious

(D) vulgar

(E) nonspecific

ANSWERS AND EXPLANATIONS


Explanation for Correct Answer B :
Choice (B) is correct. "Crude" means rough or inexpert. The author's diorama and
its materials"marbles, jack balls, and Ping-Pong balls" (lines 30-31)could well
be described as "crude," or rough and inexpert, especially since the author used
everyday materials in an attempt to model the solar system.

Explanation for Incorrect Answer A :


Choice (A) is incorrect. The materials mentioned ("marbles, jack balls, and Ping-
Pong balls") were manufactured items, not natural ones.

Explanation for Incorrect Answer C :


Choice (C) is incorrect. "Obvious" means easily perceived or understood. The


author's description emphasizes the fact that the materials used in the diorama

ed
were rough and unsophisticated, not the idea that they were easily perceived or
understood.

Explanation for Incorrect Answer D :


er
Choice (D) is incorrect. "Vulgar" means indecent or in poor taste. The passage does


not suggest that the materials used in the diorama were either indecent or in poor
taste.
t

is
Explanation for Incorrect Answer E :
Choice (E) is incorrect. On the contrary, the author's account suggests that the
materials used in the diorama were chosen quite specifically to represent objects in


eg

the solar system.


nR

21
In line 42, the author refers to Plutos moon most likely in order to


(A) illustrate a feature of our solar system discovered since the authors
childhood

(B) cite an object too small in scale to have been included in the authors
diorama
(C) draw a parallel between it and our own moon

(D) contrast the scientific curiosity of todays children with that of children
years ago
(E) emphasize the need for a greater commitment to space exploration

ANSWERS AND EXPLANATIONS


Explanation for Correct Answer A :
Choice (A) is correct. The author refers to Pluto's moon in order to compare our
current knowledge of our solar system with that of the past. The author's daughter
would need to include Pluto's moon if she were to build a diorama, because the
moon's existence is now current knowledge.

Explanation for Incorrect Answer B :


Choice (B) is incorrect. Pluto's moon was absent from the author's diorama not
because it was too small, but because it had not yet been discovered.

Explanation for Incorrect Answer C :


Choice (C) is incorrect. The author is comparing past and present knowledge about
objects in space, not the objects themselves.

file://E:\\c3.htm 2006-11-12
The Official SAT Online Course 20/22

Explanation for Incorrect Answer D :


Choice (D) is incorrect. The author is not comparing the children's curiosity.
Instead, the author is comparing the knowledge of today's children with that of
children years ago.

Explanation for Incorrect Answer E :


Choice (E) is incorrect. Although the author might well be in favor of continued
space exploration, the idea is not mentioned in the passage.

22 The reasoning process presented in lines 49-53 (As . . . stars) is best described
as
(A) inference based on an untested theory
(B) extrapolation from similar situations
(C) analysis of a single case by multiple observers

(D) hypothesis confirmed by direct observation

(E) comparison of theory with physical evidence

ANSWERS AND EXPLANATIONS


Explanation for Correct Answer B :

Choice (B) is correct. "Extrapolate" means infer. The passage states, "we know


d
about them [the planets] indirectly through the gravitational effects they exert on
their parent stars" (51-53). In other words, the passage describes planets that are

re
known only by their gravitational effects. This knowledge would have to be arrived


at by extrapolation, or inference, from observations of known planets and the
gravitational effects of their planet stars.
te


is
Explanation for Incorrect Answer A :
Choice (A) is incorrect. The author mentions that the basis for knowledge about
these planets is the "gravitational effects" these planets "exert on their parent
eg


stars" (lines 52-53). Thus the basis of knowledge about these planets is the theory
of gravity. This is not an untested theory.


Explanation for Incorrect Answer C :
nR

Choice (C) is incorrect. The author does not mention the number of observers
involved. Further, it seems unlikely that the number of observers involved would


alter any description of the reasoning process.

Explanation for Incorrect Answer D :

Choice (D) is incorrect. The passage states that none of the planets have "actually
been seen through a telescope" (lines 50-51). Thus none of the planets mentioned
in lines 49-53 have been directly observed.

Explanation for Incorrect Answer E :


Choice (E) is incorrect. Although the author mentions the discovery of a rock from
Mars (lines 5-6), no physical evidence has been collected to support the existence
of the planets described in lines 49-53.

23

In lines 53-59 the author refers to the Goldilocks fairy tale (Yet . . . water) in
order to make which point about a particular planet?

(A) The planets environment may be conducive to a result some scientists


are eager to find.
(B) The planets atmosphere was once thought to be too cold to support
biological life.
(C) The simple methods astronomers used to discover the features of this
planet resemble the explorations of curious children.
(D) Scientists wishful speculations about the existence of this planet deserve
little more credence than a fairy tale.
(E) Only after much trial and error did astronomers determine the precise
location of this planet.

file://E:\\c3.htm 2006-11-12
The Official SAT Online Course 21/22

ANSWERS AND EXPLANATIONS


Explanation for Correct Answer A :
Choice (A) is correct. The planet was nicknamed "Goldilocks" because its
temperature was "just right," or perfect, for the presence of liquid water. The
possibility of liquid water, "not known to exist anywhere in our solar system now
except on Earth, is thought crucial to biological life." Scientists are eager to find a
planet with liquid water because it could support life.

Explanation for Incorrect Answer B :


Choice (B) is incorrect. The passage does not say what scientists previously thought
about the atmospheric temperature of the planet.

Explanation for Incorrect Answer C :


Choice (C) is incorrect. In lines 53-59, the author uses the fairy tale to make a
point about what Goldilocks found (things that were "just right"), not to make a
point about the methods that Goldilocks used.

Explanation for Incorrect Answer D :


Choice (D) is incorrect. Though the planet has not been observed visually, the


author does not express any doubt that it exists. Further, the author describes
scientists' means of discovering this and other planets: "we know about them
indirectly through the gravitational effects they exert on their parent stars" (lines


51-53).

d
Explanation for Incorrect Answer E :

re
Choice (E) is incorrect. Although Goldilocks did find things that were "just right"
through trial and error, the author does not mention the use of trial and error in her
discussion of this planet's location. Instead, the author simply states that the planet


te
was found by measuring the gravitational effects exerted on its parent star (lines
51-53).


is

24


eg

Which of the following, if true, would the hopeful scientists (line 62) most likely
interpret as evidence of the potential for life on Mars?

Mars was affected by the same planetary collision that caused the


(A)
extinction of dinosaurs.
nR

(B) Mars had a very mild atmospheric temperature millions of years ago.


(C) Mars had a wet environment at one time in the past.

(D) The rock that fell from Mars resembled rocks found on the Antarctic ice


field.
U

(E) The rock that fell from Mars had very few microscopic fissures.

ANSWERS AND EXPLANATIONS


Explanation for Correct Answer C :
Choice (C) is correct. Since water "is thought crucial to biological life" (line 61),
evidence that Mars was once wet would be seen as suggesting that life could have
existed there.

Explanation for Incorrect Answer A :


Choice (A) is incorrect. Although the passage does mention the possibility that Mars
may have experienced a "planetary collision of the sort that purportedly killed off
our dinosaurs" (lines 8-9), the passage does not suggest that such a planetary
collision would have any bearing on Mars's capacity to support life.

Explanation for Incorrect Answer B :


Choice (B) is incorrect. A mild atmospheric temperature need not necessarily
include water, which is essential to life. Furthermore, there is no mention of the
effects of a mild atmospheric temperature on the development of life on Mars.

Explanation for Incorrect Answer D :


Choice (D) is incorrect. The passage does not say whether rocks in Antarctica have

file://E:\\c3.htm 2006-11-12
The Official SAT Online Course 22/22

any properties that would suggest the potential for life, or that rocks in the Antarctic
ice field bear any resemblance, or likeness, to rocks from Mars.

Explanation for Incorrect Answer E :


Choice (E) is incorrect. Fossils in a rock, which the author describes as "suggestive
traces of something that might once have lived and died" (lines 26-27), are
mentioned as evidence suggestive of life. Fissures, or cracks, are not.

Back to Score Report

Copyright 2006 The College Board. All rights reserved. Privacy Policy Terms of Use Contact Us



r

ed
te

is


eg


nR

file://E:\\c3.htm 2006-11-12
The Official SAT Online Course 1/12

Help | Profile | My Organizer | My Bookmarks | Logout

Answers and Explanations

Test Sections Back to Score Report

Section 1 View Answers and Explanations


Section 2 Online - Practice Test #3

Section 3
1 Each month, a telephone service charges a base rate of and an additional
Section 4
Section 5
per call for the first calls and for every call after that. How much
does the telephone service charge for a month in which calls are made?
Section 7
Section 8 (A)
Section 9 (B)
Section 10
(C)

(D)


(E)


d
ANSWERS AND EXPLANATIONS

re
Explanation for Correct Answer C :
Choice (C) is correct. The bill for calls will be


. This simplifies to


te
.


is


eg

Explanation for Incorrect Answer A :


Choice (A) is not correct. This amount, would only account for the base
cost of plus in additional charges for


individual calls. The first calls, which would cost would
nR

cost more than .


Explanation for Incorrect Answer B :
Choice (B) is not correct. This amount, would only account for the base

cost of plus in additional charges for


individual calls. The first calls, which would cost would
cost more than .

Explanation for Incorrect Answer D :


Choice (D) is not correct. This amount, would be enough for the base cost
of plus in additional charges for calls. This

would be enough for an average per-call charge of which is

larger than the given per-call charges of either or .

Explanation for Incorrect Answer E :


Choice (E) is not correct. This amount, would be enough for the base cost
of plus in additional charges for calls. This

would be enough for an average per-call charge of which is

larger than the given per-call charges of either or .

file://E:\\c4.htm 2006-11-12
The Official SAT Online Course 2/12

According to the chart above, Company experienced its largest increase in


monthly profits between which two consecutive months?

(A) January and February


(B) February and March
(C) March and April

(D) April and May

(E) May and June



ANSWERS AND EXPLANATIONS

d
Explanation for Correct Answer C :

e
Choice (C) is correct. Between January and February, profits increased by about
Between February and March, profits decreased by about
er
Between March and April, profits increased by about Between April and
May, profits decreased by about
by
Between May and June, profits increased
The largest monthly increase occurred between March and April, with
st


an increase of
i


eg


Explanation for Incorrect Answer A :
Choice (A) is not correct. Between January and February the profits did increase by
nR

about but between March and April profits increased by about


Explanation for Incorrect Answer B :

Choice (B) is not correct. Between February and March profits decreased.

Explanation for Incorrect Answer D :


Choice (D) is not correct. Between April and May profits decreased.

Explanation for Incorrect Answer E :


Choice (E) is not correct. Between May and June, profits increased by about
but between March and April profits increased by about

In the figure above, the measure of is the measure of If the

file://E:\\c4.htm 2006-11-12
The Official SAT Online Course 3/12

measure of is the measure of a right angle, what is the measure of

(A)

(B)

(C)

(D)

(E)

ANSWERS AND EXPLANATIONS


Explanation for Correct Answer A :

Choice (A) is correct. A right angle measures so

Since is the measure of



Explanation for Incorrect Answer B :

ed
Choice (B) is not correct. This would be the measure of if its measure


were the measure of a right angle, instead of the measure of

r

Explanation for Incorrect Answer C :
te
Choice (C) is not correct. This choice is the measure of but the


is
problem states that the measure of is the measure of


eg

Explanation for Incorrect Answer D :


Choice (D) is not correct. This is the measure of not the measure of


nR

Explanation for Incorrect Answer E :


Choice (E) is not correct. The measure of
which is less than so the measure of
is less than the measure of
cannot be more than

Each square in the grid above is to be filled with either or Each number to the
right of the grid is the sum of the numbers in the row to its left, and each number
below the grid is the sum of the numbers in the column above it. For example, there
is a below the third column because the sum of the numbers in that column is
When the and are all entered correctly into the grid, what will row be?

(A)

file://E:\\c4.htm 2006-11-12
The Official SAT Online Course 4/12

(B)

(C)

(D)

(E)

ANSWERS AND EXPLANATIONS


Explanation for Correct Answer E :
Choice (E) is correct. Since the sum of row is there cannot be any in
row and it must be filled with On the other hand, the sum for row is
so the three empty boxes in that row can be filled in with The same is true
for the three empty boxes in the second column, which also has a sum of The
first column, in order to have a sum of must have zeros except for the already
in the first position in row At this point, there are only two empty boxes, the
column four boxes on row and on row Row needs a in the column four
box to have a total sum of In summary, row reads (from left to right)



d
Explanation for Incorrect Answer A :

re
Choice (A) is not correct. Row must have a in the first column and since the
sum in the first column is only row


cannot also have a in the first column.


te
Explanation for Incorrect Answer B :
Choice (B) is not correct. This choice contradicts the given information that the
third column is all zeroes.


is
Explanation for Incorrect Answer C :
Choice (C) is not correct. This choice contradicts the given information that the


eg

third column is all zeroes.

Explanation for Incorrect Answer D :


Choice (D) is not correct. Without a in the second column of row the second
nR

column would not have a sum of



5
U

If which of the following can be true?

I.

II.

III.

(A) None
(B) I only
(C) II only

(D) I and II only

(E) I, II, and III

ANSWERS AND EXPLANATIONS


Explanation for Correct Answer E :
Choice (E) is correct. Since either or or
both. If then If then It is possible for either
or to equal or for both and to equal Therefore, I, II, and III can
all be true.

file://E:\\c4.htm 2006-11-12
The Official SAT Online Course 5/12

Explanation for Incorrect Answer A :


Choice (A) is not correct. Since either or or both, must
equal

Explanation for Incorrect Answer B :


Choice (B) is not correct. It is possible that but it is also possible that
and

Explanation for Incorrect Answer C :


Choice (C) is not correct. It is possible that but it is also possible that
and

Explanation for Incorrect Answer D :


Choice (D) is not correct. It is possible that both and are equal to and so
can be true.



d
re

te

is


eg

Which of the following could be the equation of the graph of function shown


above?
nR


(A)

(B)

(C)

(D)

(E)

ANSWERS AND EXPLANATIONS


Explanation for Correct Answer B :
Choice (B) is correct. The graph shown intersects the -axis at only one point,
This implies that the quadratic function has the form
where is a constant. Every answer choice in this problem has leading coefficient
so the only possible choice that could represent the graph shown is the

equation

Explanation for Incorrect Answer A :


Choice (A) is not correct. The graph of this equation intersects the -axis at two
points, and because This equation
cannot represent the graph shown.

file://E:\\c4.htm 2006-11-12
The Official SAT Online Course 6/12

Explanation for Incorrect Answer C :


Choice (C) is not correct. The graph of this equation does not intersect the -axis
at the point because not This equation cannot
represent the graph shown.

Explanation for Incorrect Answer D :


Choice (D) is not correct. The graph of this equation does not intersect the -axis
at the point because not This equation cannot
represent the graph shown.

Explanation for Incorrect Answer E :


Choice (E) is not correct. The graph of this equation does not intersect the -axis
at the point because not This equation cannot
represent the graph shown.



e d

In above, and er is the midpoint of What is the length of

(A) (approximately
)
t

is
(B) (approximately )
(C) (approximately )


eg

(D) (approximately )
(E) (approximately )


nR

ANSWERS AND EXPLANATIONS


Explanation for Correct Answer A :
Choice (A) is correct. Since it is given that the measures of two of the three angles

of triangle are triangle is equilateral. Also, since is the

midpoint of and Since is


isosceles and the measures of and are Thus, must
be a triangle. Based on the properties of
triangles,

Explanation for Incorrect Answer B :


Choice (B) is not correct. Since it is given that the measures of two of the three
angles of triangle are triangle is equilateral. Also,

and Since is isosceles and the


measures of and are Thus, must be a
The properties of such a special right angle triangle do not
include sides with lengths and one that is an integer multiple of

Explanation for Incorrect Answer C :


Choice (C) is not correct. An answer of about is too large. Since the three
line segments and form a triangle, the length of must be less
than the sum of the lengths of and Because both and have
length the length of must be less than

file://E:\\c4.htm 2006-11-12
The Official SAT Online Course 7/12

Explanation for Incorrect Answer D :


Choice (D) is not correct. An answer of about is too large. Since the three line
segments and form a triangle, the length of must be less than
the sum of the lengths of and Because both and have length
the length of must be less than

Explanation for Incorrect Answer E :


Choice (E) is not correct. An answer of about is too large. Since the three line
segments and form a triangle, the length of must be less than
the sum of the lengths of and Because both and have length
the length of must be less than

8 If and are positive numbers, what percent of is

(A)

(B)


(C)


(D)

d
(E)

re

te
ANSWERS AND EXPLANATIONS


is
Explanation for Correct Answer E :
Choice (E) is correct. To find what fraction is of divide by


eg

To put this in percent form, multiply by to get .


nR


Explanation for Incorrect Answer A :

Choice (A) is not correct. If the formula were correct, it would work

for any values of and Certainly, is of However, using and

in this formula gives not

Explanation for Incorrect Answer B :

Choice (B) is not correct. If the formula were correct, it would work for

any values of and Certainly, is of However, using and

in this formula gives not

Explanation for Incorrect Answer C :

Choice (C) is not correct. If the formula were correct, it would work

for any values of and Certainly, is of However, using and

in this formula gives not

Explanation for Incorrect Answer D :

Choice (D) is not correct. If the formula were correct, it would work

file://E:\\c4.htm 2006-11-12
The Official SAT Online Course 8/12

for any values of and Certainly, is of However, using and

in this formula gives not

9
How many -pound sticks of butter together weigh as much as pounds of

butter?

Your Response:

Correct Response(s): 100

Explanation:

The correct answer is Since the weight of each stick is pound, sticks

together make a pound. To get pounds, requires sticks.

The problem can also be represented algebraically. Let


be the number of sticks


d
needed, then therefore, or

re

te
10
If , what is the value of


is

Your Response:


eg

Correct Response(s): 3

Explanation:


nR


The correct answer is . The equation can be solved for

using the following steps: and

11 In isosceles triangle the measure of angle is If another angle of the


triangle measures where what is one possible value of

Your Response:

Correct Response(s): 20 or 50

Explanation:

The correct answers are and . The sum of the measures of the angles must
be and the question states that Since the triangle is isosceles, two
of the angles must have the same measure. Either the angles are and
or the angles are and This means that either
or In the first case, and in the
second case Either answer is correct.

file://E:\\c4.htm 2006-11-12
The Official SAT Online Course 9/12

12
If is directly proportional to and if when what is the value of

when

Your Response:

Correct Response(s): 8/15, .533

Explanation:

The correct answer is or (a truncated form of the repeating decimal). If

is directly proportional to then for some constant Using the value

of when gives the equation Solving for gives that

Then for or . The answer can be

entered into the grid either as a fraction or as a decimal.



d
13

re

te

is


eg


nR

The Lyndhurst High School twelfth graders are represented in the circle graph in
Figure 1. Figure 2 is another way to illustrate the use of computers by these twelfth
graders. If the same twelfth graders are represented in both figures, what is the
total number of twelfth graders represented by the shaded circle in Figure 2?

Your Response:

Correct Response(s): 180

Explanation:

The correct answer is . The shaded circle in Figure 2 represents all the
students who use computers in school, including those who use computers both at
home and at school. From Figure 1 in the question, of the students, or

file://E:\\c4.htm 2006-11-12
The Official SAT Online Course 10/12

students, use computers at school only, and of the students, or


students, use computers both at home and at school. The entire shaded circle,
then, represents students who use computers at school.

14

In the figure above, the lengths and widths of rectangles and are whole
numbers. The areas of rectangles and are and respectively.
What is the area of the entire figure?

Your Response:

Correct Response(s): 144

Explanation:



d
The correct answer is . Rectangle has an area of which can be
factored as or Rectangle and rectangle share a side. The

re
length of the shared side is either or since the area of rectangle is


which has and as possible factors. If the length of the shared side were
then the dimensions of rectangle would be by and would be the


te
length of the side shared by rectangle and rectangle Since the area of
rectangle is it cannot have a side of length From this it follows that the
length of the side shared by rectangle and rectangle must be The length


is
of the side shared by rectangle and rectangle is Since the area of


eg

rectangle is the other dimension of rectangle is Since

rectangle has dimensions by rectangle has dimensions by and


rectangle has dimensions by the dimensions of the large rectangle are
nR

by Therefore, the total area of the large rectangle is

15

In the chart above, if the number chosen in step is what number will be the
result of step

Your Response:

Correct Response(s): 20

Explanation:

file://E:\\c4.htm 2006-11-12
The Official SAT Online Course 11/12

The correct answer is . First take the chosen number for step which is
and use it in step Here is not prime, because So add
to get Then in step divide this number by to get The
result of step is the greatest integer less than or equal to which is .

16 In an art class, there were just enough staplers, rulers and glue bottles so that every
students had to share a stapler, every students had to share a ruler, and every
students had to share a glue bottle. If the sum of the number of staplers, rulers,
and glue bottles used by the class was how many students were in the class?

Your Response:

Correct Response(s): 60

Explanation:

The correct answer is . Let represent the total number of students in the art

class. Since every students had to share a stapler, there were exactly

staplers. Since every



students had to share a ruler, there were exactly


d
rulers. Since every students had to share a glue bottle, there were exactly

e
glue bottles. From the given information that the sum of the number of staplers,
rulers, and glue bottles used by the class was it follows that
er
which simplifies to and further to


t
Therefore,


is


eg

17
If and are integers such that and what is the greatest

possible value of


nR


Your Response:

Correct Response(s): 608

Explanation:

The correct answer is . Since and since and have no

common factors other than it must be the case that is a multiple of and

is a multiple of Solving for in terms of yields Since

or or The largest

multiple of less than is So and and

which is less than

18

How many positive integers less than are multiples of and are equal to
times an even integer?

Your Response:

file://E:\\c4.htm 2006-11-12
The Official SAT Online Course 12/12

Correct Response(s): 33

Explanation:

The correct answer is . Since all of these integers are equal to times an even
integer, they are all multiples of and It is given that these integers are also
multiples of Since these integers must be multiples of and they must
all be multiples of the least common multiple of these numbers, which is
The integers must also be less than Integers that fulfill all
these conditions are and so on, up to
which is the greatest such number since is not
less than So there are integers that meet these conditions.

Back to Score Report

Copyright 2006 The College Board. All rights reserved. Privacy Policy Terms of Use Contact Us



e d

er

st


eg


nR

file://E:\\c4.htm 2006-11-12
The Official SAT Online Course 1/21

Help | Profile | My Organizer | My Bookmarks | Logout

Answers and Explanations

Test Sections Back to Score Report

Section 1 View Answers and Explanations


Section 2 Online - Practice Test #3

Section 3
1 Industrial growth that was being stifled by the countrys dictatorship, but now they
Section 4
are developing their full economic potential.
Section 5
(A) Industrial growth that was being stifled by the countrys dictatorship, but
Section 7
now they are developing their full economic potential.
Section 8
(B) The dictatorship had stifled industrial growth, but the country is now
Section 9 developing their full economic potential.
Section 10 (C) Industrial growth was stifled by the countrys dictatorship, and so now
they are developing their full economic potential.
(D) Though the dictatorship had stifled industrial growth, the country is now
developing its full economic potential.
(E)
had stifled industrial growth.

Now developing their full economic potential, the countrys dictatorship


d
ANSWERS AND EXPLANATIONS
Explanation for Correct Answer D :

re
Choice (D) is correct. It avoids the error of the original by properly combining a
te

dependent clause (before the comma) with an independent clause (after the
comma) to state a complete thought. The logical relationship of the two parts of the
sentence is introduced by the subordinating conjunction "though."


is


eg

Explanation for Incorrect Answer A :


Choice (A) is a sentence fragment. Although the clause after the coordinating


conjunction "but" is complete, the phrase before the conjunction has no
independent verb and therefore does not state a complete thought.
nR


Explanation for Incorrect Answer B :
Choice (B) has an error in pronoun agreement. The plural pronoun "their" does not
agree with the singular noun to which it presumably refers, "country."

Explanation for Incorrect Answer C :


Choice (C) uses vague pronouns. The sentence contains no nouns to which the
plural pronouns "they" and "their" can refer.

Explanation for Incorrect Answer E :


Choice (E) exhibits improper modification. The introductory phrase, "Now
developing their full economic potential," cannot logically modify the noun that
follows, "dictatorship."

2
Looking down through the boats glass bottom, a school of yellow fish was seen
swimming along with the turtles.

(A) a school of yellow fish was seen


(B) a school of yellow fish were seen
(C) we saw a school of yellow fish

(D) we seen a school of yellow fish

(E) yellow fish in schools are seen

ANSWERS AND EXPLANATIONS

file://E:\\c5.htm 2006-11-12
The Official SAT Online Course 2/21

Explanation for Correct Answer C :


Choice (C) is correct. It avoids the error of the original by placing the pronoun "we"
immediately after the introductory phrase ("Looking down through the boats glass
bottom") that modifies it.

Explanation for Incorrect Answer A :


Choice (A) exhibits improper modification. The introductory phrase, "Looking down
through the boats glass bottom," cannot logically modify the noun phrase that
immediately follows, "a school of yellow fish."

Explanation for Incorrect Answer B :


Choice (B) has an error in subject-verb agreement. Although the plural verb "were"
agrees with the plural intervening noun "fish," it does not agree with the singular
subject "school."

Explanation for Incorrect Answer D :


Choice (D) uses an incorrect form of the irregular verb. The context requires not the
past participle, "seen," but the past tense, "saw."

Explanation for Incorrect Answer E :


Choice (E) displays improper modification. The introductory phrase, "Looking down
through the boats glass bottom," does not modify the noun that immediately
follows, "fish."



d
3

re
A radio system consists of a means of transforming sounds into electromagnetic
waves and of transmitting those waves through space; after this those waves must


te
be transformed back into sounds.

(A) waves and of transmitting those waves through space; after this those
waves must be transformed


is
(B) waves, transmitting the waves through space, and transforming them

(C) waves, of transmitting them through space, and then the translation of


eg

them
(D) waves and of transmitting them through space; after this the waves have
to be translated


(E) waves, of the transmitting of those waves through space and of
nR

translating same


ANSWERS AND EXPLANATIONS

Explanation for Correct Answer B :


Choice (B) is correct. It avoids the error of the original by using three parallel
verbal phrases ("transforming sounds," "transmitting the waves," and "transforming
them") to describe the three functions of a radio system.

Explanation for Incorrect Answer A :


Choice (A) fails to maintain parallelism. It uses parallel verbal phrases
("transforming sounds" and "transmitting those waves") to describe the first two
functions of a radio system but shifts (after the semicolon) to an independent
clause to describe the third function.

Explanation for Incorrect Answer C :


Choice (C) violates parallelism. The noun "translation" is not parallel with the earlier
verbal forms "transforming" and "transmitting."

Explanation for Incorrect Answer D :


Choice (D) displays a flaw in parallelism. The independent clause used (after the
semicolon) to describe the third function of a radio system is not parallel with the
verbal phrases ("transforming sounds" and "transmitting them") that describe the
first two functions.

Explanation for Incorrect Answer E :


Choice (E) uses an inappropriate idiom. As a substitute for the noun "waves," the
pronoun "same" is less effective than the pronoun "them."

file://E:\\c5.htm 2006-11-12
The Official SAT Online Course 3/21

4 It underlies the poem that human beings are free to choose and may be blamed for
their choices.
(A) It underlies the poem
(B) In the poem, they assumed
(C) In the poem, a basic assumption which is made is

(D) It is an assumption that underlies the poem

(E) The basic assumption of the poem is

ANSWERS AND EXPLANATIONS


Explanation for Correct Answer E :
Choice (E) is correct. It avoids the error of the original by using a specific noun,
"assumption," instead of the vague pronoun "it."

Explanation for Incorrect Answer A :


Choice (A) uses a vague pronoun. Since the pronoun "it" apparently refers to the
entire idea stated by the long dependent clause (introduced by "that"), the meaning
is unclear.


d
Explanation for Incorrect Answer B :
Choice (B) uses a vague pronoun. The sentence contains no noun prior to the

re
pronoun "they" to which that pronoun can refer.

Explanation for Incorrect Answer C :



te
Choice (C) exhibits wordiness. The clause "which is made" can be deleted without
any loss of meaning.


is
Explanation for Incorrect Answer D :
Choice (D) contains an unnecessary word. With a minor change in word order ("An
assumption that underlies the poem is"), the vague pronoun "it" can be eliminated.


eg


nR

The modern city may not have new citadels or cathedrals, but there is a great many


new office buildings and freeways.

(A) but there is a great many new office buildings and freeways

(B) but it does have a great many new office buildings and freeways
(C) but a great many new office buildings and freeways

(D) although many new office buildings and freeways are there

(E) although a great many new office buildings and freeways are seen

ANSWERS AND EXPLANATIONS


Explanation for Correct Answer B :
Choice (B) is correct. It avoids the error of the original by using both a singular
subject, "it," and a singular verb, "does have," to form a clause that is parallel to
the earlier clause, "The modern city may not have."

Explanation for Incorrect Answer A :


Choice (A) involves subject-verb disagreement. In the inverted construction
introduced by "there," the singular verb "is" does not agree with its plural
compound subject, "buildings and freeways."

Explanation for Incorrect Answer C :


Choice (C) fails to maintain parallelism. The phrase without any verb ("but a great
many new office buildings and freeways") is not parallel with the independent
clause before the comma.

file://E:\\c5.htm 2006-11-12
The Official SAT Online Course 4/21

Explanation for Incorrect Answer D :


Choice (D) violates parallelism. The phrase "are there" (with "office buildings and
freeways" as its subject) is not parallel with the earlier verb phrase "may not
have" (with "citadels or cathedrals" as its direct object).

Explanation for Incorrect Answer E :


Choice (E) displays flawed parallelism. The verb phrase "are seen" (with "office
buildings and freeways" as its subject) is not parallel with the earlier verb phrase
"may not have" (with "citadels or cathedrals" as its direct object).

6 The remains of the Apatosaurus provide evidence of there being giants existing on
Earth during the late Jurassic period.
(A) of there being giants existing
(B) of there having been giants existing
(C) of there existing giants

(D) that giants have existed

(E) that giants existed


ANSWERS AND EXPLANATIONS
Explanation for Correct Answer E :


Choice (E) is correct. It avoids the error of the original by using a straightforward

d
clause ("that giants existed") and by eliminating unnecessary words.

e
er

Explanation for Incorrect Answer A :
Choice (A) displays wordiness. Five words ("of there being giants existing") can be
t
reduced to three ("that giants existed").


is
Explanation for Incorrect Answer B :
Choice (B) exhibits wordiness. The idea expressed with six words ("of there having


eg

been giants existing") can be stated more smoothly and efficiently with only three
("that giants existed").


Explanation for Incorrect Answer C :
nR

Choice (C) contains an unnecessary word. The awkward phrase, "of there existing
giants," can be simplified and shortened to "of giants existing."


Explanation for Incorrect Answer D :
Choice (D) uses an improper verb tense. The present perfect tense, "have existed,"

incorrectly suggests that the existence of giants continues up to the present time.

7 Farming in that area of the country is inefficient because of their farm machinery
shortage and that they lack the knowledge to operate it.

(A) because of their farm machinery shortage and that they lack the
(B) in that they have a shortage of farm machinery and a lack of
(C) because of the shortage of farm machinery and their lacking the

(D) in that there is both a shortage of farm machinery as well as a lack of

(E) because of the shortage of farm machinery and the lack of

ANSWERS AND EXPLANATIONS


Explanation for Correct Answer E :
Choice (E) is correct. It avoids the error of the original by omitting the vague and
unnecessary pronouns "their" and "they."

Explanation for Incorrect Answer A :

file://E:\\c5.htm 2006-11-12
The Official SAT Online Course 5/21

Choice (A) includes vague pronouns. The sentence contains no plural noun to which
the plural pronouns "their" and "they" can refer.

Explanation for Incorrect Answer B :


Choice (B) uses a vague pronoun. The meaning of the plural pronoun "they"
remains uncertain because the sentence has no plural noun.

Explanation for Incorrect Answer C :


Choice (C) contains a vague pronoun. Since the sentence has no plural noun, the
use of the plural pronoun "their" is incorrect.

Explanation for Incorrect Answer D :


Choice (D) uses an inappropriate idiom. Since the adjective "both" introduces two
parallel nouns ("shortage" and "lack"), the proper conjunction to link them is "and,"
not "as well as."

8 The most popular painting in the exhibit of works by local artists was done by a
seventy-year-old woman, who painted an exquisite self-portrait of herself.

(A) was done by a seventy-year-old woman, who painted an exquisite self-


portrait of herself
(B) was an exquisite self-portrait of a seventy-year-old woman, who painted
herself


(C) was an exquisite self-portrait of a seventy-year-old woman

(D) was done by a seventy-year-old woman, and it is her own exquisite self-


d
portrait
(E) was by a seventy-year-old woman, being an exquisite self-portrait

ANSWERS AND EXPLANATIONS


e re
Explanation for Correct Answer C :


st
Choice (C) is correct. It avoids the wordiness of the original by improving word
order (placing "an exquisite self-portrait" immediately after the linking verb "was")


and by eliminating the unnecessary words "done," "who painted," and "of herself."
i
eg



Explanation for Incorrect Answer A :
nR

Choice (A) uses excess words. With a minor change in word order (placing "an
exquisite self-portrait" immediately after the linking verb "was"), the words "done,"


"who painted," and "of herself" become unnecessary.

Explanation for Incorrect Answer B :

Choice (B) exhibits wordiness. The final clause, "who painted herself," repeats the
idea contained in "self-portrait" and is therefore unnecessary.

Explanation for Incorrect Answer D :


Choice (D) displays wordiness. If the phrase "[an] exquisite self-portrait" is placed
immediately after the linking verb "was," the words "done" and "and it is her own"
are not needed.

Explanation for Incorrect Answer E :


Choice (E) has an excess word. The word "being" is not needed if the noun phrase
"an exquisite self-portrait" is placed before the prepositional phrase "by a seventy-
year-old woman."

9
The tiles are sorted not only by their surface appearance but also according to their
hardness and their capacity of conducting heat.

(A) according to their hardness and their capacity of conducting heat


(B) according to their hardness and of their heat-conducting capacity
(C) by their hardness and if they have the capacity for heat conduction

(D) by their hardness and their capacity for conducting heat

(E) by their hardness and capacity in heat conduction

file://E:\\c5.htm 2006-11-12
The Official SAT Online Course 6/21

ANSWERS AND EXPLANATIONS


Explanation for Correct Answer D :
Choice (D) is correct. It avoids the error of the original by using the same
preposition, "by," to introduce parallel phrases describing the two criteria used in
sorting tiles.

Explanation for Incorrect Answer A :


Choice (A) fails to maintain parallelism. The preposition "according to" is not
parallel with the earlier preposition "by" even though both introduce phrases
describing criteria for sorting tiles.

Explanation for Incorrect Answer B :


Choice (B) displays wordiness. The preposition "of" is unnecessary.

Explanation for Incorrect Answer C :


Choice (C) contains excess words. The phrase "if they have the" can be reduced to
one word"their."

Explanation for Incorrect Answer E :


Choice (E) uses an inappropriate idiom. "In" is not the correct preposition to link
the noun "capacity" with the later phrase "heat reduction."


d
10
interfere with another.

(A) re
Radio frequencies have to be allocated to users so that one transmission will not

one transmission will not interfere with another


te
(B) each transmission cant interfere with anothers
(C) transmitting them will not interfere with one another


is
(D) no transmission is interfered with by another

(E) no one transmission would have interference with the other


eg


ANSWERS AND EXPLANATIONS
nR

Explanation for Correct Answer A :


Choice (A) is correct. It uses the appropriate noun, "transmission," and avoids
excess words in describing an event.

Explanation for Incorrect Answer B :


Choice (B) uses an incorrect pronoun form. The form "another's" refers illogically to
something possessed by another transmission rather than to the transmission itself.

Explanation for Incorrect Answer C :


Choice (C) contains a vague and ambiguous pronoun. The pronoun
"them" (apparently referring to "transmissions") cannot logically refer to either of
the nouns in the sentence, "frequencies" or "users."

Explanation for Incorrect Answer D :


Choice (D) displays wordiness. Four words ("is interfered with by") can be reduced
to two ("interferes with").

Explanation for Incorrect Answer E :


Choice (E) uses excess words. The idea expressed with four words ("would have
interference with") can be stated better with only two ("interferes with").

11 The Pony Express was an ingenious system for carrying mail; it was in existence only
briefly, however, before the telegraph system made it obsolete.

(A) mail; it was in existence only briefly, however,

file://E:\\c5.htm 2006-11-12
The Official SAT Online Course 7/21

(B) mail, for it was in existence only briefly, however,


(C) mail; however, existing only briefly

(D) mail, having existed only briefly

(E) mail, but was existing only briefly

ANSWERS AND EXPLANATIONS


Explanation for Correct Answer A :
Choice (A) is correct. It uses two appropriately worded independent clauses that
are linked correctly by a semicolon.

Explanation for Incorrect Answer B :


Choice (B) uses an inappropriate idiom. The conjunction "for" incorrectly suggests
that the clause to follow will present a reason for the condition described in the
preceding clause.

Explanation for Incorrect Answer C :


Choice (C) exhibits improper coordination. It incorrectly uses a semicolon to link


parts of unequal grammatical rank (a complete thought before the semicolon and a
phrase after it).


d
Explanation for Incorrect Answer D :
Choice (D) displays inappropriate modification and word order. If placed at the

re
beginning of the sentence, the phrase introduced by "having existed" might modify


"The Pony Express," but it does not modify the noun immediately before it, "mail."

Explanation for Incorrect Answer E :


te
Choice (E) uses an inappropriate verb tense. The past progressive tense "was
existing" suggests a continuing state of being that is inconsistent with the
completed state of being indicated by use of the simple past "was" in the first


is
clause.


eg

12


At first we panicked when we discovered we had missed our flight, but then we took a
nR


bus to another airport, where there are several planes leaving for Denver that

evening. No error

ANSWERS AND EXPLANATIONS


Corrected Sentence: At first we panicked when we discovered we had missed our
flight, but then we took a bus to another airport, where there were several planes
leaving for Denver that evening.

Explanation for Correct Answer C :


The error in this sentence occurs at (C), where the tense sequence is incorrect. The
present tense of the verb, "are," is inconsistent with the earlier past-tense verbs,
"panicked," "discovered," and "took."

Explanation for Incorrect Answer A :


There is no error at (A). The past perfect tense, "had missed," is appropriate to
describe an action that was completed before the time of another action in the past
(the discovery).

Explanation for Incorrect Answer B :


There is no error at (B). The past-tense verb "took" is consistent with other verbs in

file://E:\\c5.htm 2006-11-12
The Official SAT Online Course 8/21

past tense ("panicked" and "discovered").

Explanation for Incorrect Answer D :


There is no error at (D). The participle "leaving" functions correctly to introduce an
adjective phrase describing the noun "planes."

Explanation for Incorrect Answer E : There is an error in the sentence.

13
rose was to order
Only after the floodwaters had two feet the mayor willing the

evacuation of No error
some homes.

ANSWERS AND EXPLANATIONS


Corrected Sentence: Only after the floodwaters had risen two feet was the mayor
willing to order the evacuation of some homes.

Explanation for Correct Answer A :


The error in this sentence occurs at (A), where the verb form is incorrect. After the


helping verb "had," the correct form of this irregular verb is not the past, "rose,"

ed
but the past participle, "risen."


er

Explanation for Incorrect Answer B :
There is no error at (B). In this inverted construction, the singular verb "was"
t
agrees with its singular subject, "mayor," and the past tense correctly indicates a


is
time after another action in the past (the rising of the floodwaters).

Explanation for Incorrect Answer C :


eg

There is no error at (C). The infinitive "to order" functions properly to introduce the
adjective phrase that completes the sentence.


Explanation for Incorrect Answer D :
nR

There is no error at (D). The noun "evacuation" serves correctly as the direct object
of the infinitive "to order," and the preposition "of" introduces a phrase that


modifies "evacuation."

Explanation for Incorrect Answer E : There is an error in the sentence.

14

Confucianism is more a code of ethics than like a religion; it presents no deities

but fosters instead a respect for ones ancestors and for an orderly society. No error

ANSWERS AND EXPLANATIONS


Corrected Sentence: Confucianism is more a code of ethics than a religion; it
presents no deities but fosters instead a respect for ones ancestors and for an
orderly society.

Explanation for Correct Answer A :


The error in this sentence occurs at (A), where there is wordiness. After the phrase
"more . . . than," the word "like" is unnecessary and spoils the parallelism of the
two ideas being compared ("a religion" and "a code") in the opening clause.

file://E:\\c5.htm 2006-11-12
The Official SAT Online Course 9/21

Explanation for Incorrect Answer B :


There is no error at (B). The conjunction "but" is appropriate to indicate a contrast,
and the singular verb "fosters" agrees with its singular subject, "it."

Explanation for Incorrect Answer C :


There is no error at (C). The adverb "instead" functions correctly after the verb
"fosters" to indicate an alternative goal.

Explanation for Incorrect Answer D :


There is no error at (D). The adjective "orderly" appropriately describes the noun
"society."

Explanation for Incorrect Answer E : There is an error in the sentence.

15
Just as parents vary in their readiness to have their children leave home for college,

young people vary in his or her readiness to leave. No error



d
ANSWERS AND EXPLANATIONS

e
Corrected Sentence: Just as parents vary in their readiness to have their children


leave home for college, young people vary in their readiness to leave.
er
Explanation for Correct Answer C :


The error in this sentence occurs at (C), where the number of the pronouns is
incorrect. The singular pronouns "his" and "her" do not agree with the plural noun
t
to which they refer, "people."


is


eg

Explanation for Incorrect Answer A :


There is no error at (A). The phrase "Just as" is an appropriate idiom to introduce a
comparison between parents and young people.
nR


Explanation for Incorrect Answer B :
There is no error at (B). The verb "vary" is plural and agrees with its plural subject,
"people."

Explanation for Incorrect Answer D :


There is no error at (D). The infinitive "to leave" functions correctly as an adjective
to modify the noun immediately before it, "readiness."

Explanation for Incorrect Answer E : There is an error in the sentence.

16
Local party organizations have discovered that voter turnout is diminished

considerably whenever the media projected election results early in the day. No error

ANSWERS AND EXPLANATIONS


Corrected Sentence: Local party organizations have discovered that voter turnout
is diminished considerably whenever the media project election results early in the
day.

Explanation for Correct Answer C :


The error in this sentence occurs at (C), where the tense sequence is incorrect. The

file://E:\\c5.htm 2006-11-12
The Official SAT Online Course 10/21

past tense of the verb, "projected," is inconsistent with the present tense of the
earlier verb, "is diminished."

Explanation for Incorrect Answer A :


There is no error at (A). The adverb "considerably" correctly modifies the verb
immediately before it.

Explanation for Incorrect Answer B :


There is no error at (B). The subordinating conjunction "whenever" properly
introduces the dependent adverbial clause that completes the sentence.

Explanation for Incorrect Answer D :


There is no error at (D). The prepositional phrase "in the day" functions correctly as
an adverb telling when.

Explanation for Incorrect Answer E : There is an error in the sentence.

17
Absent from the speech were any mention of the students and laboratory technicians


upon whose contributions the chemist had depended heavily. No error


ed
ANSWERS
er
AND EXPLANATIONS


Corrected Sentence: Absent from the speech was any mention of the students
st
and laboratory technicians upon whose contributions the chemist had depended


heavily.
i

Explanation for Correct Answer B :


eg

The error in this sentence occurs at (B), where the verb does not agree with its
subject. In this inverted construction, the plural verb "were" does not agree with
the singular subject "mention."


nR


Explanation for Incorrect Answer A :


There is no error at (A). The predicate adjective "absent" functions correctly to
U

describe the noun "mention," and the preposition "from" introduces an adverb
phrase that modifies "absent."

Explanation for Incorrect Answer C :


There is no error at (C). The relative pronoun "whose" serves appropriately to link
the preposition "upon" with its object, "contributions."

Explanation for Incorrect Answer D :


There is no error at (D). The adverb "heavily" is used properly to modify the verb
"had depended."

Explanation for Incorrect Answer E : There is an error in the sentence.

18

Pauls letter to myself about the missing money was not intended to be read by

any other member of the organization. No error

file://E:\\c5.htm 2006-11-12
The Official SAT Online Course 11/21

ANSWERS AND EXPLANATIONS


Corrected Sentence: Pauls letter to me about the missing money was not
intended to be read by any other member of the organization.

Explanation for Correct Answer A :


The error in this sentence occurs at (A), where the pronoun is incorrect. The
reflexive form, "myself," is not appropriate, since the same person did not both
send and receive the letter.

Explanation for Incorrect Answer B :


There is no error at (B). The singular helping verb "was" agrees with its singular
subject, "letter," and the past participle form "intended" is correct after the helping
verb.

Explanation for Incorrect Answer C :


There is no error at (C). The infinitive phrase "to be read" functions appropriately as
an adverb telling how the letter was intended.

Explanation for Incorrect Answer D :


There is no error at (D). The adverb "any" serves correctly to modify the adjective
"other," and "other" properly modifies the noun "member."


ed
Explanation for Incorrect Answer E : There is an error in the sentence.


19
er

After the prince characterized modern architecture as ugly, he has been severely
t

is
criticized for having been so outspoken in public. No error


eg

ANSWERS AND EXPLANATIONS


nR

Corrected Sentence: After the prince characterized modern architecture as ugly,


he was severely criticized for having been so outspoken in public.

Explanation for Correct Answer B :


The error in this sentence occurs at (B), where the tense sequence is inappropriate.
U

The use of "After" with the past tense ("characterized") in the introductory clause
requires the use of the past tense ("was . . . criticized") rather than the present
perfect tense ("has been . . . criticized") in the main clause.

Explanation for Incorrect Answer A :


There is no error at (A). The word "as" correctly precedes the adjective "ugly" after
the verb "characterized."

Explanation for Incorrect Answer C :


There is no error at (C). The adverb "severely" is appropriate to tell how the prince
was criticized.

Explanation for Incorrect Answer D :


There is no error at (D). The adjective "outspoken" is used correctly after the verbal
"having been," and the adverb "so" properly modifies "outspoken."

Explanation for Incorrect Answer E : There is an error in the sentence.

20

file://E:\\c5.htm 2006-11-12
The Official SAT Online Course 12/21

No matter cautious are of damaging


how snowmobiles are driven, they capable the

No error
land over which they travel.

ANSWERS AND EXPLANATIONS


Corrected Sentence: No matter how cautiously snowmobiles are driven, they are
capable of damaging the land over which they travel.

Explanation for Correct Answer B :


The error in this sentence occurs at (B), where an adjective is used incorrectly. An
adverb ("cautiously") rather than the adjective ("cautious") is needed to modify the
verb "are driven."

Explanation for Incorrect Answer A :


There is no error at (A). The phrase "No matter" is an appropriate idiom to link the


introductory dependent clause to the sentence's main clause.


Explanation for Incorrect Answer C :

d
There is no error at (C). The plural verb "are" agrees with its plural subject, "they,"
and its present tense is consistent with that of the earlier verb, "are driven."

Explanation for Incorrect Answer D :

re
There is no error at (D). The preposition "of" is appropriate to begin the phrase that


te
explains what snowmobiles can do, and the gerund "damaging" serves
appropriately as the object of the preposition.


Explanation for Incorrect Answer E : There is an error in the sentence.
is


eg

21


The black squirrels drew a crowd of students, for it had never been seen on the
nR


campus before. No error

ANSWERS AND EXPLANATIONS


Corrected Sentence: The black squirrels drew a crowd of students, for they had
never been seen on the campus before.

Explanation for Correct Answer C :


The error in this sentence occurs at (C), where the number of the pronoun is
incorrect. The singular pronoun "it" does not agree with the plural noun to which it
refers, "squirrels."

Explanation for Incorrect Answer A :


There is no error at (A). The past-tense verb "drew" is correct to indicate a time
after an earlier time in the past (when the squirrels were not seen).

Explanation for Incorrect Answer B :


There is no error at (B). The conjunction "for" is used appropriately to join a clause
describing an effect with a later clause stating a cause.

Explanation for Incorrect Answer D :


There is no error at (D). After the helping verb "had," the verb forms "been" and
"seen" are correct to form passive voice and past perfect tense.

file://E:\\c5.htm 2006-11-12
The Official SAT Online Course 13/21

Explanation for Incorrect Answer E : There is an error in the sentence.

22
students who attended
A majority of the the job fair expressed interest in

becoming a doctor or lawyer No error


.

ANSWERS AND EXPLANATIONS


Corrected Sentence: A majority of the students who attended the job fair
expressed interest in becoming doctors or lawyers.

Explanation for Correct Answer D :


The error in this sentence occurs at (D), where there is noun-noun disagreement.
The singular nouns "doctor" and "lawyer" do not agree with the earlier plural noun,
"students."



d
Explanation for Incorrect Answer A :
There is no error at (A). The noun "students" is used properly as the object of the

re
preposition "of," and the relative pronoun "who" functions correctly to introduce an
adjective clause describing "students."



te
Explanation for Incorrect Answer B :
There is no error at (B). The verb in past tense, "attended," is consistent with the
later past-tense verb, "expressed."


is
Explanation for Incorrect Answer C :
There is no error at (C). The gerund "becoming" serves appropriately as the object


of the preposition "in."
eg

Explanation for Incorrect Answer E : There is an error in the sentence.


nR


23

Never before had a group of artists been so isolated from society and from official

patronage as was the so-called Impressionists. No error

ANSWERS AND EXPLANATIONS


Corrected Sentence: Never before had a group of artists been so isolated from
society and from official patronage as were the so-called Impressionists.

Explanation for Correct Answer D :


The error in this sentence occurs at (D), where there is subject-verb disagreement.
In this inverted construction, the singular verb "was" does not agree with its plural
subject, "Impressionists."

Explanation for Incorrect Answer A :


There is no error at (A). The helping verb "had" is used correctly to produce past
perfect tense and thus indicate a time prior to the time of the Impressionists.

Explanation for Incorrect Answer B :


There is no error at (B). After the helping verb "had been," the past participle form,

file://E:\\c5.htm 2006-11-12
The Official SAT Online Course 14/21

"isolated," is correct, and "so" is an appropriate adverb to modify the verb.

Explanation for Incorrect Answer C :


There is no error at (C). The conjunction "and" is appropriate to join two parallel
prepositional phrases, and the repetition of the preposition "from" makes the
parallelism more emphatic.

Explanation for Incorrect Answer E : There is an error in the sentence.

24
ordered to their mother less
The flowers that Jane and Jonathan to be sent were fresh

Carr's Flower Shop No error


and much more expensive than .

ANSWERS AND EXPLANATIONS


Corrected Sentence: The flowers that Jane and Jonathan ordered to be sent to


their mother were less fresh and much more expensive than those from Carr's
Flower Shop.


Explanation for Correct Answer D :

d
The error in this sentence occurs at (D), where the comparison is illogical. The
flowers ordered are compared to a flower shop rather than to flowers from that
shop.

re

te
Explanation for Incorrect Answer A :


is
There is no error at (A). The past-tense verb "ordered" is consistent with the later
verb in past tense, "were."


eg

Explanation for Incorrect Answer B :


There is no error at (B). The prepositional phrase "to their mother" functions
correctly as an adverb telling where.


nR

Explanation for Incorrect Answer C :


There is no error at (C). The adverb "less" correctly modifies the adjective "fresh."


Explanation for Incorrect Answer E : There is an error in the sentence.

25

A possible first step in developing a nonsexist vocabulary with which to analyze the

works of the nineteenth-century writer Elizabeth Gaskell would be to stop referring to

her as Mrs. Gaskell. No error

ANSWERS AND EXPLANATIONS


Corrected Sentence: A possible first step in developing a nonsexist vocabulary
with which to analyze the works of the nineteenth-century writer Elizabeth Gaskell
would be to stop referring to her as Mrs. Gaskell.

Explanation for Correct Answer E : There is no error in this sentence.

file://E:\\c5.htm 2006-11-12
The Official SAT Online Course 15/21

Explanation for Incorrect Answer A :


There is no error at (A). The preposition "in" and its object, the gerund
"developing," correctly begin an adjective phrase that describes the noun "step."

Explanation for Incorrect Answer B :


There is no error at (B). A phrase describing the noun "vocabulary" is properly
introduced by the preposition "with" and its object, the pronoun "which."

Explanation for Incorrect Answer C :


There is no error at (C). The preposition "of" is appropriate to introduce a phrase
that describes the noun "works."

Explanation for Incorrect Answer D :


There is no error at (D). The infinitive "to stop" functions correctly after the linking
verb "would be" to name a possible first step.

26
After 140 years under the sea, the remains of the Monitor, an ironclad warship that

was sunk during the Civil War, is being gradually brought to the surface. No error



d
ANSWERS AND EXPLANATIONS

re
Corrected Sentence: After 140 years under the sea, the remains of the Monitor,
an ironclad warship that was sunk during the Civil War, are being gradually brought


te
to the surface.

Explanation for Correct Answer C :


is
The error in this sentence occurs at (C), where the verb does not agree with its
subject. Although the singular verb "is" agrees with the interrupting noun
"warship," it does not agree with the plural subject "remains."


eg


nR

Explanation for Incorrect Answer A :


There is no error at (A). The preposition "after" is appropriate to begin a phrase


that indicates a period of time.


Explanation for Incorrect Answer B :
U

There is no error at (B). The preposition "under" properly introduces the


prepositional phrase "under the sea."

Explanation for Incorrect Answer D :


There is no error at (D). After the helping verb "is being," the correct verb form is
the past participle, "brought," and the preposition "to" properly introduces an
adverb phrase telling where.

Explanation for Incorrect Answer E : There is an error in the sentence.

27

I have gone to only one football game after I graduated from high school. No error

ANSWERS AND EXPLANATIONS


Corrected Sentence: I have gone to only one football game since I graduated
from high school.

Explanation for Correct Answer C :


The error in this sentence occurs at (C), where the idiom is inappropriate. In

file://E:\\c5.htm 2006-11-12
The Official SAT Online Course 16/21

context, the conjunction "after" is less idiomatic than the conjunction "since."

Explanation for Incorrect Answer A :


There is no error at (A). The preposition "to" correctly introduces an adverbial
phrase identifying where the speaker went.

Explanation for Incorrect Answer B :


There is no error at (B). The adverb "only" correctly modifies the adjective "one,"
and "one" is used appropriately to describe the noun "game."

Explanation for Incorrect Answer D :


There is no error at (D). The past tense of the verb, "graduated," is correct to
describe a completed action.

Explanation for Incorrect Answer E : There is an error in the sentence.

28
The radio station received the most number of calls from listeners on the evening it

aired a discussion of the music of Aretha Franklin. No error




d
ANSWERS AND EXPLANATIONS
re

te
Corrected Sentence: The radio station received the largest number of calls from
listeners on the evening it aired a discussion of the music of Aretha Franklin.


is
Explanation for Correct Answer A :
The error in this sentence occurs at (A), where the idiom is inappropriate. Before
"number" the adjective "most" is less idiomatic than "largest."


eg


nR

Explanation for Incorrect Answer B :


There is no error at (B). The prepositional phrase "on the evening" functions
correctly as an adverb telling when.


Explanation for Incorrect Answer C :
U

There is no error at (C). The singular pronoun "it" agrees with the singular noun to
which it refers, "station."

Explanation for Incorrect Answer D :


There is no error at (D). The noun "music" serves appropriately as the object of the
preposition "of," and the subsequent preposition "of" introduces a phrase that
specifies the music that was discussed.

Explanation for Incorrect Answer E : There is an error in the sentence.

29

When the village elders present recommendations, there is hardly ever any opposition

against their proposals. No error

ANSWERS AND EXPLANATIONS


Corrected Sentence: When the village elders present recommendations, there is

file://E:\\c5.htm 2006-11-12
The Official SAT Online Course 17/21

hardly ever any opposition to their proposals.

Explanation for Correct Answer D :


The error in this sentence occurs at (D), where the idiom is not suitable. "Against" is
not the appropriate preposition to introduce a phrase that follows the noun
"opposition."

Explanation for Incorrect Answer A :


There is no error at (A). The plural verb "present" agrees with its plural subject,
"elders," and the present tense is consistent with that of the later verb "is."

Explanation for Incorrect Answer B :


There is no error at (B). In this inverted construction, the singular verb "is" agrees
with its singular subject, "opposition."

Explanation for Incorrect Answer C :


There is no error at (C). By telling when, the adverb "ever" correctly modifies the
verb "is," and another adverb, "hardly," appropriately modifies and limits "ever."

Explanation for Incorrect Answer E : There is an error in the sentence.



(1) People today have placed emphasis on the kinds of work that others do, it is

d
wrong. (2) Suppose a woman says she is a doctor. (3) Immediately everyone
assumes that she is a wonderful person, as if doctors were incapable of doing

re
wrong. (4) However, if you say youre a carpenter or mechanic, some people


think that youre not as smart as a doctor or a lawyer. (5) Cant someone
just want to do this because he or she loves the work ?
te

(6) Also, who decided that the person who does your taxes is more important
than the person who makes sure that your house is warm or that your car
runs ? (7) I know firsthand how frustrating it can be. (8) They think of you


is
only in terms of your job. (9) I used to clean houses in the summer because
the money was good; but yet all the people whose houses I cleaned seemed to
assume that because I was vacuuming their carpets I did not deserve their
eg


respect. (10) One woman came into the bathroom while I was scrubbing the
tub. (11) She kept asking me if I had any questions. (12) Did she want me to
ask whether to scrub the tub counter-clockwise instead of clockwise ? (13) Her


attitude made me angry! (14) Once I read that the jobs people consider
nR

important have changed. (15) Carpenters used to be much more admired than
doctors. (16) My point is, then, that who I want to be is much more important


than what I want to be!

30
Of the following, which is the best way to phrase sentence 1 (reproduced below) ?

People today have placed emphasis on the kinds of work that others do, it is wrong.

(A) (As it is now)

(B) People today place too much emphasis on the kinds of work that others
do.
(C) What kinds of work others do is being placed too much emphasis on by
people today.
(D) The wrong kind of emphasis had been placed on the kinds of work others
do today.
(E) The wrong emphasis is being placed today on people and what kind of
work they do.

ANSWERS AND EXPLANATIONS


Explanation for Correct Answer B :
Choice (B) is correct. The phrase "too much" smoothly embeds a negative
judgment within the first clause, thereby making the second clause unnecessary.

Explanation for Incorrect Answer A :

file://E:\\c5.htm 2006-11-12
The Official SAT Online Course 18/21

Choice (A) is unsatisfactory because it uses a comma improperly to join two


independent clauses.

Explanation for Incorrect Answer C :


Choice (C) is unsatisfactory because the word order is garbled. The preposition
"on" (one of several misplaced words) is far removed from its object, "kinds."

Explanation for Incorrect Answer D :


Choice (D) is unsatisfactory because the past perfect tense ("had been placed") is
inappropriate to describe an action occurring today.

Explanation for Incorrect Answer E :


Choice (E) is unsatisfactory because of wordiness. The sentence does not need both
the noun "people" and the pronoun "they."

31 In context, which of the following is the best way to revise and combine the
underlined portions of sentences 2 and 3 (reproduced below) ?

Suppose a woman says she is a doctor. Immediately everyone assumes that she is a
wonderful person, as if doctors were incapable of doing wrong.

(A) Suppose a woman says she is a doctor, but immediately


(B)


If a woman says she is a doctor, for instance, immediately
(C) When a woman says she is a doctor, however, immediately


d
(D) Immediately, if they say, for example, she is a doctor,

(E)

re
Therefore, a woman is maybe saying she is a doctor; immediately



te
ANSWERS AND EXPLANATIONS
Explanation for Correct Answer B :


is
Choice (B) is correct. In the combined sentence, the dependent clause
(appropriately introduced by "if") states a possible condition, and the main clause
then describes a likely result. "For instance" indicates that the situation illustrates


eg

the statement in sentence 1.


nR

Explanation for Incorrect Answer A :


Choice (A) is unsatisfactory because the connecting word "but" usually introduces a
contrast rather than a result.

Explanation for Incorrect Answer C :


Choice (C) is unsatisfactory because the transition word "however" inappropriately
suggests that a contrasting idea will follow.

Explanation for Incorrect Answer D :


Choice (D) is unsatisfactory because it uses a vague pronoun, "they."

Explanation for Incorrect Answer E :


Choice (E) is unsatisfactory because the introductory word "therefore" is
inappropriate. It incorrectly suggests that this sentence is drawing a conclusion
based on evidence presented earlier.

32

In context, the phrase do this in sentence 5 would best be replaced by

(A) hold this particular opinion


(B) resist temptation
(C) ask someone for assistance

(D) become a carpenter or a mechanic

(E) aspire to learn medicine

file://E:\\c5.htm 2006-11-12
The Official SAT Online Course 19/21

ANSWERS AND EXPLANATIONS


Explanation for Correct Answer D :
Choice (D) is correct. In context, the verb "become" is more precise than "do," and
the nouns "carpenter" and "mechanic" are much more specific than the pronoun
"this."

Explanation for Incorrect Answer A :


Choice (A) is unsatisfactory because its implication is illogical. Holding a negative
opinion would not be the likely result of loving one's work.

Explanation for Incorrect Answer B :


Choice (B) is unsatisfactory because it introduces an irrelevant idea. No other
sentence in the passage even mentions temptation.

Explanation for Incorrect Answer C :


Choice (C) is unsatisfactory because the relationship between the two clauses
becomes illogical. Loving one's work would not necessarily produce a request for
assistance.


Explanation for Incorrect Answer E :
Choice (E) is unsatisfactory because it shifts emphasis away from the value of work
done by carpenters and mechanics. Instead of continuing the idea in sentence 4,


ed
this choice introduces a new thought.

33
r
Which of the following is the best way to revise and combine the underlined portions


te
of sentences 7 and 8 (reproduced below) ?

I know firsthand how frustrating it can be. They think of you only in terms of your


job.
is

(A) be; they--people, that is--think of you


eg

(B) be when they are thinking of one


(C) be how people think of you


nR

(D) be when people think of you


(E) be; having people think of you


ANSWERS AND EXPLANATIONS
U

Explanation for Correct Answer D :


Choice (D) is correct. The subordinating conjunction "when" provides an
appropriate link between the two clauses, and the noun "people" replaces the
vague pronoun "they."

Explanation for Incorrect Answer A :


Choice (A) is unsatisfactory because of wordiness. The pronoun "they" and the
phrase "that is" are not needed.

Explanation for Incorrect Answer B :


Choice (B) is unsatisfactory because it retains the vague pronoun "they" and
introduces another inappropriate pronoun, "one" (that is inconsistent with the later
pronoun "your").

Explanation for Incorrect Answer C :


Choice (C) is unsatisfactory because "how" is not an acceptable transition word to
link the first clause (ending with "be") with the second clause.

Explanation for Incorrect Answer E :


Choice (E) is unsatisfactory because it uses improper coordination. The semicolon
incorrectly links unequal parts (an independent clause before the semicolon and a
phrase after it).

file://E:\\c5.htm 2006-11-12
The Official SAT Online Course 20/21

34 In context, the phrase but yet in sentence 9 would best be replaced by

(A) incidentally,
(B) however,
(C) in fact,

(D) in addition,

(E) for example,

ANSWERS AND EXPLANATIONS


Explanation for Correct Answer B :
Choice (B) is correct. The word "however" properly indicates a contrast between a
positive aspect of the job (good pay) and a negative aspect (lack of respect).


Explanation for Incorrect Answer A :
Choice (A) is unsatisfactory because "incidentally" does not indicate contrast.
Instead, it suggests that the information to follow is of minor importance.


d
Explanation for Incorrect Answer C :

re
Choice (C) is unsatisfactory because the phrase "in fact" does not prepare for a


contrast. It implies only that the second clause will correct or clarify an earlier
misconception.


te
Explanation for Incorrect Answer D :
Choice (D) is unsatisfactory because the phrase "in addition" fails to introduce a
contrast. It actually implies that the second clause will continue or reinforce the


is
idea presented earlier.

Explanation for Incorrect Answer E :


eg

Choice (E) is unsatisfactory because the phrase "for example" does not imply
contrast. It suggests instead that specific details to follow will support an earlier
generalization.


nR

35


The best place to begin a new paragraph in sentences 6-16 would be with sentence

(A) 10
(B) 11
(C) 12

(D) 14

(E) 15

ANSWERS AND EXPLANATIONS


Explanation for Correct Answer D :
Choice (D) is correct. The account of the writer's own experience as a house cleaner
ends with sentence 13, and sentence 14 appropriately begins a new paragraph
about historical changes in attitudes toward different kinds of work.

Explanation for Incorrect Answer A :


Choice (A) is unsatisfactory because such a division would improperly separate a
specific example (starting in sentence 10) from the general statement it illustrates
(in sentence 9).

Explanation for Incorrect Answer B :

file://E:\\c5.htm 2006-11-12
The Official SAT Online Course 21/21

Choice (B) is unsatisfactory because this break would awkwardly interrupt the
narrative beginning in sentence 10 and continuing through sentence 13.

Explanation for Incorrect Answer C :


Choice (C) is unsatisfactory because such an interruption in the narrative (running
from sentence 10 through sentence 13) would be inappropriate.

Explanation for Incorrect Answer E :


Choice (E) is unsatisfactory because the resulting separation of a specific example
(in sentence 15) from the generalization that it supports (in sentence 14) would be
improper.

Back to Score Report

Copyright 2006 The College Board. All rights reserved. Privacy Policy Terms of Use Contact Us



ed
er

t

is


eg


nR

file://E:\\c5.htm 2006-11-12
The Official SAT Online Course 1/22

Help | Profile | My Organizer | My Bookmarks | Logout

Answers and Explanations

Test Sections Back to Score Report

Section 1 View Answers and Explanations


Section 2 Online - Practice Test #3

Section 3
1
Section 4 Much of our knowledge of dinosaurs comes from excavated bones, which, in -------
other clues such as fossilized tracks and eggs, help us to ------- the evolution of these
Section 5
creatures.
Section 7
(A) convergence with. . supplant
Section 8
Section 9 (B) divergence from. . decode
Section 10 (C) dependence on. . belie

(D) opposition to. . amplify

(E) conjunction with. . trace



ANSWERS AND EXPLANATIONS

d
re
Explanation for Correct Answer E :


Choice (E) is correct. "Conjunction with" means together with. "Trace" means track
or follow. If one were to insert these terms into the text, the sentence would read
"Much of our knowledge of dinosaurs comes from excavated bones, which, in


te
conjunction with other clues such as fossilized tracks and eggs, help us to trace the
evolution of these creatures." The first missing term suggests that knowledge and
clues come together. The second missing term indicates what the knowledge and


is
clues enable people to do. It makes sense to say that knowledge and clues work
together to enable people to trace the evolution of dinosaurs.


eg


Explanation for Incorrect Answer A :
nR

Choice (A) is incorrect. "Convergence with" means come together with. "Supplant"
means replace. If one were to insert these terms into the text, the sentence would


read "Much of our knowledge of dinosaurs comes from excavated bones, which, in
convergence with other clues such as fossilized tracks and eggs, help us to supplant
the evolution of these creatures." The first missing term suggests that knowledge

and clues come together. The second missing term indicates what the knowledge
and clues enable people to do. Supplant does not make sense in the sentence
because evolution cannot be replaced.

Explanation for Incorrect Answer B :


Choice (B) is incorrect. "Divergence from" means departing from. "Decode" means
to put a coded message into plain language. If one were to insert these terms into
the text, the sentence would read "Much of our knowledge of dinosaurs comes from
excavated bones, which, in divergence from other clues such as fossilized tracks
and eggs, help us to decode the evolution of these creatures." The first missing
term suggests that knowledge and clues come together. The second missing term
indicates what the knowledge and clues enable people to do. Knowledge and clues
would enable people to decode how dinosaurs evolved However, "divergence from"
does not suggest that the knowledge and clues are coming together.

Explanation for Incorrect Answer C :


Choice (C) is incorrect. "Dependence on" means leaning on. "Belie" means
contradict or fail to confirm. If one were to insert these terms into the text, the
sentence would read "Much of our knowledge of dinosaurs comes from excavated
bones, which, in dependence on other clues such as fossilized tracks and eggs, help
us to belie the evolution of these creatures." The first missing term suggests that
knowledge and clues come together. The second missing term indicates what the
knowledge and clues enable people to do. Bones themselves would not depend on
clues. Additionally, while knowledge and clues might belie, or contradict,
evolutionary theories, they do not belie evolution itself.

Explanation for Incorrect Answer D :


Choice (D) is incorrect. "Opposition to" means against. "Amplify" means make

file://E:\\c6.htm 2006-11-12
The Official SAT Online Course 2/22

larger or greater. If one were to insert these terms into the text, the sentence
would read "Much of our knowledge of dinosaurs comes from excavated bones,
which, in opposition to other clues such as fossilized tracks and eggs, help us to
amplify the evolution of these creatures." The first missing term suggests that
knowledge and clues come together. The second missing term indicates what the
knowledge and clues enable people to do. People cannot amplify the evolution of
creatures.

2 Vernal pools are among the most ------- of ponds: they form as a result of snowmelt
and a high water table in winter, and then they ------- by late summer.
(A) transitory . . expand
(B) anachronistic . . overflow
(C) immutable . . drain

(D) itinerant . . teem

(E) ephemeral . . evaporate

ANSWERS AND EXPLANATIONS


Explanation for Correct Answer E :
Choice (E) is correct. "Ephemeral" means brief. "Evaporate" means disappear by
turning into vapor. If one were to insert these terms into the text, the sentence
would read "Vernal pools are among the most ephemeral of ponds: they form as a


d
result of snowmelt and a high water table in winter, and then they evaporate by
late summer." "Vernal pools" are spring pools. The first missing term is a

re
characteristic of a vernal pool. What follows the colon is an explanation of the first


missing term. Evaporation shows that the ponds disappear after a period of time.
This corresponds with the use of ephemeral to describe a vernal pool.


te

is
Explanation for Incorrect Answer A :
Choice (A) is incorrect. "Transitory" means temporary or brief. "Expand" means


eg

increase in size. If one were to insert these terms into the text, the sentence would
read "Vernal pools are among the most transitory of ponds: they form as a result of
snowmelt and a high water table in winter, and then they expand by late summer."
"Vernal pools" are spring pools. The first missing term is a characteristic of a vernal


pool. What follows the colon is an explanation of the first missing term. A transitory
nR

pond would disappear after a brief time. However, ponds that expand are not
temporary.


Explanation for Incorrect Answer B :
Choice (B) is incorrect. "Anachronistic" means from the wrong time period.

"Overflow" means flow over the limits of something. If one were to insert these
terms into the text, the sentence would read "Vernal pools are among the most
anachronistic of ponds: they form as a result of snowmelt and a high water table in
winter, and then they overflow by late summer." "Vernal pools" are spring pools.
The first missing term is a characteristic of a vernal pool. What follows the colon is
an explanation of the first missing term. A pond cannot be anachronistic.

Explanation for Incorrect Answer C :


Choice (C) is incorrect. "Immutable" means unchangeable. "Drain" means flow
away. If one were to insert these terms into the text, the sentence would read
"Vernal pools are among the most immutable of ponds: they form as a result of
snowmelt and a high water table in winter, and then they drain by late summer."
"Vernal pools" are spring pools. The first missing term is a characteristic of a vernal
pool. What follows the colon is an explanation of the first missing term. A pond that
is immutable would not fill in winter and drain in summer.

Explanation for Incorrect Answer D :


Choice (D) is incorrect. "Itinerant" means roaming or wandering from place to
place. "Teem" means become filled to overflowing. If one were to insert these
terms into the text, the sentence would read "Vernal pools are among the most
itinerant of ponds: they form as a result of snowmelt and a high water table in
winter, and then they teem by late summer." "Vernal pools" are spring pools. The
first missing term is a characteristic of a vernal pool. What follows the colon is an
explanation of the first missing term. Itinerant is not described by the concept of
teeming. As well, a pond stays in the same place; it cannot be itinerant.

file://E:\\c6.htm 2006-11-12
The Official SAT Online Course 3/22

3 The ------- experiences of Madonna Swan, the 1983 North American Indian Woman of
the Year, cannot be fully appreciated if they are ------- in a tidy summary.
(A) varied . . interposed
(B) diverse . . condensed
(C) profound . . magnified

(D) transformative . . embellished

(E) impressive . . immersed

ANSWERS AND EXPLANATIONS


Explanation for Correct Answer B :
Choice (B) is correct. "Diverse" means composed of different elements, and
"condensed" means made compact. If one were to insert these terms into the text,
the sentence would read "The diverse experiences of Madonna Swan, the 1983
North American Indian Woman of the Year, cannot be fully appreciated if they are
condensed in a tidy summary." Experiences combining different elements cannot be
easily compacted in a tidy summary.

Explanation for Incorrect Answer A :



d
Choice (A) is incorrect. "Varied" means mixed, and "interposed" means inserted. If
one were to insert these terms into the text, the sentence would read "The varied
experiences of Madonna Swan, the 1983 North American Indian Woman of the

re
Year, cannot be fully appreciated if they are interposed in a tidy summary." While
varied experiences may, with difficulty, be summarized, it does not make sense to
speak of experiences being inserted into a summary. That would suggest that the


te
summary of experiences already exists.

Explanation for Incorrect Answer C :


is
Choice (C) is incorrect. "Profound" means deep. "Magnified" means enlarged. If one
were to insert these terms into the text, the sentence would read "The profound
experiences of Madonna Swan, the 1983 North American Indian Woman of the


eg

Year, cannot be fully appreciated if they are magnified in a tidy summary." While
deeply meaningful experiences may be summarized, a summary is by definition a
brief version of something. It makes no sense to say that something could be


enlarged in a summary.
nR

Explanation for Incorrect Answer D :


Choice (D) is incorrect. "Transformative" means causing a change. "Embellished"
means enhanced. If one were to insert these terms into the text, the sentence
would read "The transformative experiences of Madonna Swan, the 1983 North


American Indian Woman of the Year, cannot be fully appreciated if they are
U

embellished in a tidy summary." A summary presents main points in a general way;


it does not "embellish," or enhance, those points.

Explanation for Incorrect Answer E :


Choice (E) is incorrect. "Impressive" means inspiring awe or admiration, and
"immersed" means submerged. If one were to insert these terms into the text, the
sentence would read "The impressive experiences of Madonna Swan, the 1983
North American Indian Woman of the Year, cannot be fully appreciated if they are
immersed in a tidy summary." Although impressive experiences might be
mentioned in a summary, it would be strange to describe such experiences as being
submerged.

The representative was a traditionalist, reluctant to support any legislation


inconsistent with the nations most ------- principles.

(A) orthodox
(B) impassioned
(C) precarious

(D) impressionable

(E) indeterminate

file://E:\\c6.htm 2006-11-12
The Official SAT Online Course 4/22

ANSWERS AND EXPLANATIONS


Explanation for Correct Answer A :
Choice (A) is correct. "Orthodox" means conforming to traditional beliefs. If one
were to insert this term into the text, the sentence would read "The representative
was a traditionalist, reluctant to support any legislation inconsistent with the
nation's most orthodox principles." The missing term describes the kind of
legislative principles that a "traditionalist," a person who upholds traditional beliefs,
would support. Orthodox principles are exactly the type of principles a traditionalist
would tend to support.

Explanation for Incorrect Answer B :


Choice (B) is incorrect. "Impassioned" means passionate. If one were to insert this
term into the text, the sentence would read "The representative was a
traditionalist, reluctant to support any legislation inconsistent with the nation's most
impassioned principles." The missing term describes the kind of legislative
principles that a "traditionalist," a person who upholds traditional beliefs, would
support. It is odd to say that principles themselves could be impassioned. This word
is usually used to describe people, not things such as principles. In any case,
impassioned principles would not necessarily be in line with tradition.

Explanation for Incorrect Answer C :


Choice (C) is incorrect. "Precarious" means on edge or unsafe. If one were to insert
this term into the text, the sentence would read "The representative was a


d
traditionalist, reluctant to support any legislation inconsistent with the nation's most
precarious principles." The missing term describes the kind of legislative principles

re
that a "traditionalist," a person who upholds traditional beliefs, would support. A


traditionalist may or may not support principles that are precarious.

Explanation for Incorrect Answer D :


te
Choice (D) is incorrect. "Impressionable" means capable of being easily impressed
or swayed. If one were to insert this term into the text, the sentence would read
"The representative was a traditionalist, reluctant to support any legislation


is
inconsistent with the nation's most impressionable principles." The missing term
describes the kind of legislative principles that a "traditionalist," a person who
upholds traditional beliefs, would support. Impressionable is a word that applies to


eg

people, not to things such as principles.

Explanation for Incorrect Answer E :


Choice (E) is incorrect. "Indeterminate" means vague. If one were to insert this
nR

term into the text, the sentence would read "The representative was a
traditionalist, reluctant to support any legislation inconsistent with the nation's most


indeterminate principles." The missing term describes the kind of legislative
principles that a "traditionalist," a person who upholds traditional beliefs, would
support. Although principles could be described as indeterminate, a traditionalist

may or may not be likely to support them.

5 The author constructed a scenario in which playful, creative children are rewarded for
their ------- and strict, dour adults are punished for their -------.

(A) spontaneity . . rigidity


(B) digressions . . mirth
(C) solemnity . . malice

(D) inflexibility . . rigor

(E) improvisations . . buoyancy

ANSWERS AND EXPLANATIONS


Explanation for Correct Answer A :
Choice (A) is correct. "Spontaneity" is uncontrolled and impulsive behavior.
"Rigidity" means stiffness. If one were to insert these terms into the text, the
sentence would read "The author constructed a scenario in which playful, creative
children are rewarded for their spontaneity and strict, dour adults are punished for
their rigidity." The first missing term should be associated with creativity or
playfulness, and the second should be related to "strict" and "dour," which mean
limiting and stern. Spontaneity is often associated with creativity, and someone
who is strict or unyielding could aptly be described as exhibiting rigidity.

file://E:\\c6.htm 2006-11-12
The Official SAT Online Course 5/22

Explanation for Incorrect Answer B :


Choice (B) is incorrect. "Digressions" are wanderings. "Mirth" is gladness. If one
were to insert these terms into the text, the sentence would read "The author
constructed a scenario in which playful, creative children are rewarded for their
digressions and strict, dour adults are punished for their mirth." The first missing
term should be associated with creativity or playfulness, and the second should be
related to "strict" and "dour," which mean limiting and stern. Mirth does not work in
the sentence because it is not a quality of strict, dour people.

Explanation for Incorrect Answer C :


Choice (C) is incorrect. "Solemnity" means gloomy seriousness. "Malice" means
hatred. If one were to insert these terms into the text, the sentence would read
"The author constructed a scenario in which playful, creative children are rewarded
for their solemnity and strict, dour adults are punished for their malice." The first
missing term should be associated with creativity or playfulness, and the second
should be related to "strict" and "dour," which mean limiting and stern. Solemnity is
the opposite of playfulness, and thus could not be the answer.

Explanation for Incorrect Answer D :


Choice (D) is incorrect. "Inflexibility" means the state of being rigid or unyielding.
"Rigor" means strictness. If one were to insert these terms into the text, the
sentence would read "The author constructed a scenario in which playful, creative


children are rewarded for their inflexibility and strict, dour adults are punished for
their rigor." The first missing term should be associated with creativity or
playfulness, and the second should be related to "strict" and "dour," which mean


limiting and stern. Playful, creative children would most likely not be inflexible.

d
Explanation for Incorrect Answer E :

re
Choice (E) is incorrect. "Improvisations" are things created without planning.
"Buoyancy" means the state of being light or cheerful. If one were to insert these
terms into the text, the sentence would read "The author constructed a scenario in


te
which playful, creative children are rewarded for their improvisations and strict,
dour adults are punished for their buoyancy." The first missing term should be
associated with creativity or playfulness, and the second should be related "strict"


and "dour," which mean limiting and stern. Although improvisations works in the
is
sentence, the second word does not. Buoyancy is not associated with strict, dour
people.


eg


nR

6 Although usually warm and ------- in greeting friends, Lauren was too reserved ever
to be truly -------.


(A)

(B)
joyous . . conventional

cordial . . effusive

(C) restrained . . gracious

(D) dismissive . . ebullient

(E) genial . . antisocial

ANSWERS AND EXPLANATIONS


Explanation for Correct Answer B :
Choice (B) is correct. "Cordial" means warm and friendly. "Effusive" means
unrestrained in emotional expression. If one were to insert these terms into the
text, the sentence would read "Although usually warm and cordial in greeting
friends, Lauren was too reserved ever to be truly effusive." The first missing term is
similar in meaning to "warm," which, in this instance, means affectionate. The
second term should have a meaning that is the opposite of "reserved," which
means restrained or shy. "Cordial" means socially warm and "effusive" is the
opposite of reserved.

Explanation for Incorrect Answer A :


Choice (A) is incorrect. "Joyous" means happy. "Conventional" means lacking
originality or individuality. If one were to insert these terms into the text, the
sentence would read "Although usually warm and joyous in greeting friends, Lauren
was too reserved ever to be truly conventional." The first missing term is similar in

file://E:\\c6.htm 2006-11-12
The Official SAT Online Course 6/22

meaning to "warm," which, in this instance, means affectionate. The second word
should have a meaning that is the opposite of "reserved," which means restrained
or shy. "Conventional" is not the opposite of reserved.

Explanation for Incorrect Answer C :


Choice (C) is incorrect. "Restrained" means kept under control. "Gracious" means
friendly. If one were to insert these terms into the text, the sentence would read
"Although usually warm and restrained in greeting friends, Lauren was too reserved
ever to be truly gracious." The first missing term is similar in meaning to "warm,"
which, in this instance, means affectionate. The second word should have a
meaning that is the opposite of "reserved," which means restrained or shy. The
word "restrained" does not suggest affection.

Explanation for Incorrect Answer D :


Choice (D) is incorrect. "Dismissive" describes an attitude of offhand rejection.
"Ebullient" means full of high spirits. If one were to insert these terms into the text,
the sentence would read "Although usually warm and dismissive in greeting friends,
Lauren was too reserved ever to be truly ebullient." The first missing term is similar
in meaning to "warm," which, in this instance, means affectionate. The second word
should have a meaning that is the opposite of "reserved," which means restrained
or shy. Lauren would not be both warm and dismissive when she greets friends,
since the these are two opposite attitudes.

Explanation for Incorrect Answer E :


Choice (E) is incorrect. "Genial" means cheerful. "Antisocial" means unsociable. If


one were to insert these terms into the text, the sentence would read "Although
usually warm and cordial in greeting friends, Lauren was too reserved ever to be
truly effusive." The first missing term is similar in meaning to "warm," which, in this
instance, means affectionate. The second word should have a meaning that is the


d
opposite of "reserved," which means restrained or shy. One would not say that a
person is too reserved to be antisocial. An extremely reserved person might be

re
considered antisocial.



te
7 Legal scholars argue that when justice is interpreted too broadly, the concept
becomes -------, easily changed and controlled by outside forces.


is
(A) malleable
(B) influential


eg

(C) coherent

(D) felicitous


nR

(E) prosaic


ANSWERS AND EXPLANATIONS

Explanation for Correct Answer A :


Choice (A) is correct. "Malleable" means capable of being changed or controlled. If
one were to insert this term into the text, the sentence would read "Legal scholars
argue that when 'justice' is interpreted too broadly, the concept becomes malleable,
easily changed and controlled by outside forces." "Malleable" properly describes a
"justice" that can be altered by outside forces.

Explanation for Incorrect Answer B :


Choice (B) is incorrect. "Influential" means powerful or having an effect on others.
If one were to insert this term into the text, the sentence would read "Legal
scholars argue that when 'justice' is interpreted too broadly, the concept becomes
influential, easily changed and controlled by outside forces." An "influential" concept
is one that changes outside forces, not one that is changed by outside forces.

Explanation for Incorrect Answer C :


Choice (C) is incorrect. "Coherent" means well ordered or clear. If one were to
insert this term into the text, the sentence would read "Legal scholars argue that
when 'justice' is interpreted too broadly, the concept becomes coherent, easily
changed and controlled by outside forces." It makes no sense to imply that a
concept can be easily changed or controlled because it is clear.

Explanation for Incorrect Answer D :


Choice (D) is incorrect. "Felicitous" means well suited or particularly appropriate to
a situation. If one were to insert this term into the text, the sentence would read

file://E:\\c6.htm 2006-11-12
The Official SAT Online Course 7/22

"Legal scholars argue that when 'justice' is interpreted too broadly, the concept
becomes felicitous, easily changed and controlled by outside forces." If something is
well suited, it makes no sense to describe it as easily changed or controlled.

Explanation for Incorrect Answer E :


Choice (E) is incorrect. "Prosaic" means ordinary. If one were to insert this term into
the text, the sentence would read "Legal scholars argue that when 'justice' is
interpreted too broadly, the concept becomes prosaic, easily changed and controlled
by outside forces." An ordinary concept might be a concept that is easily changed or
controlled, but it is not necessarily so.

8 The instructors voice was so ------- that most students preferred taking a test over
listening to its grating sound.
(A) receptive
(B) cajoling
(C) melodious

(D) muted

(E) strident

ANSWERS AND EXPLANATIONS




Explanation for Correct Answer E :

d
Choice (E) is correct. "Strident" means loud, harsh and discordant. If one were to
insert this term into the text, the sentence would read "The instructor's voice was

re
so strident that most students preferred taking a test over listening to its grating
sound." The missing term describes a voice that has a "grating sound." A strident
sound is synonymous with a grating sound.


e
st


Explanation for Incorrect Answer A :
i

Choice (A) is incorrect. "Receptive" means open and responsive to ideas or


eg

suggestions. If one were to insert this term into the text, the sentence would read
"The instructor's voice was so receptive that most students preferred taking a test
over listening to its grating sound." The missing term describes a voice that has a


"grating sound." A grating sound has nothing to do with being receptive.
Furthermore, no voice could be described as receptive.
nR


Explanation for Incorrect Answer B :
Choice (B) is incorrect. "Cajoling" means persuading or coaxing. If one were to
insert this term into the text, the sentence would read "The instructor's voice was


so cajoling that most students preferred taking a test over listening to its grating
U

sound." The missing term describes a voice that has a "grating sound." A cajoling
sound is not synonymous with a grating sound. A person who is cajoling would
most likely use a soft voice, not a grating one.

Explanation for Incorrect Answer C :


Choice (C) is incorrect. "Melodious" means sweet-sounding or full or melody. If one
were to insert this term into the text, the sentence would read "The instructor's
voice was so melodious that most students preferred taking a test over listening to
its grating sound." The missing term describes a voice that has a "grating sound." A
melodious voice is the opposite of a grating one.

Explanation for Incorrect Answer D :


Choice (D) is incorrect. "Muted" means muffled or softened. If one were to insert
this term into the text, the sentence would read "The instructor's voice was so
muted that most students preferred taking a test over listening to its grating
sound." The missing term describes a voice that has a "grating sound." A muted
voice is the opposite of a grating voice.

The big doors of the hotel are padlocked. So


far nobody

has smashed their glass panels. Nobody could

file://E:\\c6.htm 2006-11-12
The Official SAT Online Course 8/22

stand to do
it because the panels mirror your own face as
well as the
view behind your back: acres of chive grass
Line
edging the
sparkly beach, a movie-screen sky, and an
5
ocean that wants
you more than anything. No matter the outside
loneliness,
if you look inside, the hotel seems to promise
you ecstasy
and the company of all your best friends. And
music. The
shift of a shutter hinge sounds like the cough of
a trumpet;
piano keys waver a quarter note above the
10
wind so you
might miss the hurt jamming those halls and
closed-up
rooms.

ed
The passage is characterized by all of the following EXCEPT

(A) visual imagery


r

(B) auditory descriptions
te
(C) contrast


(D) an appeal to reason
is

(E) hypothetical musings


eg

ANSWERS AND EXPLANATIONS


Explanation for Correct Answer D :
nR

Choice (D) is correct. The passage does not appeal to reason but to emotion and to


the senses. It describes the feelings and impressions associated with the physical
appearance of the hotel.

Explanation for Incorrect Answer A :


Choice (A) is incorrect. The passage does use visual imagery, such as the
descriptions of the padlocked doors and the reflections in their glass panels.

Explanation for Incorrect Answer B :


Choice (B) is incorrect. The sounds of the shutter hinge and the wind are auditory
descriptions, or descriptions of sound, that are compared to music.

Explanation for Incorrect Answer C :


Choice (C) is incorrect. The lonely, vast land outside the hotel is contrasted with the
happy crowd of "best friends" that the hotel "seems to promise you."

Explanation for Incorrect Answer E :


Choice (E) is incorrect. Through hypothetical musings, or imaginative reflections,
the author describes the empty, padlocked hotel as if it were the scene of a party.

10 Lines 10-12 (piano keys . . . rooms) convey a feeling of

(A) lightheartedness
(B) bewilderment

file://E:\\c6.htm 2006-11-12
The Official SAT Online Course 9/22

(C) melancholy

(D) nostalgia

(E) detachment

ANSWERS AND EXPLANATIONS


Explanation for Correct Answer C :
Choice (C) is correct. These lines convey a feeling of melancholy, or sadness,
because of the "hurt" that haunts the imaginary cheerfulness of the inside of the
hotel.

Explanation for Incorrect Answer A :


Choice (A) is incorrect. Earlier in the passage, the hotel's inside was imagined as
lighthearted and happy, but this is contradicted by the "hurt jamming," or pain
pressing, inside the "halls" and "closed-up rooms" of the hotel.

Explanation for Incorrect Answer B :


Choice (B) is incorrect. Bewilderment, or confusion, is not conveyed in the


description of the hotel's sadness. Rather, there is a degree of clarity expressed in
the recognition of the sadness that fills the hotel.


Explanation for Incorrect Answer D :

d
Choice (D) is incorrect. Even though nostalgia, or a longing for the past, is present
elsewhere in the passage, there is no longing for the hotel's "hurt" and sadness
expressed in lines 10-12.

re
Explanation for Incorrect Answer E :


Choice (E) is incorrect. There is no feeling of detachment, or lack of involvement, in
te
the emotionally charged description of the "hurt" the author imagines "jamming
those halls and closed-up rooms."


is


eg

Diffusion theory is an umbrella idea


encompassing


various alternative theories of Americas
nR

discovery by


explorers from other parts of the world.
Columbus (and

Leif Ericsson and Zheng He) had a lot more


Line
competitors
than most people think: Prince Madoc of Wales,
5
the Zeni
brothers of Venice, Corte Real of Portugal,
Polands Jan
of Kolno. The fact is, crossing the Atlantic was
probably
not as big a deal as Columbus-centric historians
thought.
Diffusionists may not be able to pinpoint who
beat
Columbus to the punch, yet theyre sure
10
someone did.
They may well be right, but if you scrutinize
any specific
claim, it melts away. This is probably why
diffusionists

file://E:\\c6.htm 2006-11-12
The Official SAT Online Course 10/22

emphasize quantity over quality.


11 In line 1, umbrella is used to convey which of the following qualities?
(A) Comprehensiveness
(B) Impenetrability
(C) Utility

(D) Ordinariness

(E) Foresight

ANSWERS AND EXPLANATIONS


Explanation for Correct Answer A :
Choice (A) is correct. "Comprehensiveness" is the quality of being all-inclusive and
wide-ranging, covering many ideasjust as an umbrella might cover, or stretch,
over several objects. In this context, "umbrella" refers to "diffusion theory," which
covers or includes within itself "various alternative theories" (line 2) about the
discovery of America.

Explanation for Incorrect Answer B :




Choice (B) is incorrect. "Impenetrable" refers to something that cannot be

d
penetrated. It makes sense that an umbrella would be described as "impenetrable"
by rain, but the theories mentioned in the passage have been penetrated, or shown

re
to be flawed, be researchers: "if you scrutinize any specific claim, it melts


away" (lines 11-12). Thus they cannot be described as "impenetrable."

Explanation for Incorrect Answer C :


te
Choice (C) is incorrect. The utility, or usefulness, of an umbrella is not the quality
featured in the passage. The passage uses the term "umbrella" to refer to the fact
that the "diffusion theory" comprises "various alternative theories" (line 2).


is

Explanation for Incorrect Answer D :


Choice (D) is incorrect. Even though an umbrella is an ordinary object, the passage


eg

does not refer to the ordinariness of the "diffusion theory," but to its
comprehensiveness.


Explanation for Incorrect Answer E :
nR

Choice (E) is incorrect. The passage explains that "diffusion theory" includes many
alternative theories. The notion of foresight is not discussed in the passage.

12
The strategy employed by the diffusionists in making their claim is most evident in
which assertion?

(A) Since nothing can be proven with absolute certainty, we ultimately rely on
faith.
(B) Before the Wright brothers success, experts thought that craft that were
heavier than air could not fly.
(C) So many UFO sightings have been reported that at least one of them must
be authentic.
(D) Penicillin, like many other discoveries, was stumbled on by accident.

(E) Although folk medicine was at first derided by the medical establishment,
people still relied on it.

ANSWERS AND EXPLANATIONS


Explanation for Correct Answer C :
Choice (C) is correct. This assertion uses the great quantity of UFO sightings as
evidence that at least one of them must be authentic. The quantity of the sightings,
or the number of times people claim to see UFOs, is emphasized over the quality of
the sightings (a high-quality sighting is one that could be verified by independent
experts). This is the same strategy that diffusionists use; diffusionists also
"emphasize quantity over quality" (line 13).

file://E:\\c6.htm 2006-11-12
The Official SAT Online Course 11/22

Explanation for Incorrect Answer A :


Choice (A) is incorrect. Although diffusionists cannot at this point say which
explorer beat Columbus, diffusionists do not conclude that proof is impossible or
irrelevant.

Explanation for Incorrect Answer B :


Choice (B) is incorrect. The Wright Brothers decisively disproved a theory, whereas
diffusion theory has to do with ideas that have not been either proven or disproven.

Explanation for Incorrect Answer D :


Choice (D) is incorrect. Diffusionists may embrace theories that claim America was
discovered by accident, but this is not mentioned in the passage.

Explanation for Incorrect Answer E :


Choice (E) is incorrect. The passage does not describe diffusionists as being
concerned with rivalries between new technologies and old ones.

Is a persons gender an important influence on how he or she behaves with others?


Contemporary sociologists and other scholars have argued this question fiercely. The
following pair of passages presents two contrasting voices from that debate.

Passage 1


The desire to affirm that women and men

ed
are com-


pletely equal has made some scholars reluctant
to show
er
ways in which they are different, because
differences

t
between two groups of people have so often


is
Line
been used
to justify unequal treatment and


eg

5
opportunity. Much
as I understand and am in sympathy with those
who

nR

wish there were no differences between women


and

menonly reparable social injusticemy

research
on styles of conversation tells me that, at least
in this
area, it simply isnt so. I believe that there are
10
gender
differences in ways of speaking, and we need to
identify
and understand them. Without such
understanding, we
are doomed to blame others or ourselvesor
our own
relationshipsfor the otherwise mystifying and
damag-
ing effects of our contrasting conversational
15
styles.
It is clear to me that recognizing gender
differences
in conversational styles would free individuals

file://E:\\c6.htm 2006-11-12
The Official SAT Online Course 12/22

from
the burden of an inappropriate sense of being
at fault
for chronic disagreements. Many women and
men feel
dissatisfied with their close relationshipswith
20
spouses,
siblings, parentsand become even more
frustrated
when they try to talk things out. Taking a
sociolinguistic
approach to such troubling encounters makes it
possible
to explain these dissatisfactions without
accusing anyone
of being wrong and without blamingor
25
discarding
the relationship.

The sociolinguistic approach I take in my
work is based

ed
on my belief that many frictions arise because,
here in the

United States, boys and girls grow up in what
er
are essen-

tially different cultures, so that talk between
t
30
women and

is

men is actually cross-cultural communication.


For little
eg

boys, talk is primarily a means of making


statements of
achievement through games like bragging
nR


contests. This
may also be done by exhibiting knowledge or
skill and by

holding center stage through such verbal


35
performance as
storytelling, joking, or imparting information.
Little girls
appear to be eager to share and compare
interests and ideas.
Emphasis is placed on displaying similarities
and match-
ing experiences. For them, the language of
conversation
is primarily a language of rapport: a way of
40
establishing
connection and negotiating relationships. So
this view of
childrens behavior predicts that more women
than men
will be comfortable speaking one-on-one, to
individuals.

file://E:\\c6.htm 2006-11-12
The Official SAT Online Course 13/22

And even when addressing an audience, women


may be
more concerned than men with establishing
45
rapport.
Passage 2
Gender stereotypes should concern us for
several
reasons. First, they may dictate what we notice
and bias
our perceptions in the direction of expectation.
Some
researchers attempt to elucidate gender
differences in
order to help women and men understand and
50
respond
to one another better. In the process, however,
their work

encourages people to notice and attend to
differences rather

ed
than similarities, to perceive men and women in
accordance

with stereotypes that may not accurately depict
r

te
their behav-
ior or intentions. Second, gender stereotypes
55


may not only
is

describe behavior but also prescribe it, dictating


how men
eg

and women should behave. People begin to


act in ways
that support other peoples gender-role
nR


expectations of
them.


It is time to rethink our understanding of
U

60
gender, to
move away from the notion that men and
women have
two contrasting styles of interaction that were
acquired
in childhood. We need to move from a
conceptualization
of gender as an attribute or style of behavior to
an under-
standing of gender as something people do in
65
social
interaction. As a noted scholar proposes,
None of us is
feminine or is masculine or fails to be either of
those. In
particular contexts people do feminine, in
others, they do
masculine. People display contradictory

file://E:\\c6.htm 2006-11-12
The Official SAT Online Course 14/22

behaviors as they
70 encounter different social norms and pressures.
Some researchers view male-female
conversations as
cross-cultural communication. The two-cultures
approach
postulates that difficulties in communication
between men
and women arise because of a clash of
conversational
styles. But this approach has a number of
75
limitations. First,
the coherence of male and female subcultures
in childhood
has been exaggerated. We arrive at a
contrasting picture
of the cultures of boys and girls only by singling
out those
children who fit common gender stereotypes
and margin-

d
alizing others. We fail to notice the children who
80
do not fit
re
those stereotypesfor example, boys who excel


at caring
te
for younger siblings or girls who enjoy building


things in
is

shop class. Second, although children may


choose same-


eg

sex playmates as preferred partners, they


interact daily


inside and outside school with the opposite sex.
nR

85
Children


have countless experiences communicating with
people

of both sexes: they do not learn to


communicate in gender-
segregated worlds. They learn to display
different styles of
interaction in different contexts: they do not
learn a single
gender-related style. The same child may
90
display domi-
nance and give orders to a younger playmate
but show
deference and follow orders from an older
friend.
13 The two passages differ most on which topic?

(A) Whether boys and girls communicate in gender specific patterns


(B) Whether important social behavior is learned in childhood
(C) Whether adult conversational styles can be studied systematically

(D) Whether gender plays a role in determining a childs playtime activities

file://E:\\c6.htm 2006-11-12
The Official SAT Online Course 15/22

(E) Whether society concerns itself with the concept of gender roles

ANSWERS AND EXPLANATIONS


Explanation for Correct Answer A :
Choice (A) is correct. The passages differ substantially on the question of whether
gender determines how individuals communicate with each other. In Passage 1, the
author states, "I believe that there are gender differences in ways of speaking, and
we need to identify and understand them" (lines 10-12). The author of Passage 2
argues that "this approach has a number of limitations" (line 75), and stresses that
individuals need to move toward "an understanding of gender as something people
do in social interaction" (lines 64-66). Passage 2 sees gender as a fluid component
of social interactions that anyone can adopt depending on the situation, whereas
Passage 1 sees gender as a fixed determinant of communication styles.

Explanation for Incorrect Answer B :


Choice (B) is incorrect. Both passages give examples of social behaviors in
childhood, and both assume that these behaviors are at least partly learned.

Explanation for Incorrect Answer C :


Choice (C) is incorrect. Passage 1 implies that a study of conversational styles
between adults is possible and desirable, but Passage 2 does not address this topic.


ed
Explanation for Incorrect Answer D :
Choice (D) is incorrect. Passage 1 discusses the role of gender in shaping how
children approach activities such as conversation; Passage 2 does not deny that


gender is a factor in children's lives, but suggests that the situation is more
complex than sociolinguists claim. The passages do not primarily address children's
playtime activities. r

te
Explanation for Incorrect Answer E :
Choice (E) is incorrect. The authors both agree that society is very interested in


is
gender roles.


eg

14 The primary purpose of Passage 1 is to

(A)


present a historical overview of a controversy
nR

(B) acknowledge previous errors in thinking


(C) urge changes in organized activities provided for children

(D) assert the value of a particular approach to an issue

(E) downplay the significance of a recent discovery

ANSWERS AND EXPLANATIONS


Explanation for Correct Answer D :
Choice (D) is correct. Passage 1's argument is that gender-specific communication
differences account for misunderstandings between men and women. The author
asserts the value of the sociolinguistic approach to the issue of gender differences,
arguing that it is an approach that should improve understanding between men and
women.

Explanation for Incorrect Answer A :


Choice (A) is incorrect. Passage 1 does not present the history of the controversy
over gender differences, but rather explains how a sociolinguistic approach to the
problem will help people deal with these differences.

Explanation for Incorrect Answer B :


Choice (B) is incorrect. Passage 1 does acknowledge that some scholars are
"reluctant," or unwilling, to show ways in which men and women are different, but
the passage is not primarily concerned with the errors of others.

Explanation for Incorrect Answer C :

file://E:\\c6.htm 2006-11-12
The Official SAT Online Course 16/22

Choice (C) is incorrect. Passage 1 does not suggest changes in children's activities,
but rather describes how the different cultures of boys and girls lead them to
communicate differently.

Explanation for Incorrect Answer E :


Choice (E) is incorrect. Passage 1 does not discuss any recent discovery.

15 Passage 1 makes which suggestion about the work of some scholars (line 2)?
(A) It will become widely accepted in the scientific community.
(B) It is well intentioned but misguided.
(C) It attempts to be objective but does not succeed.

(D) It puts forth a convincing theory.

(E) It could be used to excuse injustice in a society.

ANSWERS AND EXPLANATIONS


Explanation for Correct Answer B :
Choice (B) is correct. Passage 1 explains that some scholars have been "reluctant


to show ways in which [men and women] are different, because differences
between two groups have so often been used to 'justify' unequal treatment" (lines
2-5). The author of Passage 1 appreciates the good intentions of those scholars,


but ultimately disagrees with them. The author clearly believes that acknowledging

d
gender differences will help people relate to members of the opposite sex.

re
te

Explanation for Incorrect Answer A :
Choice (A) is incorrect. Far from predicting that the scholars' work will eventually be
accepted, Passage 1 discounts the belief that there are no differences between


is
women and men by saying, "it simply isn't so" (line 10).

Explanation for Incorrect Answer C :


eg

Choice (C) is incorrect. Passage 1 does not suggest that the scholars are objective,
or unbiased, about gender differences; rather, it implies that their "desire to affirm
that women and men are completely equal" (lines 1-2) has affected their


observations of people's actual behavior.
nR

Explanation for Incorrect Answer D :


Choice (D) is incorrect. The author of Passage 1 firmly disagrees with the theory
that there is no difference between women and men.

Explanation for Incorrect Answer E :


Choice (E) is incorrect. Passage 1 suggests the opposite: that the scholars' work
attempts to cure social injustice by denying differences between men and women.
There is no sense that the scholars' work might be used to excuse injustice.

16 Passage 1 argues that recognizing gender differences (line 16) would most likely

(A) cause people to exaggerate their similarities when communicating with


one another
(B) lead to further dissatisfaction in conversations among friends and relatives
(C) promote the equal treatment of distinct social groups

(D) relieve individuals of much of the blame for problems in relationships

(E) affect the way that future research on gender is conducted

ANSWERS AND EXPLANATIONS


Explanation for Correct Answer D :
Choice (D) is correct. Passage 1's approach to gender difference is concerned with
men's and women's close relationships with spouses, siblings and friends. According
to the passage, "recognizing gender differences in conversational styles would free
individuals from the burden of an inappropriate sense of being at fault" (lines 16-
18). In other words, this recognition would keep people from blaming themselves

file://E:\\c6.htm 2006-11-12
The Official SAT Online Course 17/22

(or others) for gender-based misunderstandings.

Explanation for Incorrect Answer A :


Choice (A) is incorrect. Recognizing gender differences would not cause people to
exaggerate their similarities. It would make men and women aware of their
differences, which would lead to improved understanding between people.

Explanation for Incorrect Answer B :


Choice (B) is incorrect. According to Passage 1, recognizing gender differences
would decrease dissatisfaction, not increase it.

Explanation for Incorrect Answer C :


Choice (C) is incorrect. While the passage does not seek to justify unequal
treatment and opportunity for men and women, neither is it concerned with
promoting equal treatment. Rather, it argues for understanding gender-specific
communication patterns in order to improve relationships.

Explanation for Incorrect Answer E :


Choice (E) is incorrect. The passage is not concerned with how research on gender
should be conducted, but with improving the way men and women relate to each
other in conversation.



ed
17 In lines 36-41 (Little girls . . . relationships), the author of Passage 1 assumes
that for girls, a primary function of communication is to
(A) foster a sense of intimacy between speaker and listener
(B)

establish a set of conversational rules shared by speaker and listener
er

(C) convey information previously unknown by the listener

(D) promote nostalgic feelings about past friendships


t

is
(E) create an objective atmosphere for personal discussions


eg

ANSWERS AND EXPLANATIONS


Explanation for Correct Answer A :


Choice (A) is correct. Passage 1 describes how girls seek to foster intimacy, or
nR

closeness, with others through communication. Little girls are said to emphasize
"displaying similarities and matching experiences" (lines 38-39) in their


conversations. The passage points out that for girls, the purpose of language is
"establishing connection and negotiating relationships" (lines 40-41).

Explanation for Incorrect Answer B :


Choice (B) is incorrect. The passage does not mention the idea that girls try to
establish conversational rules; instead, it argues that they use communication as a
way of establishing relationships through sharing interests and ideas.

Explanation for Incorrect Answer C :


Choice (C) is incorrect. Lines 36-41 state that instead of seeking to impart
information, girls focus on "displaying similarities and matching experiences" (lines
38-39) in their conversations.

Explanation for Incorrect Answer D :


Choice (D) is incorrect. Nostalgic feelings, or feelings of longing for the past, are
not mentioned in the description of girls' communication patterns.

Explanation for Incorrect Answer E :


Choice (E) is incorrect. According to the passage, girls are interested in establishing
rapport, or creating agreement. The passage does not describe either girls or boys
as seeking to be objective, or unbiased, in their conversations.

18
The author of Passage 2 would most likely challenge the claim made in lines 27-31 of
Passage 1 (The sociolinguistic . . . communication) by arguing that

file://E:\\c6.htm 2006-11-12
The Official SAT Online Course 18/22

(A) children do not grow up in single-gender cultures


(B) children may become skilled at deceiving adults
(C) gender differences are impossible to assess scientifically

(D) there is less conflict between men and women than sociologists assume

(E) childrens behaviors have changed dramatically in recent years

ANSWERS AND EXPLANATIONS


Explanation for Correct Answer A :
Choice (A) is correct. The claim made in lines 27-31 of Passage 1 is that "boys and
girls grow up in what are essentially different cultures," so any challenge must
address that claim directly. The author of Passage 2 states that children have
"countless experiences communicating with people of both sexes" (line 86). The
author of Passage 2 would thus argue that children do not grow up in a single-
gender culture.

Explanation for Incorrect Answer B :


Choice (B) is incorrect. The author of Passage 2 does not discount sociolinguistic
research by suggesting that children deceive adults, but by suggesting that
researchers deceive themselves.


ed
Explanation for Incorrect Answer C :
Choice (C) is incorrect. Passage 2 argues that gender differences have not been


studied scientifically, or objectively, by researchers. However, the author of
Passage 2 does not claim that this task is impossible.
er

Explanation for Incorrect Answer D :
Choice (D) is incorrect. The claim made in lines 27-31 of Passage 1 is that "boys
and girls grow up in what are essentially different cultures." The author of Passage
t

2 would not be likely to address this claim by arguing that men and women
is
experience less conflict than some scholars assume.


eg

Explanation for Incorrect Answer E :


Choice (E) is incorrect. There is no mention in either passage of how the behavior
of children has changed in recent years. Neither passage is concerned with changes
over time.


nR

19

The sentence in lines 47-48 in Passage 2 (First . . . expectation) primarily

emphasizes which damaging effect of gender stereotypes?

(A) They may offend the person being stereotyped.


(B) They may distort our observations of people we meet.
(C) They have been used to justify gender inequality.

(D) They commonly cause miscommunication between men and women.

(E) They reflect negatively on those who believe in them.

ANSWERS AND EXPLANATIONS


Explanation for Correct Answer B :
Choice (B) is correct. According to the author, stereotypes "may dictate what we
notice and bias our perceptions in the direction of expectation" (lines 47-48). In
other words, when we meet people, gender stereotypes may cause us to look for
the qualities we expect those people to have, and to focus on those qualities
instead of others. These stereotypes make it difficult to observe people with a fair,
unbiased eye; they distort, or skew, our view of people.

Explanation for Incorrect Answer A :


Choice (A) is incorrect. The passage does not mention the feelings of people being
stereotyped.

file://E:\\c6.htm 2006-11-12
The Official SAT Online Course 19/22

Explanation for Incorrect Answer C :


Choice (C) is incorrect. Passage 2 does not mention the use of stereotypes to justify
unequal treatment of men and women.

Explanation for Incorrect Answer D :


Choice (D) is incorrect. The author of Passage 2 may believe that miscommunication
between men and women can be caused by stereotyping; however, lines 47-48
refer to the ways gender stereotypes distort people's views of one another.

Explanation for Incorrect Answer E :


Choice (E) is incorrect. Passage 2 states that gender stereotypes influence our
perceptions about others. It does not discuss what these stereotypes reveal about
those who believe in them.

20 The assumptions underlying the research work described in lines 48-51 of Passage 2
are most similar to the assumptions held by the
(A) scholars in line 2
(B) women and men in line 19
(C) noted scholar in line 66


(D) author of Passage 1

(E) author of Passage 2


ed
ANSWERS AND EXPLANATIONS
Explanation for Correct Answer D :


er
Choice (D) is correct. The assumptions of the researchers mentioned in lines 48-51


are very similar to those of the author of Passage 1. Both want "to elucidate gender
differences in order to help women and men understand and respond to each other
t
better." As the author of Passage 1 states, "it is clear to me that recognizing gender


differences in conversational styles would free individuals from the burden of an
is
inappropriate sense of being at fault" (lines 16-18).


eg


Explanation for Incorrect Answer A :
nR

Choice (A) is incorrect. The "scholars" in line 2 are reluctant, or unwilling, to show
ways in which men and women are different. The research work described in lines


48-51 assumes that there are gender differences and asks that they be
acknowledged.

Explanation for Incorrect Answer B :


Choice (B) is incorrect. The dissatisfied women and men mentioned in line 19 most
likely do not assume that gender differences account for their misunderstandings.
The "researchers" in Passage 2 are trying to help such people.

Explanation for Incorrect Answer C :


Choice (C) is incorrect. The "noted scholar" mentioned in line 66 does not assume
that gender differences determine the way a person acts. This scholar's idea is very
different from the claim of the researchers mentioned in lines 48-51: that
acknowledging gender differences will help men and women communicate.

Explanation for Incorrect Answer E :


Choice (E) is incorrect. The author of Passage 2 does not assume that
acknowledging gender differences will help men and women communicate. This
author is concerned about the harm done by gender stereotypes.

21 The quotation in lines 66-69 (None . . . masculine) primarily serves to

(A) introduce a personal experience


(B) provide a typical example
(C) elaborate on an idea

(D) signal a change in topic

file://E:\\c6.htm 2006-11-12
The Official SAT Online Course 20/22

(E) offer recent research data

ANSWERS AND EXPLANATIONS


Explanation for Correct Answer C :
Choice (C) is correct. The sentence preceding the quote argues that we need to
think of gender "as something people do in social interaction" (lines 65-66). The
quotation, by pointing out that "in particular contexts people do feminine, in others,
they do masculine," elaborates on the idea that people's behavior is determined by
situation rather than by gender.

Explanation for Incorrect Answer A :


Choice (A) is incorrect. The quotation is not about someone's personal experience;
it is a statement about people's experiences in general.

Explanation for Incorrect Answer B :


Choice (B) is incorrect. The quotation does not provide any kind of example; it is a
generalization, or broad idea.


Explanation for Incorrect Answer D :
Choice (D) is incorrect. Instead of signaling a new topic, this quotation extends the
topic: the idea that gender is "something people do" that varies with the situation.


ed
Explanation for Incorrect Answer E :
Choice (E) is incorrect. Even though this is a quotation by a "noted scholar," it does
not mention any recent research data.


er
22

st
Passage 2 suggests that some scholars construct a contrasting picture of the


cultures of boys and girls (lines 77-78) by studying children whose

(A) readiness to interact with strangers is apparent


i


eg

(B) demand for approval from adults is particularly strong


(C) rebellion against authority results in creative behavior


(D) personalities are highly idiosyncratic
nR

(E) actions correspond to a narrow preconception of behavior


ANSWERS AND EXPLANATIONS

Explanation for Correct Answer E :


Choice (E) is correct. According to Passage 2, in the attempt to highlight the
differences between boys and girls, some scholars are "singling out those children
who fit common gender stereotypes and marginalizing others" (lines 78-80). By
studying children who tend to behave according to stereotypes, or "narrow
preconceptions of behavior," those scholars can construct a picture that highlights
the differences in the cultures of boys and girls.

Explanation for Incorrect Answer A :


Choice (A) is incorrect. Passage 2 does not say anything about children who are
eager to interact with strangers.

Explanation for Incorrect Answer B :


Choice (B) is incorrect. Passage 2 does not suggest that scholars study children
who are in strong need of approval from adults.

Explanation for Incorrect Answer C :


Choice (C) is incorrect. The scholars discussed in Passage 2 study children's
behavior and attitudes toward each other, not toward adults or figures of authority.
Furthermore, there is no mention of rebellion or creative behavior in Passage 2.

Explanation for Incorrect Answer D :


Choice (D) is incorrect. Passage 2 does not suggest that children in the studies

file://E:\\c6.htm 2006-11-12
The Official SAT Online Course 21/22

have highly idiosyncratic personalities, or that they have very particular individual
characteristics. On the contrary, the author states that the studies focus on children
who fit common gender stereotypes while ignoring children who do not act in
expected ways.

23 The author of Passage 2 implies that the boys mentioned in line 81 and the
child mentioned in line 90 resemble one another in that they
(A) have not learned to imitate adult behaviors
(B) refuse to get along with their peers
(C) do not conform to traditional gender stereotypes

(D) openly mock adult expectations about their behavior

(E) communicate primarily with children of their own gender

ANSWERS AND EXPLANATIONS


Explanation for Correct Answer C :
Choice (C) is correct. The author cites these children as examples of children who
do not "conform to," or fit in with, traditional gender stereotypes. The "boys who


excel at caring for younger siblings" are demonstrating a quality more associated
with girls, the author implies. Likewise, the child who displays dominance with a
younger playmate and shows deference to an older friend is behaving according to


the situation, not according to gender.

d
Explanation for Incorrect Answer A : re
te

Choice (A) is incorrect. Caring for younger siblings and displaying dominance or
deference might be adult behaviors that these children are imitating.


is
Explanation for Incorrect Answer B :
Choice (B) is incorrect. The passage doesn't say whether these children get along
with their peers. The author's main point is that they do not act according to gender


eg

stereotypes.

Explanation for Incorrect Answer D :


Choice (D) is incorrect. There is nothing to suggest that these children "mock," or
nR

make fun of, adults' expectations of them, or that they are paying attention to
adults at all.


Explanation for Incorrect Answer E :
Choice (E) is incorrect. The passage suggests that these children, like most

children, actually "have countless experiences communicating with people of both


sexes" (lines 86-87).

24 Which of the following best characterizes the ideas about gender communication
styles as they are presented in the two passages?

(A) Passage 1 argues that styles are based on competition, while Passage 2
suggests that they are a form of cooperation.
(B) Passage 1 argues that styles are a burden, while Passage 2 implies that
they can help facilitate relationships between men and women.
(C) Passage 1 claims that styles are semantic, while Passage 2 suggests that
they are whimsical.
(D) Passage 1 suggests that styles are constant, while Passage 2 argues that
they are fluid.
(E) Passage 1 states that styles are random, while Passage 2 indicates that
their patterns become obvious upon closer scrutiny.

ANSWERS AND EXPLANATIONS


Explanation for Correct Answer D :
Choice (D) is correct. According to Passage 1, communication styles are determined
by gender and are thus constant and predictable. The author states: "I believe that
there are gender differences in ways of speaking, and we need to identify and
understand them" (lines 10-12). Passage 2, on the other hand, states that gender

file://E:\\c6.htm 2006-11-12
The Official SAT Online Course 22/22

communication styles are fluid and change according to the individual and the
situation: "people display contradictory behaviors as they encounter different social
norms and pressures" (lines 69-70).

Explanation for Incorrect Answer A :


Choice (A) is incorrect. In Passage 1, the author mentions that boys tend to make
competitive statements of achievement. However, Passage 1 does not argue that all
styles are based on competition, and Passage 2 does not suggest that all
conversational styles are a form of cooperation.

Explanation for Incorrect Answer B :


Choice (B) is incorrect. Passage 1 does not argue that styles are a burden, but
rather that understanding them can help improve understanding between men and
women. Passage 2 argues against the existence of gender specific styles altogether;
it points out that thinking in terms of "differences rather than similarities" is not
helpful.

Explanation for Incorrect Answer C :


Choice (C) is incorrect. Passage 1 does claim that styles are semantic, in that they
have to do with meaning in language. However, Passage 2 argues that people act
and speak according to what they have learned to do in different situations. This
has nothing to with conversational styles being whimsical.

Explanation for Incorrect Answer E :



Choice (E) is incorrect. Passage 1 does not state that conversational styles are


d
random, or a result of chance, but that they are determined by gender.
Furthermore, Passage 2 argues that closer scrutiny will show that styles of behavior
in general are less dependent on gender than is commonly supposed.

re

te

is

Back to Score Report

Copyright 2006 The College Board. All rights reserved. Privacy Policy Terms of Use Contact Us


eg


nR

file://E:\\c6.htm 2006-11-12
The Official SAT Online Course 1/16

Help | Profile | My Organizer | My Bookmarks | Logout

Answers and Explanations

Test Sections Back to Score Report

Section 1 View Answers and Explanations


Section 2 Online - Practice Test #3

Section 3
1 If then
Section 4
Section 5 (A)
Section 7 (B)
Section 8
Section 9 (C)
Section 10
(D)


(E)

ANSWERS AND EXPLANATIONS


ed
Explanation for Correct Answer B :


Choice (B) is correct. If er then and


st

Explanation for Incorrect Answer A :


eg

Choice (A) is not correct. If were then would be equal to


instead of the given value of


Explanation for Incorrect Answer C :
nR


Choice (C) is not correct. If were then would be equal to

instead of the given value of

Explanation for Incorrect Answer D :

Choice (D) is not correct. If were then would be equal to

instead of the given value of

Explanation for Incorrect Answer E :


Choice (E) is not correct. If were then would be equal to
instead of the given value of

In the triangle above,

file://E:\\c7.htm 2006-11-12
The Official SAT Online Course 2/16

(A)

(B)

(C)

(D)

(E)

ANSWERS AND EXPLANATIONS


Explanation for Correct Answer D :
Choice (D) is correct. The sum of the interior angles of a triangle is So
Solving for gives

Explanation for Incorrect Answer A :


Choice (A) is not correct. If were then the sum of the angles in the given
triangle would be instead of


Explanation for Incorrect Answer B :
Choice (B) is not correct. If were then the sum of the angles in the given
triangle would be instead of


d
Explanation for Incorrect Answer C :

re
Choice (C) is not correct. If were then the sum of the angles in the given


triangle would be instead of


Explanation for Incorrect Answer E :
te
Choice (E) is not correct. If were then the sum of the angles in the given
triangle would be instead of


is


eg

3
For every lawn mowers produced by a lawn mower factory, exactly are
defective. At this rate, how many lawn mowers were produced during a period in
which exactly


lawn mowers were defective?
nR

(A)


(B)

(C)

(D)

(E)

ANSWERS AND EXPLANATIONS


Explanation for Correct Answer D :
Choice (D) is correct. The rate of defective mower production is

If the rate continues then where x is the

number of lawn mowers produced when exactly of them are defective. Solving
for gives which reduces to

Explanation for Incorrect Answer A :


Choice (A) is not correct. Since the rate of defective mower production was
defective mowers out of the production of only lawn mowers would
result in fewer than defective mowers.

Explanation for Incorrect Answer B :


Choice (B) is not correct. This choice is the number of lawn mowers produced in a
period in which exactly were defective. The question asked for the number of

file://E:\\c7.htm 2006-11-12
The Official SAT Online Course 3/16

lawn mowers produced in a period in which exactly were defective.

Explanation for Incorrect Answer C :


Choice (C) is not correct. If lawn mowers were produced, then the number

of defective mowers would be The question asked for the

number of lawn mowers produced in a period in which exactly were defective.

Explanation for Incorrect Answer E :


Choice (E) is not correct. There were defective mowers per , so
lawn mowers would include defective ones. The question asked for the number
of lawn mowers produced in a period in which exactly were defective.

4 If what is the value of

(A)

(B)

(C)


(D)

(E)


d
ANSWERS AND EXPLANATIONS
Explanation for Correct Answer B :
Choice (B) is correct. re
is the same as So, and


te
Thus,


is


eg

Explanation for Incorrect Answer A :


Choice (A) is not correct. If the value of were equal to then
which is not equal to the given value of


nR

Explanation for Incorrect Answer C :


Choice (C) is not correct. If the value of were equal to then
which is not equal to the given value of

Explanation for Incorrect Answer D :


Choice (D) is not correct. If the value of were equal to then
which is not equal to the given value of

Explanation for Incorrect Answer E :


Choice (E) is not correct. If the value of were equal to then
which is not equal to the given value of

The figure above will be rotated about point in the direction indicated. Which
of the following represents the rotated figure?

file://E:\\c7.htm 2006-11-12
The Official SAT Online Course 4/16

(A)

(B)

(C)

(D)

(E)

ANSWERS AND EXPLANATIONS




d
Explanation for Correct Answer A :
Choice (A) is correct. This is the result of rotating the figure about in the
direction indicated.

re

te

Explanation for Incorrect Answer B :
is
Choice (B) is not correct. This figure would result from rotating the original figure
about in the direction indicated.


eg

Explanation for Incorrect Answer C :


Choice (C) is not correct. This figure would result from rotating the original figure


about in the opposite direction from that indicated in the question.
nR


Explanation for Incorrect Answer D :
Choice (D) is not correct. This figure would result from rotating the original figure
more than about in the direction indicated.

Explanation for Incorrect Answer E :


Choice (E) is not correct. This figure would result from rotating the original figure
less than about in the direction indicated.

6 percent of what is percent of


If is

(A)

(B)

(C)

(D)

(E)

ANSWERS AND EXPLANATIONS


Explanation for Correct Answer B :

Choice (B) is correct. Since is or of it follows that percent is

Thus, percent of is of which is

file://E:\\c7.htm 2006-11-12
The Official SAT Online Course 5/16

Explanation for Incorrect Answer A :


Choice (A) is not correct. Though is the value of the question asks for
percent of

Explanation for Incorrect Answer C :


Choice (C) is not correct. Though is percent of and is the value
of the question asked for percent of not of .

Explanation for Incorrect Answer D :

Choice (D) is not correct. Though percent is the question asked for

the value of percent of 10.

Explanation for Incorrect Answer E :

Choice (E) is not correct. Since is or of it follows that

percent is However, is not of


ed
er

st


In the diagram of roads above, the numbers represent road distances in miles, and
i

the arrows show the only directions in which travel is permitted on the roads. If the


eg

length of the longest route from to is miles, and if the length of the shortest
route from to is miles, then

(A)


nR

(B)


(C)

(D)

(E)

ANSWERS AND EXPLANATIONS


Explanation for Correct Answer E :
Choice (E) is correct. There are four ways to get from to The longest way is
miles, starting out traveling miles, then miles, then miles. The shortest
way is miles, starting out traveling miles, then miles, and then miles.
So the difference between the longest route and the shortest route is

Explanation for Incorrect Answer A :


Choice (A) is not correct. This is the length of the longest route, but the question
asked for the difference between the longest route and the shortest route.

Explanation for Incorrect Answer B :


Choice (B) is not correct. There is a route that has length but the question
asked for the difference between the longest route and the shortest route.

Explanation for Incorrect Answer C :


Choice (C) is not correct. This is the length of the shortest route, but the question
asked for the difference between the longest route and the shortest route.

file://E:\\c7.htm 2006-11-12
The Official SAT Online Course 6/16

Explanation for Incorrect Answer D :


Choice (D) is not correct. The length of the longest route is and the length of the
shortest route is so the difference is not

In the figure above, line (not shown) is perpendicular to and bisects


Which of the following points lies on line

(A)

(B)

(C)

ed
(D)


(E)

r

te
ANSWERS AND EXPLANATIONS


Explanation for Correct Answer D :
is
Choice (D) is correct. The midpoint of segment is The slope of segment


is so a perpendicular line will go through the point and will have
eg

a slope Using the point-slope form of the equation of a line, line is described
by the equation or The only answer choice with the


coordinate equal to the coordinate is
nR

Explanation for Incorrect Answer A :


Choice (A) is not correct. The coordinates define point If was on
line , then would not bisect since it would go through one of the
endpoints of

Explanation for Incorrect Answer B :


Choice (B) is not correct. The midpoint of is the point The line

containing and the point would bisect but would not be


perpendicular to

Explanation for Incorrect Answer C :


Choice (C) is not correct. The midpoint of is the point The line

containing and the point would bisect but would not be


perpendicular to

Explanation for Incorrect Answer E :


Choice (E) is not correct. The midpoint of is the point The line

containing and the point would bisect but would not be


perpendicular to

file://E:\\c7.htm 2006-11-12
The Official SAT Online Course 7/16

9
If and what is the value of

(A)

(B)

(C)

(D)

(E)

ANSWERS AND EXPLANATIONS


Explanation for Correct Answer A :
Choice (A) is correct. The number can be expressed as so the equation

can be rewritten as Thus, The numeric value of can

now be substituted into the second equation, which yields Solving for it

follows that



d
Explanation for Incorrect Answer B :

e
Choice (B) is not correct. If were equal to then would be and


er
Explanation for Incorrect Answer C :

st


Choice (C) is not correct. If were equal to then would be and
i


eg

Explanation for Incorrect Answer D :


Choice (D) is not correct. If were equal to then would be and
nR



Explanation for Incorrect Answer E :
U

Choice (E) is not correct. If were equal to then would be and

10

What is the radius of a circle whose circumference is

(A)

(B)

(C)

(D)

(E)

ANSWERS AND EXPLANATIONS


Explanation for Correct Answer A :
Choice (A) is correct. The formula for the circumference of a circle is where
is the radius of the circle. In this case, which simplifies to

file://E:\\c7.htm 2006-11-12
The Official SAT Online Course 8/16

Explanation for Incorrect Answer B :


Choice (B) is not correct. If the radius of the circle were then the circumference
would be instead of

Explanation for Incorrect Answer C :


Choice (C) is not correct. If the radius of the circle were then the circumference
would be instead of

Explanation for Incorrect Answer D :


Choice (D) is not correct. If the radius of the circle were then the circumference
would be instead of

Explanation for Incorrect Answer E :


Choice (E) is not correct. If the radius of the circle were then the circumference
would be instead of



d
11 How many of the prime factors of are greater than

(A) One
(B) Two
(C) Three
re

te
(D) Four

(E) Five


is

ANSWERS AND EXPLANATIONS


eg

Explanation for Correct Answer B :


Choice (B) is correct. The prime factorization of thirty is Of these
three prime factors, only two are greater than
nR

Explanation for Incorrect Answer A :


Choice (A) is not correct. Both and are prime factors of that are greater
than Hence, more than one prime factor is greater than

Explanation for Incorrect Answer C :


Choice (C) is not correct. While does have three prime factors, the question
asks how many of its prime factors are greater than

Explanation for Incorrect Answer D :


Choice (D) is not correct. The number only has three prime factors, so it
cannot have four prime factors that are greater than two.

Explanation for Incorrect Answer E :


Choice (E) is not correct. The number only has three prime factors, so it cannot
have five prime factors that are greater than two.

12

If the average (arithmetic mean) of the numbers above is what is in terms


of

file://E:\\c7.htm 2006-11-12
The Official SAT Online Course 9/16

(A)

(B)

(C)

(D)

(E)

ANSWERS AND EXPLANATIONS


Explanation for Correct Answer E :
Choice (E) is correct. Since the arithmetic mean of the three numbers is it

follows that This simplifies to or

Explanation for Incorrect Answer A :


Choice (A) is not correct. If were equal to then the average of the three

numbers would be However, the average of


d
and is

Explanation for Incorrect Answer B :

Choice (B) is not correct. If


re
were equal to
then the average of and


te
would be However, the average of and is


is

Explanation for Incorrect Answer C :


eg

Choice (C) is not correct. If were equal to then the average of the three

numbers would be However, the average of

and


is
nR


Explanation for Incorrect Answer D :

Choice (D) is not correct. If were equal to then the average of and

would be However, the average of and is

13

In the increasing sequence above, the first term is and the difference between any
two consecutive terms is What is the value of the fourth term in the sequence?

(A)

(B)

(C)

(D)

(E)

ANSWERS AND EXPLANATIONS

file://E:\\c7.htm 2006-11-12
The Official SAT Online Course 10/16

Explanation for Correct Answer C :


Choice (C) is correct. If the difference between any two consecutive terms is
then the fourth term is equal to the third term plus or Also,
the first term subtracted from the second term must be equal to So
which simplifies to Thus, the fourth term is

Explanation for Incorrect Answer A :


Choice (A) is not correct. This choice is the value of the first term, which is not
equal to the fourth term.

Explanation for Incorrect Answer B :


Choice (B) is not correct. This choice is the value of the third term, which is not
equal to the fourth term.

Explanation for Incorrect Answer D :


Choice (D) is not correct. If the value of the fourth term were then the value of
the third term would be If then making the second term
However, this would make the difference between the
second and third consecutive terms instead of

Explanation for Incorrect Answer E :




Choice (E) is not correct. If the value of the fourth term were then the value of

ed
the third term would be If then and the second term
would be This would mean the difference between the


second and third consecutive terms was instead of

r

te
14


is
Let the function f be defined by If what is the value of


(A)
eg

(B)


(C)
nR

(D)


(E)

ANSWERS AND EXPLANATIONS


Explanation for Correct Answer E :

Choice (E) is correct. If then or Solving

for gives .

Explanation for Incorrect Answer A :


Choice (A) is not correct. If were equal to then

Explanation for Incorrect Answer B :


Choice (B) is not correct. If were equal to then

Explanation for Incorrect Answer C :

Choice (C) is not correct. If were equal to then

file://E:\\c7.htm 2006-11-12
The Official SAT Online Course 11/16

Explanation for Incorrect Answer D :


Choice (D) is not correct. If were equal to then

15 In a windowless, cube-shaped storage room, the ceiling and walls, including a


door, are completely painted. The floor is not painted. If the painted area is equal to
square meters, what is the volume of the room, in cubic meters?
(A)

(B)

(C)

(D)

(E)

ANSWERS AND EXPLANATIONS


Explanation for Correct Answer C :
Choice (C) is correct. The area of five of the six square surfaces inside the room is
square meters. If represents the length in meters of each edge of the


square surfaces, then the area of each of the surfaces is and

ed
Solving for gives meters. Since the volume of a cube is

the storage room is r


cubic meters.

the length of any of its edges raised to the third power, it follows that the volume of


te

is

Explanation for Incorrect Answer A :


eg

Choice (A) is not correct. If the volume of the room were cubic meters, then
the length of each edge of the room would be meters and the area of the five


painted surfaces would be square meters instead of the
nR

square meters given in the problem.


Explanation for Incorrect Answer B :
Choice (B) is not correct. If the volume of the room were cubic meters, then

the length of each edge of the room would be meters and the area of the five

painted surfaces would be square meters instead of the

square meters given in the problem.

Explanation for Incorrect Answer D :


Choice (D) is not correct. If the volume of the room were cubic meters, then
the length of each edge of the room would be meters and the area of the

five painted surfaces would be square meters instead of

the square meters given in the problem.

Explanation for Incorrect Answer E :


Choice (E) is not correct. If the volume of the room were cubic meters, then
the length of each edge of the room would be meters and the area of the

five painted surfaces would be square meters instead of

the square meters given in the problem.

16

file://E:\\c7.htm 2006-11-12
The Official SAT Online Course 12/16

According to the formula if the temperature in degrees Fahrenheit

increases by by how much does the temperature in degrees Celsius


increase?
(A)

(B)

(C)

(D)

(E)

ANSWERS AND EXPLANATIONS


Explanation for Correct Answer B :
Choice (B) is correct. Solving for in the formula given, gives that

Every -degree change on the Fahrenheit scale results in a -

degree change on the Celsius scale. Therefore, a change of temperature of


degrees Fahrenheit corresponds to a change of temperature of

degrees Celsius.


d
Explanation for Incorrect Answer A :
re
Choice (A) is not correct. From the formula given in the problem, every -degree


te
change on the Celsius scale results in a -degree change on the Fahrenheit scale.


If the temperature in Celsius increased by the increase in the Fahrenheit
is

temperature would have been degrees, not degrees.


eg

Explanation for Incorrect Answer C :


Choice (C) is not correct. From the formula given in the problem, every -degree


nR

change on the Celsius scale results in a -degree change on the Fahrenheit scale.


If the temperature in Celsius increased by the increase in the Fahrenheit

temperature would have been degrees, not degrees.

Explanation for Incorrect Answer D :


Choice (D) is not correct. From the formula given in the problem, every -degree

change on the Celsius scale results in a -degree change on the Fahrenheit scale.

If the temperature in Celsius increased by the increase in the Fahrenheit

temperature would have been degrees, not

degrees.

Explanation for Incorrect Answer E :


Choice (E) is not correct. From the formula given in the problem, every -degree

change on the Celsius scale results in a -degree change on the Fahrenheit scale.

If the temperature in Celsius increased by the increase in the Fahrenheit

temperature would have been degrees, not degrees.

17
For all values of let be defined by Which of the following is

file://E:\\c7.htm 2006-11-12
The Official SAT Online Course 13/16

equal to

(A)

(B)

(C)

(D)

(E)

ANSWERS AND EXPLANATIONS


Explanation for Correct Answer D :
Choice (D) is correct. Since it follows that



d
Explanation for Incorrect Answer A :

re
Choice (A) is not correct. Since the definition for

must hold for so if were equal to


holds for all values of

then
it

would


te
have to be equal to However, from the original definition,

and


is

Explanation for Incorrect Answer B :


eg

Choice (B) is not correct. Since the definition for holds for all values of it

must hold for so if were equal to then


would have to be equal to However, from the original definition,
nR

and


Explanation for Incorrect Answer C :


Choice (C) is not correct. Since the definition for holds for all values of it
U

must hold for so if were equal to then

would have to be equal to However, from the original definition,

and

Explanation for Incorrect Answer E :


Choice (E) is not correct. Since the definition for holds for all values of it

must hold for so if were equal to then

would have to be equal to However, from the original

definition, and

18 If three different circles are drawn on a piece of paper, at most how many points can
be common to all three?

(A) None
(B) One
(C) Two

(D) Three

file://E:\\c7.htm 2006-11-12
The Official SAT Online Course 14/16

(E) Six

ANSWERS AND EXPLANATIONS


Explanation for Correct Answer C :
Choice (C) is correct. Two different circles can cross in at most two places. Since
two circles can only meet at two distinct points, a third circle can coincide with the
other two, at most, at those two places. An illustration of this is shown in the
figure.

Explanation for Incorrect Answer A :


Choice (A) is not correct. There can be at least one in common because three
circles can all share the same point.

Explanation for Incorrect Answer B :



Choice (B) is not correct. Though three circles can cross at only one point, the


question asks for the most points they can have in common. See the explanation

ed
for the correct answer.


Explanation for Incorrect Answer D :
Choice (D) is not correct. Though there are three circles, the circles can have at
r
most two points in common. See the explanation for the correct answer.


te
Explanation for Incorrect Answer E :
Choice (E) is not correct. Though the maximum number of points of intersection


is
common to two of the three circles could be six, all three circles would not have all
six points in common. See the explanation for the correct answer.


eg


19
A club is buying boxes of candy bars to sell for a fund-raiser. If each box contains
nR

candy bars, and each member sells bars each day, how many boxes are needed to
supply enough candy bars for members to sell for days?


(A)

(B)

(C)

(D)

(E)

ANSWERS AND EXPLANATIONS


Explanation for Correct Answer E :
Choice (E) is correct. Since each member sells candy bars a day, over the -
day period, each member will sell candy bars. Since there are members, a
total of candy bars is needed. Candy bars come in boxes with bars in

each box, so the group needs boxes.

Explanation for Incorrect Answer A :


Choice (A) is not correct. boxes would contain candy

file://E:\\c7.htm 2006-11-12
The Official SAT Online Course 15/16

bars. However, over the -day period, each of the members will sell candy
bars so only candy bars are needed.

Explanation for Incorrect Answer B :

Choice (B) is not correct. boxes would contain only candy bars. However,

over the -day period, each of the members will sell candy bars so
candy bars are needed.

Explanation for Incorrect Answer C :

Choice (C) is not correct. boxes would contain only candy bars.

However, over the -day period, each of the members will sell candy bars
so candy bars are needed.

Explanation for Incorrect Answer D :

Choice (D) is not correct. boxes would contain candy bars. However,

over the -day period, each of the members will sell candy bars so
candy bars are needed.

20

ed
er

st


eg

In the figure above, the coordinates of are and the coordinates of


are A point in square is to be chosen at random. If the probability


nR

that the point will be in the shaded triangle is what is the value of


(A)

(B)

(C)

(D)

(E)

ANSWERS AND EXPLANATIONS


Explanation for Correct Answer C :
Choice (C) is correct. The probability that a point chosen at random from the
square is in the shaded triangle is equal to the ratio of the area of the triangle to
the area of the square. The area of the square is If A is

the area of the triangle, then so The base of the triangle is

and the height of the triangle is Using the

formula for the area of a triangle, it follows that so

file://E:\\c7.htm 2006-11-12
The Official SAT Online Course 16/16

Explanation for Incorrect Answer A :


Choice (A) is not correct. The base of the triangle is

and the height of the triangle is If were then the area of the

triangle would be and this area should represent of

the area of the square. However, the square has area and is not

of

Explanation for Incorrect Answer B :


Choice (B) is not correct. The base of the triangle is

and the height of the triangle is If were then the area of the

triangle would be and this area should represent

of the area of the square. However, the square has area and is

not of

Explanation for Incorrect Answer D :


Choice (D) is not correct. If the value of were equal to then the -


coordinate of point would be However, the

point is to the right of the origin, so the -coordinate of point can not be


negative.

ed
Explanation for Incorrect Answer E :


Choice (E) is not correct. The base of the triangle is
er
and the height of the triangle is If were then the area of the triangle

would be


and this area should represent of the area of
t

the square. However, the square has area and is not of
is


eg


nR


Back to Score Report

Copyright 2006 The College Board. All rights reserved. Privacy Policy Terms of Use Contact Us

file://E:\\c7.htm 2006-11-12
The Official SAT Online Course 1/16

Help | Profile | My Organizer | My Bookmarks | Logout

Answers and Explanations

Test Sections Back to Score Report

Section 1 View Answers and Explanations


Section 2 Online - Practice Test #3

Section 3
1
Section 4 Originally ------- mainly by young, urban audiences, rap music was ultimately -------
Section 5 by its appreciative listeners of all ages across the country.
Section 7 (A) admired . . embraced
Section 8 (B) performed . . condemned
Section 9
(C) derided . . ignored
Section 10
(D) appropriated . . relinquished

(E) applauded . . instigated

ANSWERS AND EXPLANATIONS



d
Explanation for Correct Answer A :

re
Choice (A) is correct. "Admired" means thought highly of. "Embraced" means held


closely. If one were to insert these terms into the text, the sentence would read
"Originally admired mainly by young, urban audiences, rap music was ultimately
embraced by its appreciative listeners of all ages across the country." The phrase


te
"appreciative listeners of all ages" indicates that rap music's fan base expanded
from what it was "originally." The correct response is the only one in which both
missing terms have a positive meaning, indicating that the appreciation began with


is
one group and expanded to include a much larger group. Both "admired" and
"embraced" are positive terms.


eg


Explanation for Incorrect Answer B :
nR

Choice (B) is incorrect. "Performed" means acted out or achieved. "Condemned"


means disapproved of. If one were to insert these terms into the text, the sentence


would read "Originally performed mainly by young, urban audiences, rap music was
ultimately condemned by its appreciative listeners of all ages across the country."
The phrase "appreciative listeners of all ages" indicates that rap music's fan base

expanded from what it was "originally." The correct response is the only one in
which both missing terms have a positive meaning, indicating that the appreciation
began with one group and expanded to include a much larger group. Audiences do
not typically perform music, and listeners would not be described as appreciative of
something they are condemning.

Explanation for Incorrect Answer C :


Choice (C) is incorrect. "Derided" means ridiculed. "Ignored" means disregarded. If
one were to insert these terms into the text, the sentence would read "Originally
derided mainly by young, urban audiences, rap music was ultimately ignored by its
appreciative listeners of all ages across the country." The phrase "appreciative
listeners of all ages" indicates that rap music's fan base expanded from what it was
"originally." The correct response is the only one in which both missing terms have
a positive meaning, indicating that the appreciation began with one group and
expanded to include a much larger group. Both "derided" and "ignored" have a
negative, not a positive, connotation. Music would not be both appreciated and
ignored by the same group of listeners.

Explanation for Incorrect Answer D :


Choice (D) is incorrect. "Appropriated" means something is taken and used.
"Relinquished" means abandoned. If one were to insert these terms into the text,
the sentence would read "Originally appropriated mainly by young, urban
audiences, rap music was ultimately relinquished by its appreciative listeners of all
ages across the country." The phrase "appreciative listeners of all ages" indicates
that rap music's fan base expanded from what it was "originally." The correct
response is the only one in which both missing terms have a positive meaning,
indicating that the appreciation began with one group and expanded to include a
much larger group. Listeners would not relinquish the music they appreciate.

file://E:\\c8.htm 2006-11-12
The Official SAT Online Course 2/16

Explanation for Incorrect Answer E :


Choice (E) is incorrect. "Applauded" means approved of. "Instigated" means
provoked. If one were to insert these terms into the text, the sentence would read
"Originally applauded mainly by young, urban audiences, rap music was ultimately
instigated by its appreciative listeners of all ages across the country." The phrase
"appreciative listeners of all ages" indicates that rap music's fan base expanded
from what it was "originally." The correct response is the only one in which both
missing terms have a positive meaning, indicating that the appreciation began with
one group and expanded to include a much larger group. Listeners would not have
instigated rap music.

2 It was out of ------- that Professor Green, the author of several highly respected
books in his field, described himself to his colleagues as -------.
(A) embarrassment . . a paragon
(B) magnanimity . . an avenger
(C) insolence . . a pedant

(D) egotism . . an apprentice

(E) modesty . . a dilettante

ANSWERS AND EXPLANATIONS




d
Explanation for Correct Answer E :
Choice (E) is correct. "Modesty" means humility. "A dilettante" is someone who

re
merely dabbles in a subject. If one were to insert these terms into the text, the


sentence would read "It was out of modesty that Professor Green, the author of
several highly respected books in his field, described himself to his colleagues as a
dilettante." The correct response for the second missing term will be the result of


te
Professor Green's feelings as described in the first missing term. Because Professor
Green is highly respected in his field, his description of himself as a dilettante is
simply modest; he describes himself at less than his true value.


is


eg

Explanation for Incorrect Answer A :


Choice (A) is incorrect. "Embarrassment" means self-consciousness. "A paragon" is


a model of excellence. If one were to insert these terms into the text, the sentence
nR

would read "It was out of embarrassment that Professor Green, the author of
several highly respected books in his field, described himself to his colleagues as a


paragon." The correct response for the second missing term will be the result of
Professor Green's feelings as described in the first missing term. Professor Green
would not describe himself as a paragon, proclaiming his excellence, as a result of

his embarrassment.

Explanation for Incorrect Answer B :


Choice (B) is incorrect. "Magnanimity" means forgiving. "An avenger" is one who
seeks revenge. If one were to insert these terms into the text, the sentence would
read "It was out of magnanimity that Professor Green, the author of several highly
respected books in his field, described himself to his colleagues as an avenger." The
correct response for the second missing term will be the result of Professor Green's
feelings as described in the first missing term. Professor Green cannot be both an
avenger and magnanimous.

Explanation for Incorrect Answer C :


Choice (C) is incorrect. "Insolence" means disrespect. "A pedant" is someone overly
concerned with the formal rules of his field of knowledge. If one were to insert
these terms into the text, the sentence would read "It was out of insolence that
Professor Green, the author of several highly respected books in his field, described
himself to his colleagues as a pedant." The correct response for the second missing
term will be the result of Professor Green's feelings as described in the first missing
term. Professor Green would be unlikely to describe himself as a pedant, or
someone overly concerned with the formal rules of his field of knowledge. If he
described himself as a pedant, however, it would not be as a result of insolence.

Explanation for Incorrect Answer D :


Choice (D) is incorrect. "Egotism" means conceit. "An apprentice" is a beginning
learner. If one were to insert these terms into the text, the sentence would read "It
was out of egotism that Professor Green, the author of several highly respected
books in his field, described himself to his colleagues as an apprentice." The correct
response for the second missing term will be the result of Professor Green's feelings

file://E:\\c8.htm 2006-11-12
The Official SAT Online Course 3/16

as described in the first missing term. If Professor Green wrote several highly
respected books and was egotistical, he would hardly describe himself as an
apprentice, or beginning learner.

3 Historian Carlo Botta often contradicted himself, as when he first championed and
then ------- the ideals of the French Revolution.
(A) invoked
(B) investigated
(C) conceived

(D) coveted

(E) denounced

ANSWERS AND EXPLANATIONS


Explanation for Correct Answer E :
Choice (E) is correct. "Denounced" means spoke ill of. If one were to insert this
term into the text, the sentence would read "Historian Carlo Botta often
contradicted himself, as when he first championed and then denounced the ideals of


the French Revolution." The word "contradicted" indicates that the correct response
will mean the opposite of championing, or supporting, ideals. Denouncing ideals is
the opposite of championing them. Thus, to do both would be to contradict oneself.


Explanation for Incorrect Answer A :
r ed
Choice (A) is incorrect. "Invoked" means to call upon. If one were to insert this


te
term into the text, the sentence would read "Historian Carlo Botta often
contradicted himself, as when he first championed and then invoked the ideals of
the French Revolution." The word "contradicted" indicates that the correct response


will mean the opposite of championing, or supporting, ideals. For Botta to invoke
is
the Revolution's ideals after having championed them would not be a contradiction.


eg

Explanation for Incorrect Answer B :


Choice (B) is incorrect. "Investigated" means researched. If one were to insert this
term into the text, the sentence would read "Historian Carlo Botta often


contradicted himself, as when he first championed and then investigated the ideals
of the French Revolution." The word "contradicted" indicates that the correct
nR

response will mean the opposite of championing, or supporting, ideals. While it


might have been better for Botta to investigate the ideals before he championed
them, his investigation doesn't mean he is contradicting himself.


Explanation for Incorrect Answer C :
U

Choice (C) is incorrect. "Conceived" means thought up. If one were to insert this
term into the text, the sentence would read "Historian Carlo Botta often
contradicted himself, as when he first championed and then conceived the ideals of
the French Revolution." The word "contradicted" indicates that the correct response
will mean the opposite of championing, or supporting, ideals. Botta could not have
conceived the ideals of the French Revolution after having championed them. He
would have conceived them and later championed them.

Explanation for Incorrect Answer D :


Choice (D) is incorrect. "Coveted" means strongly desired. If one were to insert this
term into the text, the sentence would read "Historian Carlo Botta often
contradicted himself, as when he first championed and then coveted the ideals of
the French Revolution." The word "contradicted" indicates that the correct response
will mean the opposite of championing, or supporting, ideals. Coveting ideals may
be odd; however, coveting and championing ideals is not a contradiction.

4 Luisa worked with extreme precision, ------- that served her well in her law career.

(A) a meticulousness
(B) an effrontery
(C) an inhibition

(D) a litigiousness

file://E:\\c8.htm 2006-11-12
The Official SAT Online Course 4/16

(E) impetuousness

ANSWERS AND EXPLANATIONS


Explanation for Correct Answer A :
Choice (A) is correct. "Meticulousness" means excessive care or attention to detail.
If one were to insert this term into the text, the sentence would read "Luisa worked
with extreme precision, a meticulousness that served her well in her law career." A
person who works with extreme precision would be properly described as
demonstrating "meticulousness."

Explanation for Incorrect Answer B :


Choice (B) is incorrect. "Effrontery" means offensive boldness. If one were to insert
this term into the text, the sentence would read "Luisa worked with extreme
precision, an effrontery that served her well in her law career." Working with
precision is very different from the type of shameless or offensive behavior that is
termed "effrontery."

Explanation for Incorrect Answer C :


Choice (C ) is incorrect. "Inhibition" means something that holds a person back. If


one were to insert this term into the text, the sentence would read "Luisa worked
with extreme precision, an inhibition that served her well in her law career." As the
sentence makes clear, working precisely allowed Luisa to succeed; it did not hold


d
her back.

e
Explanation for Incorrect Answer D :


Choice (D) is incorrect. "Litigiousness" means a tendency to engage in lawsuits. If
one were to insert this term into the text, the sentence would read "Luisa worked
er
with extreme precision, a litigiousness that served her well in her law career."


Although "litigious" is a word that would describe a lawyer, it does not mean
extreme precision.
t

is
Explanation for Incorrect Answer E :
Choice (E) is incorrect. "Impetuousness" means impulsiveness. If one were to insert
this term into the text, the sentence would read "Luisa worked with extreme


eg

precision, an impetuousness that served her well in her law career." Work that is
done on impulse is probably not performed with extreme precision.


nR


5
In 1916 Yellowstone National Park had only 25 bison, but the population has since
------- to more than 2,000.


(A) dispersed
U

(B) mediated
(C) attenuated

(D) burgeoned

(E) reconciled

ANSWERS AND EXPLANATIONS


Explanation for Correct Answer D :
Choice (D) is correct. "To burgeon" means to grow and flourish. If one were to
insert this term into the text, the sentence would read "In 1916 Yellowstone
National Park had only 25 bison, but the population has since burgeoned to more
than 2,000." Since the park began with 25 bison and now has more than 2,000, the
correct answer is the word that most closely means "increased." "Burgeoned" and
"increased" are similar in meaning.

Explanation for Incorrect Answer A :


Choice (A) is incorrect. "To disperse" means to spread out. If one were to insert this
term into the text, the sentence would read "In 1916 Yellowstone National Park had
only 25 bison, but the population has since dispersed to more than 2,000." Since
the park began with 25 bison and now has more than 2,000, the correct answer is

file://E:\\c8.htm 2006-11-12
The Official SAT Online Course 5/16

the word that most closely means "increased." However, "dispersed" and
"increased" are not similar in meaning.

Explanation for Incorrect Answer B :


Choice (B) is incorrect. "To mediate" means to settle a dispute between conflicting
parties. If one were to insert this term into the text, the sentence would read "In
1916 Yellowstone National Park had only 25 bison, but the population has since
mediated to more than 2,000." Since the park began with 25 bison and now has
more than 2,000, the correct answer is the word that most closely means
"increased." However, "mediated" and "increased" are not similar in meaning.
Mediation does not describe the bison's increase in population.

Explanation for Incorrect Answer C :


Choice (C) is incorrect. "To attenuate" means to lessen in amount or force. If one
were to insert this term into the text, the sentence would read "In 1916 Yellowstone
National Park had only 25 bison, but the population has since attenuated to more
than 2,000." Since the park began with 25 bison and now has more than 2,000, the
correct answer is the word that most closely means "increased." However,
"attenuated" and "increased" are not similar in meaning. What happened to the
bison population is the opposite of attenuate.

Explanation for Incorrect Answer E :


Choice (E) is incorrect. "To reconcile" means to adapt or make peace. If one were to
insert this term into the text, the sentence would read "In 1916 Yellowstone
National Park had only 25 bison, but the population has since reconciled to more


than 2,000." Since the park began with 25 bison and now has more than 2,000, the
correct answer is the word that most closely means "increased." However,
"reconciled" and "increased" are not similar in meaning.


ed
6 Though surgeon and researcher Charles Drew never enjoyed celebrity, he truly
deserves to be ------- for his lifes achievements.
r

te
(A) mollified
(B) lionized


is
(C) accosted

(D) galvanized


eg

(E) vilified

ANSWERS

AND EXPLANATIONS
nR


Explanation for Correct Answer B :
Choice (B) is correct. "Lionized" means celebrated. If one were to insert this term
into the text, the sentence would read "Though surgeon and researcher Charles
Drew never enjoyed celebrity, he truly deserves to be lionized for his lifes

achievements." This sentence makes sense because it states that Drew deserves to
be celebrated for his many achievements.

Explanation for Incorrect Answer A :


Choice (A) is incorrect. "Mollified" means soothed. If one were to insert this term
into the text, the sentence would read "Though surgeon and researcher Charles
Drew never enjoyed celebrity, he truly deserves to be mollified for his lifes
achievements." There is no indication in this sentence that Drew is angry or upset,
thus it is unlikely that he would deserve or need to be soothed.

Explanation for Incorrect Answer C :


Choice (C) is incorrect. "Accosted" means approached aggressively. If one were to
insert this term into the text, the sentence would read "Though surgeon and
researcher Charles Drew never enjoyed celebrity, he truly deserves to be accosted
for his lifes achievements." This sentence is illogical because it does not explain
why Drew, a man of many accomplishments, would deserve to be approached
aggressively.

Explanation for Incorrect Answer D :


Choice (D) is incorrect. "Galvanized" means spurred into action. If one were to
insert this term into the text, the sentence would read "Though surgeon and
researcher Charles Drew never enjoyed celebrity, he truly deserves to be
galvanized for his lifes achievements." It is unclear why a man of many
achievements would deserve or need to be spurred into action.

file://E:\\c8.htm 2006-11-12
The Official SAT Online Course 6/16

Explanation for Incorrect Answer E :


Choice (E) is incorrect. "Vilified" means to be criticized harshly. If one were to insert
this term into the text, the sentence would read "Though surgeon and researcher
Charles Drew never enjoyed celebrity, he truly deserves to be vilifed for his lifes
achievements." If Drew did anything to deserve harsh criticism, this sentence does
not mention it.

The passage below is from a 1991 autobiography that focuses on an African American
womans adolescent experiences at a prestigious boarding school. The passage
describes one part of a meeting of parents, admissions officers, and prospective
students. The story the mother recounts at this meeting took place in 1965.

My mother began to tell a story about a


science award
I had won in third grade. She started with the
winning
the long, white staircase in the auditorium, and
how the
announcer called my name twice because we
Line
were way at
the back and it took me so long to get down

ed
5
those steps.
Mamas eyes glowed. She was a born
raconteur, able

er
to increase the intensity of her own presence
and fill the

t
room. She was also a woman who seldom found


is

new audi-
ences for her anecdotes, so she made herself


eg

happy, she
insisted, with us children, her mother, her


10
sisters, her
nR

grandparentsan entire clan of storytellers


competing for
a turn on the family stage. This time all eyes

were on my
mother. Her body, brown and plump and
smooth, was shot
through with energy. This time the story had a
purpose.
She told them how my science experiment
15
almost did
not get considered in the citywide competition.
My third-
grade teacher, angry that Id forgotten to
bring a large box
for displaying and storing the experiment, made
me pack
it up to take home. (Our teacher had told us
that the boxes
were needed to carry the experiments from our
20
class to the
exhibition room, and shed emphasized that

file://E:\\c8.htm 2006-11-12
The Official SAT Online Course 7/16

she would not


be responsible for finding thirty boxes on the
day of the
fair. Without a box, the experiment would have
to go home.
Other kids, White kids, had forgotten boxes
during the
week. Theyd brought boxes the next day. I
25
asked for the
same dispensation, but was denied. The next
day was the
fair, she said. That was different.)
I came out of school carrying the pieces of
an experi-
ment my father had picked out for me from a
textbook.
This was a simple buoyancy experiment where I
30
weighed

each object in the air and then in water, to
prove they

d
weighed less in water. I had with me the scale,
a brick, a
re
piece of wood, a bucket, and a carefully lettered


poster.
te
Well, my mother marched me and my


armload of
is

buoyant materials right back into school and


35
caught the


eg

teacher before she left. The box was the only


problem?


Just the box? Nothing wrong with the
nR

experiment? An


excited eight year old had forgotten a lousy,
stinking box

that you can get from the supermarket and for


that, she
was out of the running? The teacher said I had
40
to learn to
follow directions. My mother argued that I had
followed
directions by doing the experiment by myself,
which was
more than you could say for third graders
whod brought
dry-cell batteries that lit light bulbs and papier-
mch
45 volcanoes that belched colored lava.
Dont you ever put me in a position like
that again,
Mama said when we were out of earshot of the
classroom.
You never know who is just waiting for an

file://E:\\c8.htm 2006-11-12
The Official SAT Online Course 8/16

excuse to shut
us out.
We got the box; my experiment went into
50
the fair; I won
the prize at school. I won third prize for my age
group in
the city.
When Mama finished her story, my ears
began to burn.
I could not help but believe that they would see
through
this transparent plug, and before I had even
55
laid hands on
an application. Theyd think we were forward
and pushy.
I forgot, for the moment, how relieved Id felt
when


Mama had stood in front of that teacher
defending me


with a blinding sense of purpose, letting the

ed
teacher
60
alone as r
know that I was not as small and Black and


te
I seemed, that I came from somewhere, and
where I


came from, shed better believe, somebody
is

was home.


The other mothers nodded approvingly. My
eg

father


gave me a wide, clever-girl smile. The officials
from
nR


the school looked at me deadpan. They seemed
65
amused
by my embarrassment.

The story was an answer (part rebuke and


part condo-
lence) to the school stories that the admissions
people
told, where no parents figured at all. It was a
message
about her maternal concerns, and a way to
70
prove that
racism was not some vanquished enemy, but a
real, live
person, up in your face, ready, for no apparent
reason,
to mess with your kid. When I was in third
grade, Mama
could do her maternal duty and face down a
White teacher
who would have deprived me of an award. Who
75
at this

file://E:\\c8.htm 2006-11-12
The Official SAT Online Course 9/16

new school would stand up for her child in her


stead?
7 In line 11, competing portrays the members of the authors family as
(A) vying for the mothers attention
(B) feeling eager to tell their own stories
(C) taking issue with each other over household duties

(D) selectively sharing information about their experiences

(E) comparing educational accomplishments

ANSWERS AND EXPLANATIONS


Explanation for Correct Answer B :
Choice (B) is correct. "Eager" means enthusiastic or impatient. The author describes
her family as "an entire clan of storytellers competing for a turn on the family
stage" (lines 11-12). This sentence indicates that the entire family was "eager" for
a chance to tell their stories on the "family stage."

Explanation for Incorrect Answer A :



ed
Choice (A) is incorrect. The sentence clearly states that the mother was just one of
"an entire clan of storytellers" (line 11). There is no indication that anyone was
trying to get her attention.

Explanation for Incorrect Answer C :



er
Choice (C) is incorrect. There is no mention of household duties or the family's


feelings about them in the passage.
t
Explanation for Incorrect Answer D :


is
Choice (D) is incorrect. The passage does not indicate that family members were
selective about sharing information, only that they competed eagerly to be heard.


eg

Explanation for Incorrect Answer E :


Choice (E) is incorrect. The passage refers to the family members' desire to have
their stories heard, regardless of subject. Further, the only educational


accomplishments mentioned are those of the daughter.
nR

8


U

The third paragraph (lines 15-27) presents the authors third-grade teacher as being
primarily

(A) critical of the authors grandiose ambitions


(B) disillusioned about her students lack of interest in science
(C) concerned that children would never develop a sense of decorum

(D) arbitrary in implementing rules affecting the class

(E) bitter and outspoken about injustices in her school district

ANSWERS AND EXPLANATIONS


Explanation for Correct Answer D :
Choice (D) is correct. The author writes, "Other kids, White kids, had forgotten
boxes during the week. They'd brought boxes the next day. I asked for the same
dispensation, but was denied" (lines 24-26). According to the passage, the author's
third grade teacher arbitrarily made allowances for some students (White students
in particular) and not for others.

Explanation for Incorrect Answer A :


Choice (A) is incorrect. The third paragraph presents the teacher as critical of the
author for not having followed directions, not for having overly grand ambitions.

file://E:\\c8.htm 2006-11-12
The Official SAT Online Course 10/16

Explanation for Incorrect Answer B :


Choice (B) is incorrect. The paragraph doesn't indicate how the teacher feels about
her students' interest in science; it only indicates to what degree and for which
students she is willing to bend her rules.

Explanation for Incorrect Answer C :


Choice (C) is incorrect. The teacher does not indicate in the third paragraph or
anywhere else in the passage that she is concerned about the students' decorum, or
manners.

Explanation for Incorrect Answer E :


Choice (E) is incorrect. Nowhere in the passage does the teacher mention her
feelings about injustice or about the school district. On the contrary, the author
suggests that the teacher herself is guilty of injustice.

9 In line 26, dispensation refers to permission for the author to

(A) have an additional day to complete the experiment


(B) bring a container for her experiment the next day
(C) ask her father to help her design the display

(D) leave school early to look for a box

(E) discuss her experiment with the other children



d
ANSWERS AND EXPLANATIONS
Explanation for Correct Answer B :

re
Choice (B) is correct. "Dispensation" means favor or courtesy. The sentences
immediately preceding line 26 indicate that the author wished to receive the same


te
"dispensation," the same courtesy, given the "White" children, namely, being
allowed to bring a box the next day.


is


eg

Explanation for Incorrect Answer A :


Choice (A) is incorrect. The author has already completed the experiment and does
not need "an additional day to complete the experiment."


nR

Explanation for Incorrect Answer C :


Choice (C) is incorrect. The passage indicates that the dispensation the author is


requesting is to bring a box the following day, not permission to ask her father for
help.

Explanation for Incorrect Answer D :


Choice (D) is incorrect. The author does not ask to leave school early to look for a
box, but to bring a box the next day.

Explanation for Incorrect Answer E :


Choice (E) is incorrect. The passage indicates that the author wants a
"dispensation" to bring a box the following day, not to discuss the experiment with
others.

10
The child most likely intended to use the bucket (line 33) as

(A) an object to counterbalance the weight of another object


(B) a means of steadying the scale in the experiment
(C) a container to carry the other materials for the project

(D) a receptacle for the water used in the experiment

(E) a way of transporting liquid from place to place

ANSWERS AND EXPLANATIONS


Explanation for Correct Answer D :
Choice (D) is correct. The author describes the experiment as a "simple buoyancy

file://E:\\c8.htm 2006-11-12
The Official SAT Online Course 11/16

experiment" which involves weighing "each object in the air and then in water, to
prove they weighed less in water" (lines 30-32). Since the experiment calls for
weighing objects in water, the bucket is most likely to be used to hold the water
that is needed.

Explanation for Incorrect Answer A :


Choice (A) is incorrect. The experiment involves weighing particular objects in air
and in water (lines 30-32). The author does not mention the use of objects to
counterbalance the weight of other objects.

Explanation for Incorrect Answer B :


Choice (B) is incorrect. The author does not mention the need to steady the scale.
Further, the piece of wood or the brick from the list would be more appropriate
objects to use to steady a scale.

Explanation for Incorrect Answer C :


Choice (C) is incorrect. The author's statement that "I came out of school carrying
the pieces of an experiment" (lines 28-29) suggests that the author does not have a
suitable container to carry the materials for the project. Because one of the main
points of the story is that the author does not have a suitable container for her
materials, it is clear that the bucket is not being used for this purpose.

Explanation for Incorrect Answer E :


Choice (E) is incorrect. The experiment calls for weighing objects in water; this does
not require liquid to be transported from place to place.


d
11

(A) sudden bewilderment re


Lines 34-36 (Well . . . left) portray the mothers attitude of


te
(B) weary disappointment


(C) protective generosity
is
(D) overwhelming dismay


eg

(E) righteous indignation


ANSWERS AND EXPLANATIONS
nR

Explanation for Correct Answer E :


Choice (E) is correct. "Righteous indignation" suggests just or correct anger. The
passage states, "Well, my mother marched me and my armload of buoyant
materials right back into the school" (lines 34-35). This shows the mother's actions

as determined and forceful; they are part of an angry response to what she sees as
an injustice committed against her daughter.

Explanation for Incorrect Answer A :


Choice (A) is incorrect. The mother's decisive action in seeking out the teacher
indicates not bewilderment but anger and determination.

Explanation for Incorrect Answer B :


Choice (B) is incorrect. The mother is not disappointed but angry and determined to
rectify the situation. Further, there is no sign that the mother is weary of standing
up for her daughter.

Explanation for Incorrect Answer C :


Choice (C) is incorrect. The mother is certainly protective of her daughter's
interests, but her attitude is not one of generosity. Rather, she shows her
determination to right a wrong.

Explanation for Incorrect Answer D :


Choice (D) is incorrect. The mother is dismayed but hardly overwhelmed, as her
actions in immediately returning to the school to confront the teacher clearly
indicate.

file://E:\\c8.htm 2006-11-12
The Official SAT Online Course 12/16

12 The mother most probably intended the questions in lines 36-40 to


(A) underscore the absurdity of the teachers position
(B) request clarification from the child about the incident
(C) express concern over her daughters forgetfulness

(D) lessen the childs preoccupation with how her project would be received

(E) help herself understand her childs defensiveness about the box issue

ANSWERS AND EXPLANATIONS


Explanation for Correct Answer A :
Choice (A) is correct. The mother's questions, "The box was the only problem? Just
the box? Nothing wrong with the experiment? An excited eight year old had
forgotten a lousy, stinking box" and "for that she was out of the running?" (lines
36-40), are part of her doubting query. The mother clearly believes that the teacher
has placed undue importance on the box. Her questions are intended to highlight
the "absurdity of the teacher's position."

Explanation for Incorrect Answer B :


Choice (B) is incorrect. The questions are being put to the teacher and not to the
child. This is clearly shown by the continuing dialogue which occurs between mother


d
and teacher in lines 40-45.

re
Explanation for Incorrect Answer C :


Choice (C) is incorrect. The mother's disdain for the idea that a box alone should
keep her daughter's experiment out of the competition indicates that she is not
concernedor at least not interested in expressing concern to the teacherabout
te

her daughter's forgetfulness.

Explanation for Incorrect Answer D :


is
Choice (D) is incorrect. The questions are being asked of the teacher. The mother is
more interested in securing the author's place in the competition than in reassuring
the author about the project's reception.


eg

Explanation for Incorrect Answer E :


Choice (E) is incorrect. The mother does not need help understanding her child's


defensiveness about the box, since her questions indicate that she finds the issue
nR

ridiculous.



13
U

Between the mention of a hypothetical box in line 23 and its characterization in


line 38, the box has changed from a

(A) requirement to something that is no longer needed


(B) necessity to something that has little inherent value
(C) diversion to something that requires a desperate search

(D) tool to something that is a source of entertainment

(E) puzzle to something that provides clarity and strength

ANSWERS AND EXPLANATIONS


Explanation for Correct Answer B :
Choice (B) is correct. In line 23, the box as the teacher sees it is a necessary
element for the presentation of the experiment at the science fair. In line 38, it is,
as the mother sees it, a "lousy, stinking box that you can get from the
supermarket," and its degree of importance has shifted from necessary to minor.

Explanation for Incorrect Answer A :


Choice (A) is incorrect. The box is still a requirement, as indicated by the fact that
the mother and daughter find a box for the experiment materials before the
experiment proceeds to the fair.

file://E:\\c8.htm 2006-11-12
The Official SAT Online Course 13/16

Explanation for Incorrect Answer C :


Choice (C) is incorrect. Neither the teacher nor the author considered the box a
"diversion," or something that is amusing, and it does not later require a "desperate
search." As the mother points out, a box can be found almost anywhere (line 39).

Explanation for Incorrect Answer D :


Choice (D) is incorrect. The box is never a tool, but a required carrier for the
experiment. At no point is the box considered a source of entertainment for anyone
involved.

Explanation for Incorrect Answer E :


Choice (E) is incorrect. The box is never presented as either a puzzle or as
something that provides clarity and strength. It is a requirement that later
becomes, as the mother indicates, a petty impediment to her daughter's potential
success.

14 In line 55, the author uses the word plug primarily to emphasize her feeling that

(A) the conversational void was nearly intolerable


(B) the boarding school had been highly overrated
(C) her mother had gone too far in promoting her

(E)
(D) her mothers words and actions were entirely at odds

the interviewers praise would prove to be insincere


d
ANSWERS AND EXPLANATIONS
Explanation for Correct Answer C :
re

te
Choice (C) is correct. In this context, the word "plug" means promote or boost. The
author uses the word "plug," a word associated with selling a product, to express
her sense that her mother is overpromoting her.


is


eg

Explanation for Incorrect Answer A :


Choice (A) is incorrect. The author is not embarrassed because of a pause in


conversation; she is embarrassed because her mother is openly promoting her.
nR

Explanation for Incorrect Answer B :


Choice (B) is incorrect. In this passage, the author does not indicate what her
feelings are about the boarding school.

Explanation for Incorrect Answer D :


Choice (D) is incorrect. The author feels that her mother's words and actions are in
harmony; both are intended to promote and protect her daughter.

Explanation for Incorrect Answer E :


Choice (E) is incorrect. The author uses the word "plug" to describe her feelings
about what she sees as her mother's shameless promotion. She worries that this
promotion will have the unintended effect of causing the interviewers not to praise
her, sincerely or otherwise, but to see her family as "forward and pushy" (line 56).

15 In line 59, blinding suggests all of the following EXCEPT

(A) unswerving
(B) dazzling
(C) overpowering

(D) determined

(E) sudden

ANSWERS AND EXPLANATIONS


Explanation for Correct Answer E :

file://E:\\c8.htm 2006-11-12
The Official SAT Online Course 14/16

Choice (E) is correct because "blinding" is not used to mean "sudden" in this
context. We can infer that the mother's instinct is always to defend her daughter
and that her sense of purpose can therefore not be described as "sudden."

Explanation for Incorrect Answer A :


Choice (A) is incorrect because the mother's "blinding" sense of purpose can be
described as "unswerving," or steady.

Explanation for Incorrect Answer B :


Choice (B) is incorrect because the mother's "blinding" sense of purpose was
sufficiently effective to overrule, and thus stun or dazzle, the teacher.

Explanation for Incorrect Answer C :


Choice (C) is incorrect because "overpowering" is a word that can be used to
describe the mother's "blinding" sense of purpose. Indeed, her argument
overpowers the teacher's objections.

Explanation for Incorrect Answer D :


Choice (D) is incorrect because the mother's "blinding" sense of purpose can
certainly be described as "determined."

16
The phrase somebody was home (line 62) captures the mothers


ed
(A) desire to create a comfortable life for her family
(B) eagerness to learn the results of the interview

r
(C) despair over the unfairness of the teachers instructions

(D) need to capture everyones full attention


te
(E) sense of responsibility toward her daughter


is

ANSWERS AND EXPLANATIONS


eg

Explanation for Correct Answer E :


Choice (E) is correct. The phrase "somebody was home" is part of a sentence in
which the author describes her mother defending her. For the author, her mother's


vigorous defense is a way of conveying that the author is important to and
nR

supported by her family. This clearly captures the mother's "sense of responsibility
toward her daughter."

Explanation for Incorrect Answer A :


Choice (A) is incorrect. Undoubtedly the mother wants to create a comfortable life
for her family, but the phrase in line 62 refers specifically to the mother's support
and defense of her daughter.

Explanation for Incorrect Answer B :


Choice (B) is incorrect. Although we can assume that the mother was eager to learn
the results of the interview, the passage describes only part of the interview. The
passage does not discuss the results of the interview or the feelings of the
participants as to those results. Further, the phrase in line 62 refers to an earlier
event, not to the interview.

Explanation for Incorrect Answer C :


Choice (C) is incorrect. The mother's story indicates that she feels not despair but
determination to confront injustice toward her daughter.

Explanation for Incorrect Answer D :


Choice (D) is incorrect. Although the passage indicates elsewhere that the mother
enjoys attention, the phrase "somebody was home" refers specifically to her
defense of her daughter.

17 The details presented in lines 63-66 primarily serve to

file://E:\\c8.htm 2006-11-12
The Official SAT Online Course 15/16

(A) illustrate the various reactions to the mothers story


(B) satirize the pomposity of the school officials
(C) emphasize the sense of unity among parents of prospective students

(D) convey a sense of unease

(E) clarify a preceding statement

ANSWERS AND EXPLANATIONS


Explanation for Correct Answer A :
Choice (A) is correct. These lines indicate the various reactions, ranging from
approval to amusement, of the story's listeners: "[t]he other mothers nodded
approvingly. My father gave me a wide, clever-girl smile. The officials from the
school looked at me deadpan" (lines 63-65).

Explanation for Incorrect Answer B :


Choice (B) is incorrect. The details in lines 63-66 are straightforward rather than
satirical. The author interprets the school officials' "deadpan" reaction as amused,


not pompous or arrogant.

Explanation for Incorrect Answer C :


Choice (C) is incorrect. Although the other mothers nod in approval, possibly

d
signaling a sense of unity with the author's mother, lines 63-66 also stress how the
author herself and the school officials reacted to the mother's story.

Explanation for Incorrect Answer D :

re
Choice (D) is incorrect. Although the author expresses embarrassment, the point of
te

lines 63-66 is not to show her unease, but to show the reactions of everyone in the
room to her mother's story.


is
Explanation for Incorrect Answer E :
Choice (E) is incorrect. Lines 63-66 do not clarify a previous statement; instead,
they indicate the listeners' reactions to the mother's story.


eg


18
nR

The final paragraph presents the authors assessment of her mothers story as a


(A) pronouncement about forgiveness and understanding
(B) lesson and forewarning for school officials


(C) personal argument for the importance of hard work
U

(D) defense of a theory about social attitudes

(E) parting message to her daughter

ANSWERS AND EXPLANATIONS


Explanation for Correct Answer B :
Choice (B) is correct. In the final paragraph, the author calls her mother's story "a
message about her maternal concerns, and a way to prove that racism was not
some vanquished enemy" (lines 69-71). She adds, "[w]hen I was in third grade,
Mama could do her maternal duty" (lines 73-74), and asks, "[w]ho at this new
school would stand up for her child in her stead?" (75-76). The author sums up the
purpose of her mother's story as a challenge to the school officials to protect her
daughter as fiercely as she has done and will, if necessary, continue to do.

Explanation for Incorrect Answer A :


Choice (A) is incorrect. The author indicates that her mother's story was "an answer
(part rebuke and part condolence) to the school stories that the admissions people
told, where no parents figured at all" (lines 67-69). Forgiveness and understanding
do not figure prominently in the mother's story.

Explanation for Incorrect Answer C :

file://E:\\c8.htm 2006-11-12
The Official SAT Online Course 16/16

Choice (C) is incorrect. The author sees the story in terms of her mother's
determination to defend her daughter rather than as her mother's way of conveying
her personal ideas on hard work.

Explanation for Incorrect Answer D :


Choice (D) is incorrect. Although the author describes a theory about a particular
social attitudeone that might imagine that "racism was not some vanquished
enemy, but a real, live person, up in your face, ready, for no apparent reason, to
mess with your kid" (lines 71-73)this theory is one part of her mother's story.
The author sees the story as her mother's way of expressing her reaction to this
theory rather than as a defense of the theory.

Explanation for Incorrect Answer E :


Choice (E) is incorrect. As the author sees it, the message of the mother's story is
intended for the school officials and not for her daughter.

19 A central purpose of the passage is to


(A) illustrate the character of the authors mother
(B) portray the admissions process for boarding schools at that time
(C) show the authors repressed hostility toward her mother


(D) comment on examples of racism in the United States

(E) reveal how the author became skeptical of human nature


d
ANSWERS AND EXPLANATIONS
Explanation for Correct Answer A :

re
Choice (A) is correct. The main point of the passage is to show what kind of person


te
the author's mother is.


is

Explanation for Incorrect Answer B :


eg

Choice (B) is incorrect. The boarding school admissions process is barely mentioned
in the passage.


Explanation for Incorrect Answer C :
nR

Choice (C ) is incorrect. Although the author feels embarrassed by her mother's


story, the passage does not suggest that she feels hostility, repressed or otherwise,


toward her mother. Rather, she feels admiration and thanks for her mother and her
actions.

Explanation for Incorrect Answer D :


Choice (D) is incorrect. The mother's story indicates that she is well aware of
racism; however, this is not the central purpose of the passage. The central
purpose of the passage is to describe the mother's response to life and to her
family.

Explanation for Incorrect Answer E :


Choice (E) is incorrect. The author does not indicate or suggest that she has
become skeptical of human nature.

Back to Score Report

Copyright 2006 The College Board. All rights reserved. Privacy Policy Terms of Use Contact Us

file://E:\\c8.htm 2006-11-12
The Official SAT Online Course 1/13

Help | Profile | My Organizer | My Bookmarks | Logout

Answers and Explanations

Test Sections Back to Score Report

Section 1 View Answers and Explanations


Section 2 Online - Practice Test #3

Section 3
1
Section 4 If what is the value of
Section 5
Section 7 (A)
Section 8 (B)
Section 9
(C)
Section 10
(D)

(E)

ANSWERS AND EXPLANATIONS



d
Explanation for Correct Answer B :

Choice (B) is correct. If

re
then


which simplifies to


te

is
Explanation for Incorrect Answer A :

Choice (A) is not correct. If were equal to then the expression would


eg

equal not


nR

Explanation for Incorrect Answer C :


Choice (C) is not correct. If were equal to then the expression would

equal not

Explanation for Incorrect Answer D :

Choice (D) is not correct. If were equal to then the expression would

be undefined, and not equal to

Explanation for Incorrect Answer E :

Choice (E) is not correct. If were equal to then the expression would

equal not

Which of the following is the best estimate of the length of segment on the
number line above?

file://E:\\c9.htm 2006-11-12
The Official SAT Online Course 2/13

(A)

(B)

(C)

(D)

(E)

ANSWERS AND EXPLANATIONS


Explanation for Correct Answer A :
Choice (A) is correct. Point on the number line has a value of about and
point has a value of about . The length of is approximately
.

Explanation for Incorrect Answer B :


Choice (B) is not correct. Point on the number line has a value of about
and point has a value of about . While is close to the length of
the value is a better estimate.



Explanation for Incorrect Answer C :

d
Choice (C) is not correct. Point on the number line has a value of about
and point has a value of about . The length of is greater than the
distance of between the points labeled
better estimate.
re
and on the number line, so is a


te
Explanation for Incorrect Answer D :
Choice (D) is not correct. Since point is to the left of and point is to the
right of the distance between the two points must be at least so is a


is
better estimate.

Explanation for Incorrect Answer E :


eg

Choice (E) is not correct. Since point is to the left of and point is to the
right of the distance between the two points must be at least so is a


better estimate.
nR

3

For which of the following lists of numbers is the average (arithmetic mean) less


than the median?
U

(A)

(B)

(C)

(D)

(E)

ANSWERS AND EXPLANATIONS


Explanation for Correct Answer A :
Choice (A) is correct. The median is the middle number when the numbers are
listed in order, so the median is The average is

The question asks for a list of numbers

for which the average is less than the median, and is less than

Explanation for Incorrect Answer B :


Choice (B) is not correct. The median is the middle number when the numbers

file://E:\\c9.htm 2006-11-12
The Official SAT Online Course 3/13

are listed in order, so the median is The average is

For this list, the average is equal to the

median.

Explanation for Incorrect Answer C :


Choice (C) is not correct. The median is the middle number when the numbers
are listed in order, so the median is The average is

For this list, the average is equal to the

median.

Explanation for Incorrect Answer D :


Choice (D) is not correct. The median is the middle number when the numbers
are listed in order, so the median is The average is

For this list, the average is equal to the

median.

Explanation for Incorrect Answer E :


Choice (E) is not correct. The median is the middle number when the numbers
are listed in order, so the median is The average is


For this list, the mean is greater than

the median.


d
4

re

te

is


eg

If the perimeter of the rectangle above is what is the value of

(A)


nR

(B)


(C)

(D)

(E)

ANSWERS AND EXPLANATIONS


Explanation for Correct Answer C :
Choice (C) is correct. The formula for the perimeter of this rectangle is
where is the length and is the width. Therefore,
so and

Explanation for Incorrect Answer A :


Choice (A) is not correct. This would be the value of if were the area of the
rectangle rather than the perimeter.

Explanation for Incorrect Answer B :


Choice (B) is not correct. If were equal to then the perimeter of the
rectangle would be This cannot be
the case since it is given that the perimeter is

Explanation for Incorrect Answer D :


Choice (D) is not correct. If were equal to then the perimeter of the

file://E:\\c9.htm 2006-11-12
The Official SAT Online Course 4/13

rectangle would be This cannot be


the case since it is given that the perimeter is

Explanation for Incorrect Answer E :


Choice (E) is not correct. If were equal to then the perimeter of the
rectangle would be This cannot
be the case since it is given that the perimeter is

5 For which of the following functions is it true that

(A)

(B)

(C)

(D)

(E)

ANSWERS AND EXPLANATIONS




d
Explanation for Correct Answer D :
Choice (D) is correct. For this function, and

e
This is the only choice where
er

st


Explanation for Incorrect Answer A :
i

Choice (A) is not correct. For this function, and so


eg


Explanation for Incorrect Answer B :
nR

Choice (B) is not correct. For this function, and


so


Explanation for Incorrect Answer C :
U

Choice (C) is not correct. For this function, and

so

Explanation for Incorrect Answer E :


Choice (E) is not correct. For this function, and

so

Wayne would like to buy a school jacket priced at but the price of the jacket is
more than he has. In which of the following equations does represent the
number of dollars Wayne has?

(A)

(B)

(C)

(D)

(E)

file://E:\\c9.htm 2006-11-12
The Official SAT Online Course 5/13

ANSWERS AND EXPLANATIONS


Explanation for Correct Answer D :
Choice (D) is correct. Since the jacket costs more than Wayne has,
Wayne has dollars. Thus, if represents the number of dollars Wayne
has, then or

Explanation for Incorrect Answer A :


Choice (A) is not correct. For this equation to be true would have to be a
negative number. However, Wayne has a positive amount of money.

Explanation for Incorrect Answer B :


Choice (B) is not correct. In this equation, represents the number of dollars
Wayne would have if the price of the jacket were less than the amount Wayne
has. However, the jacket costs more than the amount of money that Wayne
has.

Explanation for Incorrect Answer C :


Choice (C) is not correct. For this equation to be true would have to be a


negative number. However, Wayne has a positive amount of money.

Explanation for Incorrect Answer E :


Choice (E) is not correct. In this equation, represents the number of dollars

ed
Wayne would have if the price of the jacket were less than the amount Wayne
has. However, the jacket costs more than the amount of money that Wayne


has.

r
te
7


is
Which of the following must be true for all integers and


eg

I.

II.


nR

III.


(A) I only

(B) II only
(C) III only

(D) I and II

(E) II and III

ANSWERS AND EXPLANATIONS


Explanation for Correct Answer A :
Choice (A) is correct. Subtracting zero from any integer will always result in the
integer so I must be true. On the other hand, does not hold for
all integers. For example, if and while
so II is not true. Also, does not hold
for all integers. For example, if and then
while so
III is not true.

Explanation for Incorrect Answer B :


Choice (B) is not correct. II is not true for all integers. For example, if and
then while

file://E:\\c9.htm 2006-11-12
The Official SAT Online Course 6/13

Explanation for Incorrect Answer C :


Choice (C) is not correct. III is not true for all integers. For example, if
and then while

Explanation for Incorrect Answer D :


Choice (D) is not correct. I is true, but II is not true for all integers. For example, if
and then while

Explanation for Incorrect Answer E :


Choice (E) is not correct. Neither II nor III is true for all integers. For example, if
and then while and
while

8
If and if and are positive numbers, then

(A)


(B)


(C)

d
(D)

(E)
re

te

ANSWERS AND EXPLANATIONS
is

Explanation for Correct Answer D :


eg

Choice (D) is correct. If and and are positive, then


nR

Explanation for Incorrect Answer A :

Choice (A) is not correct. If were equal to then would be equal to

which is not the same expression as the given

Explanation for Incorrect Answer B :


Choice (B) is not correct. If were equal to then would be equal to

which is not the same expression as the given

Explanation for Incorrect Answer C :

Choice (C) is not correct. If were equal to then would be equal to

which is not the same expression as the given

Explanation for Incorrect Answer E :

Choice (E) is not correct. If were equal to then would be equal to

which is not the same expression as the given

file://E:\\c9.htm 2006-11-12
The Official SAT Online Course 7/13

9
On a map, the length of the road from Town to Town is measured to be

inches. On this map, inch represents an actual distance of miles. What is the

actual distance, in miles, from Town to Town along this road?


(A)

(B)

(C)

(D)

(E)

ANSWERS AND EXPLANATIONS


Explanation for Correct Answer A :

Choice (A) is correct. Since inch represents miles, inch represents

miles. Therefore, inches represent miles.



ed
Explanation for Incorrect Answer B :

Choice (B) is not correct. Since


inch represents miles, inch represents
er

miles. If the actual distance were miles, it would be represented
st
on the map by

or inches, which is less than inches, the


i


eg

value given in the problem.

Explanation for Incorrect Answer C :


Choice (C) is not correct. Since inch represents miles, inch represents
nR


miles. If the actual distance were miles, it would be represented

on the map by inches, which is less than inches, the value given in

the problem.

Explanation for Incorrect Answer D :

Choice (D) is not correct. Since inch represents miles, inch represents

miles. If the actual distance were miles, it would be represented

on the map by or inches, which is less than inches, the

value given in the problem.

Explanation for Incorrect Answer E :

Choice (E) is not correct. Since inch represents miles, inch represents

miles. If the actual distance were miles, it would be represented

on the map by or inches, which is less than inches, the

value given in the problem.

file://E:\\c9.htm 2006-11-12
The Official SAT Online Course 8/13

10

In the figure above, if the area of triangle is equal to the area of rectangle
what is the length of segment


(A)

(B)


d
(C)

(D)

(E)
re

te
ANSWERS AND EXPLANATIONS


is

Explanation for Correct Answer C :


Choice (C) is correct. The area of rectangle is The area of a


eg

triangle is where is the length of the base of the triangle and is the


height of the triangle. Therefore, the area of triangle is equal to
nR

Since the areas of the triangle and the rectangle are the same,

This simplifies to Thus, line segment


has length
U

Explanation for Incorrect Answer A :


Choice (A) is not correct. The area of rectangle is The area of

triangle is equal to If the length of line segment

were equal to then the area of the triangle would equal not

Explanation for Incorrect Answer B :


Choice (B) is not correct. The area of rectangle is The area of

triangle is equal to If the length of line segment

were equal to then the area of the triangle would equal not

Explanation for Incorrect Answer D :


Choice (D) is not correct. The area of rectangle is The area of

triangle is equal to If the length of line segment

were equal to then the area of the triangle would equal not

file://E:\\c9.htm 2006-11-12
The Official SAT Online Course 9/13

Explanation for Incorrect Answer E :


Choice (E) is not correct. The area of rectangle is The area of triangle

is equal to If the length of line segment were equal

to then the area of the triangle would equal not

11 Six points are placed on a circle. What is the greatest number of different lines that
can be drawn so that each line passes through two of these points?
(A)

(B)

(C)

(D)

(E)

ANSWERS AND EXPLANATIONS


Explanation for Correct Answer B :


Choice (B) is correct. Five lines can be drawn from each point through another point
on the circle. You can draw different lines from the first point, but only
different lines from the second point, since one of the lines to that point has already


been drawn. Thus, the greatest number of lines that can be drawn so that each line

ed
passes through two of these points is

r

te

is


eg

Explanation for Incorrect Answer A :


Choice (A) is not correct. It is possible to draw more than different lines so that
nR

each line passes through two of the points. The figure shows the lines that can
be drawn.


Explanation for Incorrect Answer C :


Choice (C) is not correct. There are only lines that can be drawn, as shown in
U

the figure.

Explanation for Incorrect Answer D :


Choice (D) is not correct. You can draw lines from each of the points, but
since the line through a point and another point, is the same as the line
through point and point there are only different lines that can be drawn,
as shown in the figure.

Explanation for Incorrect Answer E :


Choice (E) is not correct. There are only different lines that can be drawn, as
shown in the figure.

12

file://E:\\c9.htm 2006-11-12
The Official SAT Online Course 10/13

Point is the point with the greatest -coordinate on the semicircle shown above.
What is the -coordinate of point

(A)

(B)

(C)

(D)

(E)

ANSWERS AND EXPLANATIONS


Explanation for Correct Answer C :
Choice (C) is correct. Since has the greatest -coordinate on the semicircle,
the radius of the circle equals the -coordinate of which is equal to The
center of the circle has the same -coordinate as so the center has coordinates
The distance between the center of the circle and is the radius of the
circle, so the -coordinate of is equal to



d
Explanation for Incorrect Answer A :
Choice (A) is not correct. The center of the circle has coordinates and the
circle has radius If the

re
-coordinate of
between the center of the circle to the point

were then the distance
on the circle would be equal to
te

which is greater than the radius. Therefore, the -coordinate of cannot be
equal to


is
Explanation for Incorrect Answer B :
Choice (B) is not correct. The center of the circle has coordinates and the


eg

circle has radius If the -coordinate of were then the distance between
the center of the circle to the point on the circle would be equal to which is
greater than the radius. Therefore, the -coordinate of cannot be equal to


nR


Explanation for Incorrect Answer D :
Choice (D) is not correct. The center of the circle has coordinates and the
circle has radius If the -coordinate of were then the distance between

the center of the circle to the point on the circle would be equal to which is
less than the radius. Therefore, the -coordinate of cannot be equal to

Explanation for Incorrect Answer E :


Choice (E) is not correct. The center of the circle has coordinates and the
circle has radius If the -coordinate of were then the distance
between the center of the circle to the point on the circle would be equal to
which is less than the radius. Therefore, the -coordinate of cannot be equal to

13 A garden has parallel rows of plants, with plants in each row. If plants are
added to each row, how many plants will then be in the garden, in terms of and

(A)

(B)

(C)

(D)

(E)

file://E:\\c9.htm 2006-11-12
The Official SAT Online Course 11/13

ANSWERS AND EXPLANATIONS


Explanation for Correct Answer C :
Choice (C) is correct. The total number of plants will be the number of rows,
multiplied by the total number of plants per row, which gives

Explanation for Incorrect Answer A :


Choice (A) is not correct. This expression corresponds to a situation in which the
number of plants per row is multiplied by The question asks for an expression
that corresponds to adding plants to each row of the garden.

Explanation for Incorrect Answer B :


Choice (B) is not correct. This expression corresponds to the situation in which
plants are added to the entire garden. The question asks for an expression that
corresponds to adding plants to each row of the garden.

Explanation for Incorrect Answer D :


Choice (D) is not correct. This expression corresponds to the situation in which
rows with plants each are added to the garden. The question asks for an


expression that corresponds to adding plants to each row of the garden.

Explanation for Incorrect Answer E :


d
Choice (E) is not correct. The number of plants per row in the garden, needs to
be multiplied by the number of rows, to get the total number of plants in the

re
garden before new plants are added to each row. The expression does


not involve this necessary multiplication of and


te
14 Three lines are drawn in a plane so that there are exactly three different intersection


points. Into how many non-overlapping regions do these lines divide the plane?
is

(A) Three


eg

(B) Four
(C) Five


(D) Six
nR

(E) Seven


ANSWERS AND EXPLANATIONS

Explanation for Correct Answer E :


Choice (E) is correct. Since there are three different intersection points, the three
lines form a triangle. They also separate the remaining space in the plane into six
regions, for a total of seven non-overlapping regions. See figure below.

Explanation for Incorrect Answer A :


Choice (A) is not correct. The lines separate the plane into more than three regions.
The figure shows the plane divided into seven regions.

Explanation for Incorrect Answer B :


Choice (B) is not correct. The lines separate the plane into more than four regions.
The figure shows the plane divided into seven regions.

Explanation for Incorrect Answer C :

file://E:\\c9.htm 2006-11-12
The Official SAT Online Course 12/13

Choice (C) is not correct. The lines separate the plane into more than five regions.
The figure shows the plane divided into seven regions

Explanation for Incorrect Answer D :


Choice (D) is not correct. The lines separate the plane into more than six regions.
The figure shows the plane divided into seven regions.

15

In the figure above, side of is on line What is in terms of

(A)


ed
(B)

(C)

(D)

(E) r

te

ANSWERS AND EXPLANATIONS
is

Explanation for Correct Answer A :


eg

Choice (A) is correct. The measure of is and the


measure of is Therefore,


so and
nR


Explanation for Incorrect Answer B :

Choice (B) is not correct. If then which means


that must be and that must be a right triangle.
However, there are other values of that satisfy the conditions given in the figure.

Explanation for Incorrect Answer C :


Choice (C) is not correct. If then
which means that must be and that must be an equilateral triangle.
However, there are other values of that satisfy the conditions given in the figure.

Explanation for Incorrect Answer D :


Choice (D) is not correct. If then
which is equivalent to the false statement Therefore, cannot equal

Explanation for Incorrect Answer E :


Choice (E) is not correct. If then
which means that must be and
but from the figure, is positive. Therefore, cannot equal

16

file://E:\\c9.htm 2006-11-12
The Official SAT Online Course 13/13

If denotes the sum of the integers from to inclusive, and denotes the sum
of the integers from to inclusive, what is the value of

(A)

(B)

(C)

(D)

(E)

ANSWERS AND EXPLANATIONS


Explanation for Correct Answer E :
Choice (E) is correct. If and then
and Another way to solve this
problem is to notice that is the sum

of numbers each of which is equal to so equals

Explanation for Incorrect Answer A :


Choice (A) is not correct. Since and
it is not true that


ed
Explanation for Incorrect Answer B :


Choice (B) is not correct. Since and
er it is not true that

Explanation for Incorrect Answer C :


Choice (C) is not correct. Since and
it is not true that
t

is

Explanation for Incorrect Answer D :


eg

Choice (D) is not correct. Since and it is not true that


nR



Back to Score Report
U

Copyright 2006 The College Board. All rights reserved. Privacy Policy Terms of Use Contact Us

file://E:\\c9.htm 2006-11-12
The Official SAT Online Course 1/9

Help | Profile | My Organizer | My Bookmarks | Logout

Answers and Explanations

Test Sections Back to Score Report

Section 1 View Answers and Explanations


Section 2 Online - Practice Test #3

Section 3
1 A recent discovery is the finding that people who both drink and smoke are greater
Section 4 cancer risks than those who do only one of these things.
Section 5
(A) A recent discovery is the finding that
Section 7
Section 8 (B) A recent discovery came when they learned that
Section 9 (C) Recently, a finding is that
Section 10
(D) It has recently been discovered that

(E) It is a recent discovery that

ANSWERS AND EXPLANATIONS




d
Explanation for Correct Answer D :
Choice (D) is correct. It avoids the error of the original by eliminating the
unnecessary phrase "the finding."

re

te
Explanation for Incorrect Answer A :


is
Choice (A) displays wordiness. The phrase "the finding" is not needed, since it
merely repeats the idea of the earlier noun "discovery."


eg

Explanation for Incorrect Answer B :


Choice (B) uses a vague pronoun. The sentence contains no noun to which the
pronoun "they" can refer.


nR

Explanation for Incorrect Answer C :


Choice (C) involves an inappropriate idiom. In context, the adverb "recently" is less


effective than the adjective "recent" ("A recent finding is that").

Explanation for Incorrect Answer E :

Choice (E) contains an unnecessary word. With a minor change in word order ("A
recent discovery is that"), the vague pronoun "it" can be deleted.

2
The notion that a biography should be full of praise and free of criticism prevailed
during most of the nineteenth century.

(A) The notion that a biography should be full of praise and free of criticism
prevailed during most of the nineteenth century.
(B) The notion that prevailed about a biography during most of the nineteenth
century was that of being full of praise and free of criticism.
(C) During most of the nineteenth century, they had a prevalent notion that a
biography should be full of praise and free of criticism.
(D) Prevalent as a notion during most of the nineteenth century was for a
biography to be full of praise and free of criticism.
(E) Prevalent during most of the nineteenth century, the notion prevailed that
a biography should be full of praise and free of criticism.

ANSWERS AND EXPLANATIONS


Explanation for Correct Answer A :
Choice (A) is correct. It uses appropriate words to express a complete idea.

file://E:\\c10.htm 2006-11-12
The Official SAT Online Course 2/9

Explanation for Incorrect Answer B :


Choice (B) uses an inappropriate idiom. The phrase "that of being" is not suitable to
introduce a phrase describing the prevalent notion.

Explanation for Incorrect Answer C :


Choice (C) includes a vague pronoun. Since the sentence contains no noun to which
the pronoun "they" can refer, its meaning is not clear.

Explanation for Incorrect Answer D :


Choice (D) creates a sentence fragment. Without a noun, a phrase, or a clause to
serve as the subject of the verb "was," this group of words does not state a
complete thought.

Explanation for Incorrect Answer E :


Choice (E) displays wordiness. The adjective "prevalent" is not needed, since it
conveys the same idea as the verb "prevailed."

3 A native New Yorker, Gloria Naylors first novel won an American Book Award in
1983.
(A) A native New Yorker, Gloria Naylors first novel
(B)


A native New Yorker, the first novel by Gloria Naylor
(C) The first novel by native New Yorker Gloria Naylor


ed
(D) Gloria Naylor, a native New Yorker, wrote her first novel thus having

(E) Gloria Naylor wrote her first novel and the native New Yorker

ANSWERS
r
AND EXPLANATIONS

te
Explanation for Correct Answer C :


Choice (C) is correct. It avoids the dangling modifier of the original by using a
is
correctly placed prepositional phrase ("by native New Yorker Gloria Naylor") to
embed details describing the "first novel."


eg


nR

Explanation for Incorrect Answer A :


Choice (A) contains a dangling noun phrase. Although the two noun phrases ("A


native New Yorker" and "Gloria Naylors first novel") may appear at first to be
parallel appositives, the first phrase refers to a person and the second to that
person's work. Thus, the first phrase has no grammatical connection to the rest of


the sentence.
U

Explanation for Incorrect Answer B :


Choice (B) displays improper word order. Since the noun phrase "A native New
Yorker" obviously refers to a person (not to that person's "first novel"), the phrase
should be placed immediately before or after the name "Gloria Naylor."

Explanation for Incorrect Answer D :


Choice (D) uses improper tense sequence. The present perfect tense of the verbal
"having won" illogically suggests that Naylor received the award before she wrote
the novel.

Explanation for Incorrect Answer E :


Choice (E) exhibits ineffective coordination. The coordinating conjunction "and"
merely links two complete thoughts without appropriate subordination or
embedding to indicate their relationship and relative importance.

4 Charlie Chaplin developed definite ideas about the art of comedy and as a result
sentiment, satire, and social criticism were introduced into his work.

(A) sentiment, satire, and social criticism were introduced


(B) sentiment, satire, and social criticism were introduced by him
(C) having introduced sentiment, satire, and social criticism

file://E:\\c10.htm 2006-11-12
The Official SAT Online Course 3/9

(D) introduced sentiment, satire, and social criticism

(E) the introduction of sentiment, satire, and social criticism

ANSWERS AND EXPLANATIONS


Explanation for Correct Answer D :
Choice (D) is correct. It avoids the error of the original by using two parallel active
verbs, "developed" and "introduced," with "Charlie Chaplin" as their subject.

Explanation for Incorrect Answer A :


Choice (A) fails to maintain parallelism. The first independent clause uses active
voice ("Charlie Chaplin developed"), but the second shifts awkwardly to passive
voice ("were introduced").

Explanation for Incorrect Answer B :


Choice (B) uses excess words. The phrase "by him" is not needed.

Explanation for Incorrect Answer C :


Choice (C) exhibits improper modification. The phrase "having introduced


sentiment, satire, and social criticism" cannot coherently modify any part of the
sentence, and it incorrectly suggests that this introduction came before the
development of Chaplin's ideas about comedy.


d
Explanation for Incorrect Answer E :

re
Choice (E) violates parallelism. Since the noun "introduction" is not parallel with the


earlier clause, "Charlie Chaplin developed definite ideas," the second half of the
sentence has no verb and is left awkwardly incomplete.


te

5
is
Someone living in a technological, consumption-oriented culture probably taxes the
environment at a rate many times that of a country such as Myanmar.

(A) that of a country such as Myanmar


eg

(B) that of someone living in a country like Myanmar


(C) what you find in Myanmar, for instance


nR

(D) the rate in a country such as Myanmar


(E) a citizen of Myanmar, for instance


ANSWERS AND EXPLANATIONS
U

Explanation for Correct Answer B :


Choice (B) is correct. It avoids the error of the original by adding the phrase
"someone living in" so that the effects of the actions of two individuals in two
different countries are compared.

Explanation for Incorrect Answer A :


Choice (A) presents an illogical comparison. The effect of a single person
("someone") on the environment is compared to the effect of an entire country
("Myanmar").

Explanation for Incorrect Answer C :


Choice (C) has an inappropriate pronoun. The pronoun "you" is used loosely to
mean any person.

Explanation for Incorrect Answer D :


Choice (D) suggests an illogical comparison. The rate at which an individual taxes
the environment in a technological culture is compared to the rate in the whole
country of Myanmar.

Explanation for Incorrect Answer E :


Choice (E) makes an illogical comparison. Without the phrase "that of" immediately
before "a citizen of Myanmar," the statement compares a rate (of taxing the

file://E:\\c10.htm 2006-11-12
The Official SAT Online Course 4/9

environment) with a person.

6 Airport runways must be constantly swept clear of trash and other debris that could
be sucked into a jet-engine intake or it could cause a serious accident.
(A) intake or it could cause a serious accident
(B) intake, this causes a serious accident
(C) intake and cause a serious accident

(D) intake, preventing a serious accident

(E) intakes and avoiding a serious accident

ANSWERS AND EXPLANATIONS


Explanation for Correct Answer C :
Choice (C) is correct. It avoids the error of the original by using the appropriate
conjunction "and" to link a clear cause (debris being sucked into a jet engine) with
its expected effect (a serious accident).

Explanation for Incorrect Answer A :



d
Choice (A) contains the erroneous singular pronoun "it," which appears to refer to
the plural antecedents "trash and other debris."

Explanation for Incorrect Answer B :

re
Choice (B) displays incorrect coordination. A comma is used improperly to join two
complete thoughts ("Airport runways . . . into a jet-engine intake" and "this causes


te
a serious accident").

Explanation for Incorrect Answer D :


is
Choice (D) exhibits improper modification. The dangling phrase, "preventing a
serious accident," apparently indicates a reason for sweeping runways, but it is not
in a position to modify correctly any part of the sentence.


eg

Explanation for Incorrect Answer E :


Choice (E) violates parallelism. The verbal "avoiding" is neither grammatically nor


logically parallel with the earlier verb phrase "could be sucked," since this earlier
nR

phrase describes a likely cause of an accident, not a method of preventing one.



7 In believing that firsthand experience would enhance the credibility of his biography
U

of Columbus, Professor Morison retraced the route of Columbus first voyage.

(A) In believing that


(B) Believing that
(C) In his belief that

(D) He believed that

(E) By believing that

ANSWERS AND EXPLANATIONS


Explanation for Correct Answer B :
Choice (B) is correct. It avoids the error of the original by eliminating the
unnecessary preposition "in."

Explanation for Incorrect Answer A :


Choice (A) exhibits wordiness. The preposition "in" is not needed.

Explanation for Incorrect Answer C :


Choice (C) displays wordiness. Four words ("In his belief that") can be reduced to
two ("Believing that").

file://E:\\c10.htm 2006-11-12
The Official SAT Online Course 5/9

Explanation for Incorrect Answer D :


Choice (D) involves improper coordination. Two complete thoughts ("He believed . .
. of Columbus" and "Professor Morrison retraced . . . first voyage") are joined with
only a comma.

Explanation for Incorrect Answer E :


Choice (E) uses an inappropriate idiom. The preposition "by" incorrectly suggests a
means by which Morrison retraced the route, rather than the mental state in which
he did so.

8 Except in mathematics, absolute proof is more often an ideal to be sought than a goal
to be reached, a fact that the courts recognize by setting varying standards of proof
for different kinds of cases.
(A) a fact that the courts recognize by setting
(B) which the courts recognize and set
(C) and this is recognized when the courts are setting

(D) and it is recognized by the courts when they set

(E) and the courts recognize this fact setting

ANSWERS AND EXPLANATIONS



d
Explanation for Correct Answer A :
Choice (A) is correct. It uses a specific noun, "fact," instead of a vague and
ambiguous pronoun.

re

te
Explanation for Incorrect Answer B :


is
Choice (B) uses a vague pronoun. Since the pronoun "which" refers loosely to the
idea expressed in the entire preceding clause (but not to any specific noun), the
meaning is not clear.


eg

Explanation for Incorrect Answer C :


Choice (C) contains a vague pronoun. The meaning of the pronoun "this" remains


unclear, since it apparently refers to the general idea expressed in the preceding
nR

clause, not to any specific noun in the sentence.


Explanation for Incorrect Answer D :
Choice (D) exhibits vague and ambiguous pronoun use. Since the pronoun "it"
cannot logically refer to any of the singular nouns in the sentence ("proof," "ideal,"

or "goal"), its meaning (apparently the general idea expressed in the preceding
clause) is not clear.

Explanation for Incorrect Answer E :


Choice (E) displays ineffective coordination. Using the conjunction "and" to connect
two independent clauses is grammatically correct, but such a linkage does not
indicate the logical relationship between the two clauses.

9
My grandfather never learned to use a calculator, as he shops he can accurately
compute his grocery bill in his head to within a dollar.

(A) as he shops
(B) while shopping
(C) but as he shops

(D) therefore, when shopping

(E) however, he shops so that

ANSWERS AND EXPLANATIONS


Explanation for Correct Answer C :
Choice (C) is correct. It avoids the error of the original by adding an appropriate

file://E:\\c10.htm 2006-11-12
The Official SAT Online Course 6/9

conjunction, "but," to join the two independent clauses that express contrasting
ideas.

Explanation for Incorrect Answer A :


Choice (A) displays improper coordination. It uses only a comma to join two
complete thoughts ("My grandfather . . . calculator" and "as he shops . . . within a
dollar").

Explanation for Incorrect Answer B :


Choice (B) uses improper coordination. Two complete thoughts ("My grandfather . .
. calculator" and "while shopping . . . within a dollar") are joined by only a comma.

Explanation for Incorrect Answer D :


Choice (D) exhibits improper coordination. Since the transition word "therefore"
does not serve as a coordinating conjunction, this choice incorrectly links two
complete thoughts ("My grandfather . . . calculator" and "therefore, when
shopping . . . within a dollar") with only a comma.

Explanation for Incorrect Answer E :


Choice (E) results in improper coordination. Two independent clauses ("My
grandfather . . . calculator" and "however, he shops . . . within a dollar") are
connected by only a comma.



ed
10 The first African American woman to win the Pulitzer Prize for poetry, Gwendolyn
Brooks with her special interest in encouraging young poets.

(A)

(B) r
had a special interest in encouraging
with her special interest in encouraging


te
(C) having had a special interest, which was to encourage

(D) who had a special interest in encouraging


is
(E) she had a special interest to encourage


eg

ANSWERS AND EXPLANATIONS


Explanation for Correct Answer B :
nR

Choice (B) is correct. It avoids the error of the original by providing a verb, "had,"
so that the sentence states a complete idea.

Explanation for Incorrect Answer A :


Choice (A) produces a sentence fragment. With no verb (only the verbal phrases "to
win" and "encouraging"), the main clause fails to state a complete idea.

Explanation for Incorrect Answer C :


Choice (C) creates a sentence fragment. Although this choice has a verb, "was," the
verb is part of a dependent clause introduced by "which." The main clause has no
verb to complete the thought.

Explanation for Incorrect Answer D :


Choice (D) is a sentence fragment. The only verb, "had," is part of a dependent
clause ("who had a special interest in encouraging young poets"), so the main
clause cannot state a complete thought.

Explanation for Incorrect Answer E :


Choice (E) includes an excess word. The pronoun "she" simply repeats the name
immediately before it, "Gwendolyn Brooks," and is, therefore, unnecessary.

11 Although fascinated by chance and coincidence, Paul Austers novels are written
with careful attention to style and balance.

(A) Paul Austers novels are written

file://E:\\c10.htm 2006-11-12
The Official SAT Online Course 7/9

(B) Paul Austers novels were written


(C) Paul Auster writes his novels

(D) Paul Auster is a writer

(E) Paul Auster had wrote

ANSWERS AND EXPLANATIONS


Explanation for Correct Answer C :
Choice (C) is correct. It avoids the error of the original by placing the proper noun
"Paul Auster" immediately after the introductory phrase that modifies it.

Explanation for Incorrect Answer A :


Choice (A) displays improper modification. The introductory phrase, "Although
fascinated by chance and coincidence," cannot logically modify the noun that
follows, "novels," since it clearly describes the author rather than his books.

Explanation for Incorrect Answer B :


Choice (B) exhibits improper modification. The introductory phrase, "Although


fascinated by chance and coincidence," logically modifies the author Paul Auster,
not the novels that he wrote.


d
Explanation for Incorrect Answer D :
Choice (D) involves inappropriate modification. The prepositional phrase, "with

re
careful attention to style and balance," functions as an adverb telling how the


novels were written, not as an adjective describing the preceding noun, "writer."

Explanation for Incorrect Answer E :


te
Choice (E) uses an incorrect verb form. After the helping verb "had," the correct
form of this irregular verb is the past participle, "written," not the past, "wrote."


is


eg

12
Early American factories did not so much replace household manufacturing but
complement it.

(A)


but complement
nR

(B) as complement


(C) but they complemented

(D) and they complemented

(E) as they were to complement

ANSWERS AND EXPLANATIONS


Explanation for Correct Answer B :
Choice (B) is correct. It avoids the error of the original by using the appropriate
conjunction "as" to complete the idea of the introductory phrase "so much" and
thus properly link two parallel verbs ("replace" and "complement").

Explanation for Incorrect Answer A :


Choice (A) uses an inappropriate idiom. After the phrase "so much," the conjunction
"but" is improper because it suggests not a limited degree of difference but a more
complete contrast.

Explanation for Incorrect Answer C :


Choice (C) contains an idiom that is not suitable. The conjunction "but" is not
appropriate for use with the phrase "so much" because "but" implies a more
absolute contrast.

Explanation for Incorrect Answer D :


Choice (D) employs an improper idiom. The conjunction "and" suggests no contrast
but simply indicates that additional information will follow.

file://E:\\c10.htm 2006-11-12
The Official SAT Online Course 8/9

Explanation for Incorrect Answer E :


Choice (E) displays wordiness. The phrase "they were to" is unnecessary.

13 After teaching, becoming involved in several fashion enterprises, and after she
founded the Harlem Institute of Fashion, Lois Alexander Lane launched the Black
Fashion Museum.
(A) after she founded
(B) after the founding of
(C) founding

(D) she had founded

(E) having founded

ANSWERS AND EXPLANATIONS


Explanation for Correct Answer C :
Choice (C) is correct. It avoids the error of the original by using a gerund,
"founding," that is parallel with the earlier gerunds, "teaching" and "becoming."



ed
Explanation for Incorrect Answer A :
Choice (A) fails to maintain parallelism. The clause "after she founded" is not
parallel with two earlier items in the seriesthe gerunds "teaching" and


"becoming." er
Explanation for Incorrect Answer B :


Choice (B) uses excess words. The words "after," "the," and "of" are not needed.
st
Explanation for Incorrect Answer D :


Choice (D) violates parallelism. The clause "she had founded" is not parallel with
the gerunds "teaching" and "becoming"the two earlier items in the series.
i


eg

Explanation for Incorrect Answer E :


Choice (E) has an error in tense sequence. The present perfect tense of the verbal
"having founded" is not consistent with the present tense of the earlier verbals


"teaching" and "becoming."
nR

14

In eighteenth-century France, economic inequalities made many people angry, and a


violent revolution was fueled.
U

(A) angry, and a violent revolution was fueled


(B) angry; it fueled a violent revolution
(C) angry, and this anger fueled a violent revolution

(D) angry, that anger fueled a violent revolution

(E) angry; thus fueling a violent revolution

ANSWERS AND EXPLANATIONS


Explanation for Correct Answer C :
Choice (C) is correct. It avoids the error of the original by using two parallel clauses
("economic inequalities made" and "this anger fueled"), both in active voice.

Explanation for Incorrect Answer A :


Choice (A) fails to maintain parallelism. The first independent clause uses active
voice ("economic inequalities made many people angry"), but the second shifts to
ineffective passive voice ("and a violent revolution was fueled").

Explanation for Incorrect Answer B :


Choice (B) contains a vague pronoun. The singular pronoun "it" cannot logically

file://E:\\c10.htm 2006-11-12
The Official SAT Online Course 9/9

refer to the noun "France" (the only singular noun that precedes it), and it does not
agree with the plural nouns "inequalities" and "people."

Explanation for Incorrect Answer D :


Choice (D) uses improper coordination. It uses only a comma to join two complete
thoughts ("In eighteenth-century France . . . many people angry" and "that anger
fueled a violent revolution").

Explanation for Incorrect Answer E :


Choice (E) exhibits inappropriate coordination. It incorrectly uses a semicolon to link
parts of unequal grammatical rank (a complete thought before the semicolon and
only a phrase after it).

Back to Score Report

Copyright 2006 The College Board. All rights reserved. Privacy Policy Terms of Use Contact Us



d
re

te

is


eg


nR

file://E:\\c10.htm 2006-11-12

Вам также может понравиться